Sie sind auf Seite 1von 63

21/05/13 17:56 Simulador Proedumed

Pgina 1 de 1 http://www.proedumed.com.mx/simulador/pages/examen/resultadoPregunta.faces
Anlisis del Caso Clnico
Identificacin del reactivo
Area: GINECOLOGA Y OBSTETRICIA
Especialidad: GINECOLOGA
Tema: ANTICONCEPCIN Y PLANIFICACIN FAMILIAR
Subtema: MTODOS DE PLANIFICACIN FAMILIAR, TEMPORALES Y
DEFINITIVOS
CASO CLNICO CON UNA PREGUNTA
MUJER DE 32 AOS DE EDAD GESTA 1, PARA 1, QUE CUENTA CON ANTECEDENTE DE EPILEPSIA DE DIFICIL CONTROL, POR LO QUE TOMA CARBAMAZEPINA Y FENITONA
PARA SU CONTROL. ACUDE A SU CONSULTORIO SOLICITANDO MTODO HORMONAL ORAL COMO MTODO DE PLANIFICACIN HORMONAL.
ELEMENTOS CLAVE A CONSIDERAR EN EL CASO CLNICO:
Edad: 32 aos
Antecedentes: g 1 p 1 epilepsia de dif cil control toma carbamazepina y fenito na
solicita mpf hormonal oral
Sintomatologa:
Exploracin:
Laboratorio y/o gabinete:
1 - USTED LE INDICAR QUE LA EFECTIVIDAD DE ESTE MTODO DADOS SUS ANTECEDENTES SE VERA AFECTADA YA QUE LOS MEDICAMENTOS QUE ELLA
TOMA:
REDUCEN LA
ABSORCIN
INTESTINAL DE
LAS HORMONAS
Una modificacin de la acidez gstrica o una modificacin de la motilidad
gastrointestinal pueden causar una menor absorcin de un medicamento
Cualquier medicamento que modifique el pH gstrico como los anticidos los
bloqueadores H2 el omeprazol y similares pueden hacer que un anticonceptivo
hormonal oral se absorba en menor cantidad Si esto ocurre por un periodo mayor
a una semana se puede llegar a presentar disminucin de los niveles plasmticos
de las hormonas anticonceptivas y podr a llegar a ocasionar falla anticonceptiva o
sangrado uterina antes de la fecha esperada Un medicamento que acelere la
motilidad gstrica como la metoclorpramida la domperidona y similares as como
los laxantes de cualquier tipo pueden tambin disminuir la absorcin de un
anticonceptivo hormonal oral Medicamentos de este tipo incluso pueden disminuir
la reabsorcin biliar de cualquier hormona esteroide que haga circulacin
enteroheptica y aumentar la excrecin fecal total
ACTAN COMO
ANTAGONISTAS
DE LOS
RECEPTORES
HORMONALES
Los ANTAGON STA EN LOS RECEPTORES DE PROGESTERONA (PRA) son
molculas de origen hormonal o no hormonal capaz de generar respuestas
antagonistas de progesterona sobre tejidos espec ficos Ejemplos de PRA son
mifepristone y onapristone Los antiestrgenos abarcan en su sentido ms
estrecho a la clase de sustancias de aquellos compuestos que pueden expulsar a
los estrgenos desde sus respectivos receptores (ANTAGON STAS DE LOS
RECEPTORES DE ESTRGENOS) y en su sentido ms amplio tambin a los
compuestos que impiden la s ntesis de estrgenos a partir de sus compuestos
Ejemplos de stos son tamoxifeno y raloxifeno precursores metablicos en el
organismo
SON
INDUCTORES DE
LAS ENZIMAS
HEPTICAS
MICROSOMALES
Las drogas inductoras de enzimas hepticas pueden reducir la eficacia de los
anticonceptivos orales combinados sin embargo si van a ser utilizados por largo
tiempo se debe considerar que ser afectados por las drogas inductoras de
enzimas hepticas LOS ANT CONVULS VOS NDUCEN LA PRODUCC N DE
ENZ MAS HEPT CAS POR LO QUE REDUCEN LA EF CAC A DE
ANT CONCEPT VOS ORALES
AUMENTAN LA
VELOCIDAD DE
LA EXCRECIN
RENAL DE LAS
HORMONAS
Durante los PROCESOS DE EXCREC N RENAL las hormonas sexuales pueden
ser excretadas ms rpido si la usuaria ha utilizado diurticos de cualquier tipo
Por el contrario la excrecin renal puede ser ms lenta si se ha utilizado un
bloqueador del transporte activo tubular como el probenecid
Bibliografa: FARMACOLOGA. RANG HP, DALE MM, RITTER JM, MOORE PK. ELSEVIE. EDICIN 5. PAG. 441.
21/05/13 17:59 Simulador Proedumed
Pgina 1 de 1 http://www.proedumed.com.mx/simulador/pages/examen/resultadoPregunta.faces
Anlisis del Caso Clnico
Identificacin del reactivo
Area: GINECOLOGA Y OBSTETRICIA
Especialidad: GINECOLOGA
Tema: CLIMATERIO Y MENOPAUSIA
Subtema: CLIMATERIO Y MENOPAUSIA
CASO CLNICO CON UNA PREGUNTA

MUJER DE 51 AOS DE EDAD, GESTA 4 PARA 4, QUE ACUDE A CONSULTA POR PRESENTAR BOCHORNOS NOCTURNOS, FALTA DE APETITO SEXUAL, AMENORREA E
INCONTINENCIA URINARIA.

ELEMENTOS CLAVE A CONSIDERAR EN EL CASO CLNICO:
Edad: Adu to de 51 aos
Antecedentes: -
Sintomatologa: presenta sntomas vasomotores y gen tour nar os compat b es con a
per menopaus a
Exploracin: -
Laboratorio y/o gabinete: -

2 - EL TRATAMIENTO INDICADO EN ESTE CASO SERA A BASE DE:

PROGESTGENOS Los progestagenos s os no estn nd cados para e tratamento de as
man festac ones gen tour nar as de a menopaus a S os son ef caces para e
tratamento de os bochornos en as mujeres en as que ex ste a guna
contra nd cac n para admn strar estrgenos como sucede cuando ex sten
antecedentes de tromboembo as venosas o cncer mamar o S n embargo su
admn strac n se ve mtada por a guno de sus efectos adversos como
hemorrag a vag na y aumento de peso Estaran nd cados so o en e caso de
que ex st era a guna contra nd cac n para a terap a con estrgenos
MDIDAS
HIGINICO
DIETTICAS
Las act v dades dest nadas a reduc r a temperatura corpora centra como e uso
de un aban co vest rse en capas y baarse con agua fra a v an tempora mente
a d afores s nocturna y os bochornos As msmo as tcn cas de re ajac n
como a resp rac n pausada d smnuyen os sntomas Tamb n a med tac n
e dejar de fumar y bajar de peso son t es a gua que e consumo de
a mentos y beb das fras S b en formarn parte de as nd cac ones que
deberemos dar a nuestras pac entes e tratamento nd cado es a terap a
hormona
ESTRGENOS
CON
PROGESTGENOS
OJO En as mujeres con tero os progestgenos se comb nan con estrgenos
para reduc r e r esgo de cncer endometr a Con base en a b b ografa actua
a hormonoterap a est nd cada n camente para e tratamento de os sntomas
vasomotores y a atrof a vag na y para preven r o tratar a osteoporos s Es
mportante va orar de nuevo a neces dad de tratamento a nterva os de se s a
doce meses La hormonoterap a se debe prescr b r a a menor dos s ef caz y por
e per odo ms corto Tratamento Hormona La Terap a Hormona (TH) ha
mostrado ser e tratamento de pr mera opc n para e contro de os sntomas
vasomotores y atrof a Urogen ta de C mater o Rev s ones c n cas muestran
que a TH reduce a frecuenc a de sntomas vasomotores de un 80% a 90%
requ r ndose genera mente poco t empo de uso por o que os r esgos son
bajos De acuerdo a a forma de admn strac n de estrgeno y a progesterona
ex sten d ferentes esquemas de TH comb nada Terap a Cc ca Cc co-
Comb nado Cont nuo Cc co (Secuenc a ) En Nuestro med o os esquemas ms
recomendados son: Terap a cc ca cont na (tamb n denomnada secuenc a ):
E estrgeno se ut za todos os das con e agregado de progesterona 10 a 14
das por mes Terap a cont nua comb nada: Emp ea dos s f jas de estrgeno-
progestgeno d ar amente E esquema de terap a hormona ser se ecc onado
de acuerdo a a etapa de c mater o; en etapa de trans c n y per menopaus a se
recomenda esquemas comb nados secuenc a es; en a posmenopaus a
esquema cont nuo comb nado La TH comb nada cons ste en a prescr pc n de
estrgenos y progestgeno para e contro de sndrome c matr co La TH
comb nada esta nd cada en mujeres con tero ntegro para reduc r e r esgo de
h perp as a o cncer de endometr o Los progestgenos pueden ser natura es o
s ntt cos Actua mente se cuenta con progesterona natura mcron zada y
d versos t pos de estero des s ntt cos de efecto parec do a a progesterona
denomnadas progest nas estos preparados pueden ser der vados de a
progesterona o de a testosterona La potenc a Progestac ona de os frmacos
sobre protecc n endometr a rad ca en a capac dad de transformac n
endometr a de acuerdo a su gran af n dad sobre recepetores progestac ona es
de acuerdo a esta potenc a o af n dad se encuentran en orden decendente:
Tr megestona Desogestre Nomegestro acetato Acetato de Medrox progesterona
Noret sterona Acetato de c proterona La TH cc ca est nd cada en mujeres con
tero en a Per menopaus a que desean cont nuar con c c os menstrua es
Cuando se admn stra en forma cc ca e progestgeno es recomendab e que
sea por o menos 12 a 14 das de cada mes Para a se ecc n de a progest na
se deber tomar en cuenta adems de protecc n endometr a su to eranc a y
su mpacto en e metabo smo y sus efectos mnera ocort co des
g ucocort co des y androgn cos Es de pr mera e ecc n a va transdrmca en
a pac ente con sndrome c matr co portadora de h pertens n arter a
h pertr g cer dema y/o hepatopata crn ca La admn strac n tp ca est
nd cada exc us vamente en atrof a gen tour nar a en estud os a eator zados se
ha demostrado su ef cac a y to erab dad En re ac n a a durac n de
tratamento a ev denc a actua sug ere que debe cont nuarse mentras os
benef c os se hayan estab ec do o que a cons derac n de a pac ente e
b enestar supere a os r esgos y efectos secundar os sobre todo despus de no
haber pod do nterrump r a terap a Actua mente e tratamento s empre es
comb nado
ESTRGENOS
SIMPLES
Los estrgenos tanto por va ora como ntravag na so os o asoc ados a
gestgenos han demostrado ser ef caces en e tratamento de os sntomas
vag na es La mayor ef cac a de tratamento por va vag na y as bajas dos s de
estrgenos admn stradas por esta va s n que ex sta aumento s gn f cat vo de
h perp as a o pro ferac n endometr a tanto en tab etas cremas vu os o
an os vag na es aconsejan su ut zac n frente a a terap a s stmca Una
rev s n Cochrane conc uy que e estrad o por va ntravag na en cremas
supos tor os vu os o an os es efect vo para e a v o de os sntomas de a
atrof a vag na S ut zamos estrgenos vag na es para a vag n t s atrf ca
s ntomt ca as dos s recomendada son: vu os de estr o (0 5 mg) d ar amente
durante 2-3 semanas y poster ormente un vu o 2 veces a a semana E t empo
durante e cua podemos ap car estos tratamentos no est b en def n do se
aconseja emp ear os durante 1-2 aos co nc d endo con e t empo de durac n
de os estud os ex stentes Otra forma estrogn ca es e promestr eno (d ter-
estrad o ) d spon b e en cremas para ap cac n ntravag na con una pauta
s m ar a as otras formas de estrgenos

Bibliografa: GINECOLOGIA DE NOVAK. BEREK S. JONATHAN. MC GRAW-HILL. INTERMERICANA. EDICIN 12. 1996. PAG. 860-863,964-965,981-995.



21/05/13 17:59 Simulador Proedumed
Pgina 1 de 1 http://www.proedumed.com.mx/simulador/pages/examen/resultadoPregunta.faces
Anlisis del Caso Clnico
Identificacin del reactivo
Area: GINECOLOGA Y OBSTETRICIA
Especialidad: OBSTETRICIA
Tema: HEMORRAGIA POSTPARTO Y PUERPERIO ANORMAL
Subtema: HEMORRAGIA POSTPARTO
CASO CLNICO CON UNA PREGUNTA

MUJER DE 31 AOS, GESTA 2, PARA 2 QUE SE ENCUENTRA EN RECUPERACIN EN SUS PRIMERA HORAS DE PUERPERIO POSTEUTOCIA DE RECIN NACIDO FEMENINO
DE 4300 GR, QUE CURSO CON 8 HORAS DE PRIMER PERIODO DE PARTO Y PERIODO EXPULSIVO DE 1 HORA. POSTERIOR A ALUMBRAMIENTO PRESENT HEMORRAGIA
TRANSVAGINAL DE 500CC QUE CEDI AL MANEJO CON OXITCICOS. SE REVISA CAVIDAD UTERINA ENCONTRANDOSE SIN RESTOS PLACENTARIOS.

ELEMENTOS CLAVE A CONSIDERAR EN EL CASO CLNICO:
Edad:
Antecedentes: producto macrosmico
Sintomatologa: hemorragia transvaginal posterior a alumbramiento que cede con
oxitocina
Exploracin: sin restos placentarios
Laboratorio y/o gabinete:

3 - EL FACTOR DE RIESGO PRESENTE EN LA PACIENTE QUE CON MAYOR POSIBILIDAD SE ASOCIA A LA COMPLICACIN ENCONTRADA EN STA ES:

EL
NMERO
DE
GESTAS
La aton a uterina constituye una las causa de hemorragia puerperal precoz se debe a la
falta de contraccin uterina Normalmente tras el alumbramiento ocurre una contraccin
uterina intensa que sirve como principal mecanismo hemosttico postparto que en
conjunto con los factores de coagulacin previenen la hemorragia puerperal Los factores
de riesgo para aton a uterina son Polihidramnios gestacin mltiple MACROSOM A
parto rpido parto prolongado alta paridad infecciones alteraciones anatmicas del
tero UNA HEMORRAG A POSTER OR AL ALUMBRAM ENTO QUE CEDE CON LA
ADM N STRAC N DE OX TOC NA ES EL CUADRO CL N CO CARACTER ST CO DE
LA ATON A UTER NA SER SECUND GESTA NO CONST TUYE UN FACTOR DE
R ESGO EN STE CASO
EL
TIEMPO
DEL
PRIMER
PERIODO
DE
TRABAJO
DE
PARTO
El primer periodo de trabajo de parto inicia al momento en que las contracciones uterinas
comienzan a ser regulares hasta haber dilatacin completa del cuello uterino Se divide
en 2 fases latente (las contraccines sn escasas y de intensidad leve lo que hace que los
cambios cervicales sean lentos) y activa (inicia cuando el cuello uterino est dilatado de 4
a 9 cm y comienzo del descenso de la cabeza fetal) La fase de latencia en las nul paras
es de 6 5hrs con un mximo de 20 y en las mult paras de 4 7hrs con un mximo de 14
posteriormente la tasa de dilatacin se calcula en 1 cm/h LA PAC ENTE TUVO UN
PR MER PER ODO DE PARTO DE 8HRS LO QUE SE CONS DERA NORMAL EN SU
COND C N DE SECUND GESTA POR LO QUE NO CONST TUYE UN FACTOR DE
R ESGO
EL
TIEMPO
DEL
SEGUNDO
PERIODO
DE
TRABAJO
DE
PARTO
El segundo periodo del trabajo de parto comienza con la dilatacin completa del cuello
uterino y termina con el nacimiento del feto su duracin promedio oscila entre los 50min
para las nul paras y al rededor de 20 min en mult paras pero es variable Puede
prolongarse de manera anormal en la mujer con estrechez plvica un FETO GRANDE o
esfuerzos para la expulsin inadecuados debido a analgsia de conduccin o sedacin
LA PAC ENTE PRESENTA UN SEGUNDO PER ODO DE PARTO PROLONGADO (1HR)
EL CUAL ES SECUNDAR O A LA MACROSOM A FETAL POR LO QUE S ES UN
FACTOR DE R ESGO PERO SECUNDAR O
EL PESO
DEL
RECIN
NACIDO
LA MACROSOM A PROVOC EN STE CASO UN SEGUNDO PER ODO DEL
TRABAJO DE PARTO PROLONGADO Y ADEMAS CONST TUYE UNO DE LOS
FACTORES MS MPORTANTES EN LA ATON A UTER NA

Bibliografa:JOHNS HOPKINS GINECOLOGIA Y OBSTETRICIA. BRANDON J. BANKOWSKI. MARBN LIBROS, S.L. MADRID ESPAA. EDICIN 2DA. 2005. PAG.
100.



21/05/13 18:00 Simulador Proedumed
Pgina 1 de 1 http://www.proedumed.com.mx/simulador/pages/examen/resultadoPregunta.faces
Anlisis del Caso Clnico
Identificacin del reactivo
Area: GINECOLOGA Y OBSTETRICIA
Especialidad: GINECOLOGA
Tema: LESIONES BENIGNAS Y MALIGNAS DE LA MAMA
Subtema: ENFERMEDAD FIBROQUSTICA
CASO CLNICO CON UNA PREGUNTA

MUJER QUE ACUDE A SU CONSULTORIO AL SER USTED SU MDICO FAMILIAR PARA RECIBIR ORIENTACIN. EL DA DE AYER FUE DIAGNSTICADA POR EL GINECOLOGO
CON MASTOPATA FIBROQUSTICA DE MAMA.

ELEMENTOS CLAVE A CONSIDERAR EN EL CASO CLNICO:
Edad: -
Antecedentes: -
Sintomatologa: En este caso se nos menciona ya la enfermedad, dando pie a preguntar
cualquier aspecto de la enfermedad.
Exploracin: -
Laboratorio y/o gabinete: -

4 - USTED ESTA OBLIGADO A COMENTARLE QUE UNA CARACTERSTICA IMPORTANTE DE ESTA ENFERMEDAD ES:

LA EDAD DE INICIO
GENERALMENTE
ES EN LA
POSTMENOPAUSIA
MASTOPATA FIBROQUSTICA. Concepto: Conjunto de alteraciones
histolgicas, tanto proliferativas como involutivas, tanto del tejido granular de la
mama, como del tejido de sostn del mismo (tej. conectivo) como resultado de
una alteracin No tumoral. Desde el punto de vista clnico, tampoco es exacto
llamar Enfermedad a una entidad que slo produce sntomas incapacitantes
entre 5 % y 10 % de las pacientes, y que se observa clnicamente en 50 % de
las mujeres e histolgicamente en 90 %. Como podrs ver esta patologa se
presenta entre la adolescencia y la menopausia. No es comn en la etapa
postmenopasica.
LOS SNTOMAS
CESAN CUANDO
CESA LA FUNCIN
OVRICA
OJO. 1. Se presenta entre la adolescencia y la menopausia. 2. Generalmente
es de carcter bilateral y difuso. 3. Mayor incidencia en mujeres nulparas. 4.
Durante embarazo y lactancia se produce un estado de reposo. 5. Alteraciones
y manifestaciones durante el ciclo menstrual (Indicador). Al cesar la funcin
ovrica (Postmenopausia) los sntomas desaparecen porque no hay estmulo
mamario que la respalde.
LA
SINTOMATOLOGA
MEJORA CON EL
CONSUMO DE
METIL XANTINAS.
Etiologa: Como en muchos otros trastornos en los que se desconoce su causa,
la MFQ tiene una etiologa multifactorial, y rara es la vez que se deba a una
sola causa de las que se exponen. Entre las causas tenemos: 1. Inestabilidad
hormonal: En general, se ha encontrado un disbalance entre estrgenos y
prolactina (que aumentan) y progesterona (que disminuye). 2. Alteracin del
metabolismo de los andrgenos. 3. Trastorno de hormonas tiroideas. 4.
Metilxantinas: Sustancia del t, caf, chocolate o bebidas de cola (todos
alimentos estimulantes del SNC). 5. Teora alrgica: Se cree que a travs de
sustancias vasoactivas de los mastocitos se produce alteraciones
fibroqusticas. El consumo de metil xantinas se ha asociado a una mayor
incidencia de mastopata fibroqustica.
SE LE CONSIDERA
PREDECESORA DE
CNCER DE
MAMA.
Fisiopatologa: Una vez que comienza el proceso, ste va pasando por una
serie de fases hasta su evolucin final. Estas fases son: 1) Alteracin del
estroma o tejido conectivo. - Edema. - Proliferacin fibroblastos. - Y por ltimo,
fibrosis. 2) Proliferacin del tejido epitelial. - Proliferacin. - Hipertrofia
(Hiperplasia lobulillar). 3) Cambios. - Ectasias. - Esclerosis. - Formacin de
Quistes. - Fibrosis. La MFQ no compleja tiene un riesgo similar al de la
poblacin general de padecer cncer de mama. La mastopata fibroqustica se
considera una patologa benigna. No olvides recordar el cuadro clnico y el
tratamiento. Manifestaciones Clnicas: Entre las manifestaciones de la M.F.Q.
se encuentran: 1) Dolor: mastodinia (no confundir con mastalgia). 2) Secrecin
por el pezn: de color oscuro o verdoso aunque estas caractersticas tambin
se pueden dar en el cncer. 3) Tumor: El tumor pasa por fases: - Fibrosis
moderada. - Fibrosis progresiva (microquistes= <4mm). - Macroquistes (>4mm).
Tratamientos: Los tratamientos ms tiles contra la M.F. son de tipo emprico y
mejoran pero no curan la enfermedad: - Supresin de metilxantinas (Muy
efectivo): Dietas pobres en metilxantinas (caf, te, chocolate y bebidas
carbnicas) producen mejoras en un 60 % de los casos. - Tratamiento
hormonal: Progesterona (El ms efectivo): Se da en forma de gel y se debe
aplicarlo durante 2-3 meses, para que se alcancen niveles teraputicos de
progesterona.

Bibliografa:GINECOLOGIA Y OBSTETRICIA APLICADAS. ROBERTO AHUED/ CARLOS FERNANDEZ DEL CASTILLO. EL MANUAL MODERNO. EDICIN 2DA. 2003.
PG. 1012-1013.



21/05/13 18:01 Simulador Proedumed
Pgina 1 de 1 http://www.proedumed.com.mx/simulador/pages/examen/resultadoPregunta.faces
Anlisis del Caso Clnico
Identificacin del reactivo
Area: GINECOLOGA Y OBSTETRICIA
Especialidad: OBSTETRICIA
Tema: COMPLICACIONES DEL EMBARAZO Y DEL PARTO
Subtema: ISOINMUNIZACIN RH
CASO CLNICO CON UNA PREGUNTA

MUJER DE 31 AOS GESTA 3, PARTO 2. SU SEGUNDO HIJO DURANTE LA PRIMERA SEMANA DE VIDA PRESENT ICTERICIA QUE REQUIRI EXANGUINEOTRANSFUSIN.
ACUDE EN ESTA OCASIN A CONTROL PRENATAL POR EMBARAZO DE 7 SEMANAS DE GESTACIN SEGN FECHA DE LTIMA MENSTRUACIN.

ELEMENTOS CLAVE A CONSIDERAR EN EL CASO CLNICO:
Edad: -
Antecedentes: g3, p2, SEGUNDO HIJO CON ICTERICIA EN PRIMERA SEMANA DE
VIDA QUE AMERITA EXANGUINEOTRANSFUSIN.
Sintomatologa: -
Exploracin: -
Laboratorio y/o gabinete: -

5 - LA CAUSA MS PROBABLE DE LA ICTERICIA DEL HIJO PREVIO FUE:

ICTERICIA
MULTIFACTORIAL
La ictericia fisiolgica o multifactorial inicia al tercer da y generalmente no
amerita tratamiento. COINCIDE CON EL TIEMPO DE APARICIN DE LA
ICTERICIA EN EL PRODUCTO PREVIO DE LA PACIENTE PERO EL
HECHO DE HABER REQUERIDO EXANGUNEOTRASFUSIN DESCARTA
ESTA OPCIN POR COMPLETO.
INCOMPATIBILIDAD
DE GRUPO
SANGUNEO
La incompatibilidad al antgeno sanguneo A y B es muy frecuente sin
embargo, su diagnstico debe considerar las siguientes caractersticas: 1. La
enfermedad ABO se presenta ms frecuentemente en lactantes primognitos.
2. Casi todas las especies de antgenos anti-A y anti-B son IgM que no cruza
la barrera trasplacentaria y, por tanto, no puede llegar a los eritrocitos fetales.
3. Los eritrocitos fetales tienen menos sitios antignicos A y B que las clulas
del adulto y por lo tanto, son menos inmungenos. 4. La enfermedad es
siempre ms leve que la isoinmunizacin D y pocas veces produce anemia
importante. 5. La enfermedad ABO pocas veces se vuelve progresivamente
ms grave. LA INCOMPATIBILIDAD AL GRUPO NO ES LA CAUSA MS
FRECUENTE E ISOINMUNIZACIN MATERNO FETAL, SE SOSPECHA
MS CUANDO SE PRESENTA DESDE EL PRIMER EMBARAZO.
INCOMPATIBILIDAD
DE FACTOR RH
NEGATIVO
El 90% de los casos de Enfermedad Hemoltica Perinatal son el resultado de
la incompatibilidad con el antgeno D. La presencia de D condiciona un sujeto
D (+)y, por tanto Rh (+) y su ausiencia un sujeto D (-) y, por ende, Rh (-).
Recuerda que la isoinmunizacin por Rh se manifiesta a partir de la segunda
gesta dado que en la primera exposicin se producen mayormente IgM que
no atraviesan la barrera trasplacentaria sin embargo, en la segunda
exposicin se producen una gran cantidad de IgG que llegan a la circulacin
fetal desencadenando la respuesta fagocitaria. LA INCOMPATIBILIDAD AL
RH COINCIDE COMO LA CAUSA MS FRECUENTE QUE SE PRESENTA A
PARTIR DE LA SEGUNDA GESTA.
HEMOTRANSFUSIN
MATERNO-FETAL
La Hemotrasfusin Materno Fetal constituye uno de los factores etiolgicos en
la policitemia neonatal. ste fenmeno est dado por contracciones uterinas
intensas antes de la ligadura del cordn o sugetar al recin nacido por debajo
de la madre luego del parto. Ciertamente los neonatos pueden presentar
ictericia importante a las 48hrs de vida, aunque la mayoria son asintomticos,
y regularmente ir acompaada de otros sntomas resultado de la policitemia
fetal (letargo,convulsiones, hipoglucemia, taquicardia, cardiomegalia, dificultad
respiratoria, retardo en el llenado capilar distal, entre otras). LA MAYOR
PARTE DE CASOS POR POLICITEMIA SECUNDARIA A
HEMOTRASFUSIN MATERNO-FETAL SON ASINTOMTICAS Y EN EL
CASO DE SERLO DAN DATOS ASOCIADOS A LA VISCOSIDAD
SANGUNEA QUE NO ESTN PRESENTES EN LOS ANTECEDENTES DE
LA PACIENTE.

Bibliografa: GINECOLOGA Y OBSTETRICIA. HAJO I. J. WILDSCHUT. MC. GRAW-HILL. INTERAMERICANA. EDICIN 1RA. PAG. 66.



21/05/13 18:02 Simulador Proedumed
Pgina 1 de 1 http://www.proedumed.com.mx/simulador/pages/examen/resultadoPregunta.faces
Anlisis del Caso Clnico
Identificacin del reactivo
Area: GINECOLOGA Y OBSTETRICIA
Especialidad: GINECOLOGA
Tema: DOLOR PELVICO
Subtema: DISMENORREA
CASO CLNICO CON UNA PREGUNTA

MUJER DE 28 AOS DE EDAD, CON DIAGNSTICO DE DISMENORREA. USTED CONSIDERA INDICAR UN ANTINFLAMATORIO NO ESTEROIDEO.

ELEMENTOS CLAVE A CONSIDERAR EN EL CASO CLNICO:
Edad: Mujer de 28 aos de edad
Antecedentes:
Sintomatologa: Dismenorrea debes recordar de inmediato los aines que pueden ser
utilizados en esta patolog a
Exploracin:
Laboratorio y/o gabinete:

6 - DE LOS SIGUIENTES AINES EL QUE MENOS EFECTO TERAPUTICO HA DEMOSTRADO ES EL:

NAPROXENO, Dentro del Tratamiento de la dismenorrea se encuentran Analgsicos A NES
ACO Otros Anlogos de la Hormona liberadora de gonadotropinas (GnRh)
danazol Los A NES son analgsicos inhibidores no espec ficos de las enzimas
ciclooxigenasas (COX) con la consiguiente disminucin de las prostaglandinas y el
alivio del dolor
IBUPRUFENO. La evidencia sugiere que en mujeres con dismenorrea primaria los A NES son ms
efectivos que el placebo para el control del dolor No observndose evidencias
slidas de cual es mejor que otro Los A NES son un tratamiento efectivo para
dismenorrea aunque se debe de ser consciente del riesgo significativo de sus efectos
adversos no existen pruebas suficientes para determinar que A NE es el ms seguro
y efectivo en el tratamiento de la dismenorrea
CIDO ACETIL
SALICLICO.
Un meta anlisis de 56 ensayos reporto que 4 A NES estudiados (Naproxen
ibuprofeno cido mefenmico y aspirina) son efectivos en la dismenorrea primaria
siendo el naproxen y el ibuprofeno mejores que la aspirina No se encontraron
diferencias significativas entre los A NES excepto el cido acetal salic lico que
demostr tener menos eficacia que el resto de los A NES de un 70 % contra un 40 %
de efectividad en el alivio del dolor
CIDO
MEFENMICO.
Los efectos adversos ms frecuentes de los A NES son s ntomas gastrointestinales
leves como nusea dispepsia cefalea mareo somnolencia y resequedad de boca
Los anti inflamatorios de nueva generacin como los inhibidores espec ficos de la
COX 2 son efectivos para la dismenorrea aunque cuestionables sus efectos en
seguridad cardiovascular como meloxicam cido mefenmico rofecoxib y
valdecoxif estos dos ltimos retirados del mercado por efectos adversos graves A
las mujeres con dismenorrea primaria se les puede ofrecer tratamiento con A NES de
primera intencin para disminuir el dolor y mejorar la calidad de vida excepto
aquellas que tengan alguna contraindicacin para el uso de A NES Se recomienda
que el uso de los A NES sea de dos a tres d as antes del ciclo menstrual y 2 a 3 d as
despus de inicio de la menstruacin

Bibliografa:GPC. DIAGNSTICO Y TRATAMIENTO DE DISMENORREA EN EL PR MER, SEGUNDO Y TERCER NIVEL DE ATENCIN. IMSS- 183-
09

http //www.cenetec.salud.gob mx/descargas/gpc/CatalogoMaestro/183 GPC DISMENORREA/IMSS 183 09 EyR Dismenorrea pdf


21/05/13 18:05 Simulador Proedumed
Pgina 1 de 1 http://www.proedumed.com.mx/simulador/pages/examen/resultadoPregunta.faces
Anlisis del Caso Clnico
Identificacin del reactivo
Area: GINECOLOGA Y OBSTETRICIA
Especialidad: GINECOLOGA
Tema: LESIONES BENIGNAS Y MALIGNAS DE LA MAMA
Subtema: ENFERMEDAD FIBROQUSTICA
CASO CLNICO CON UNA PREGUNTA

MUJER DE 23 AOS DE EDAD, SOLTERA Y SIN VIDA SEXUAL ACTIVA. ACUDE A CONSULTA POR PRESENTAR TUMORACIN MAMARIA IZQUIERDA EN EL CUADRANTE
INFERIOR EXTERNO NICA Y DOLOROSA DURANTE LA MENSTRUACIN.

ELEMENTOS CLAVE A CONSIDERAR EN EL CASO CLNICO:
Edad: Ante un ndulo mamario, la edad no determina la conducta a seguir.
Antecedentes: -
Sintomatologa: -
Exploracin: 1. Manejo del ndulo mamario. Ante la aparicin de un ndulo mamario
debemos descartar. Siempre la malignidad, ya se trate de una paciente
que presente o haya presentado anteriormente quistes, fibroadenomas u
otros ndulos benignos. La exploracin es muy importante. Son
sospechosos de malignidad los ndulos mal definidos, poco mviles, de
forma estrellada, que infiltren o retraigan la piel o deformen la mama.
Laboratorio y/o gabinete: Se derivar al especialista: 1. Cualquier ndulo de reciente aparicin o
sin diagnosticar. 2. Ante cualquier signo de sospecha de malignidad tras
la exploracin clnica, antes incluso de realizar cualquier prueba de
imagen. 3. Ante cualquier signo de sospecha de malignidad en las
pruebas de imagen sin necesidad de realizar ninguna otra prueba
diagnstica. 4. Si es necesario realizar alguna otra maniobra diagnstica
de un ndulo, aunque sea para confirmar la benignidad (PAAF o
Biopsia).

7 - USTED SOLICITAR EL SIGUIENTE ESTUDIO PARA CONFIRMAR SU DIAGNSTICO:

ULTRASONIDO
DE MAMA.
La ecografa es de gran utilidad para definir si una masa es qustica o slida, y
forma parte de casi todos los algoritmos para el diagnstico. Ciertas caractersticas
de los tumores slidos como los bordes irregulares, los ecos internos o una relacin
entre espesor y altura menor de 1.7 cm sugieren cncer. Es de gran utilidad para
determinar datos sugestivos de mastopata fibroqustica, sin embargo ante la
presencia de una masa nica palpable se sugiere biopsia.
MASTOGRAFA. Los estudios de imagen de un tumor sospechoso comienzan con una mastografa,
que comprende amplificacin, compresin o la toma de proyecciones adicionales a
la oblicua lateral, y craneal caudal que son las habituales. A diferencia de la
mastografa de deteccin, la mastografa diagnstica se realiza en mujeres de
cualquier edad. Los resultados de las imgenes diagnsticas se deben resumir
segn la clasificacin del Breast Imaging Reporting and Data System (BI-RADS).
Las lesiones que se clasifican como BI-RADS 5 son altamente sugestivas de
cncer y > 95% de los casos, al final se diagnstica cncer. Una clasificacin baja
reduce la posibilidad de cncer. Recuerda; que la mastografa est indicada en
mujeres mayores de 40 aos debido a que la densidad del tejido mamario en
mujeres jvenes no permite visualizar lesiones por ste mtodo.
BIOPSIA DE LA
LESIN.
La combinacin de la exploracin fsica con los estudios de imagen y la biopsia con
aguja se denomina prueba triple. Cuando estos tres mtodos sugieren una lesin
benigna o un cncer mamario, se dice que la prueba triple es concordante. Una
prueba triple benigna concordante tiene una precisin del 99 % y los tumores
mamarios que caen dentro de esta categora se pueden mantener en observacin
realizando exploraciones clnicas a intervalos de seis meses. Cuando cualquiera de
los tres estudios sugiere la posibilidad de cncer, el tumor se debe extirpar sin
importar los resultados de las otras dos. Siempre es recomendable ofrecer la
ablacin de un tumor mamario bien estudiado, incluso, despus de obtener un
resultado concordante en la prueba triple, puesto que los tumores mamarios causan
gran ansiedad. Toda lesin mamaria "nica" palpable (>1cm) debe ser biopsiada
para determinar su clase histopatolgica.
NIVELES
SRICOS DE
PROLACTINA.
Como ya lo debiste de haber constatado, esta opcin no tiene nada que ver en el
estudio de la paciente con ndulo mamario. Es muy importante que adviertas que
los tres estudios anteriores son tiles y necesarios en el estudio del ndulo
mamario, sin embargo, la pregunta menciona diagnstico definitivo que implica
benignidad o malignidad, lo que slo se puede lograr con el estudio histopatolgico
de la lesin.

Bibliografa: TRATADO DE GINECOLOGA DE NOVAK. JONES III, HOWARD W. MC. GRAW HILL. 11A ED. 1991. PG. 474



21/05/13 18:06 Simulador Proedumed
Pgina 1 de 1 http://www.proedumed.com.mx/simulador/pages/examen/resultadoPregunta.faces
Anlisis del Caso Clnico
Identificacin del reactivo
Area: GINECOLOGA Y OBSTETRICIA
Especialidad: GINECOLOGA
Tema: ANTICONCEPCIN Y PLANIFICACIN FAMILIAR
Subtema: MTODOS DE PLANIFICACIN FAMILIAR, TEMPORALES Y
DEFINITIVOS
CASO CLNICO CON UNA PREGUNTA

MUJER DE 41 AOS DE EDAD QUE DESEA REVERSIN DE SALPINGOCLASIA BILATERAL.

ELEMENTOS CLAVE A CONSIDERAR EN EL CASO CLNICO:
Edad: 41 aos
Antecedentes: desea reversin de salpingoclasia bilateral
Sintomatologa:
Exploracin:
Laboratorio y/o gabinete:

8 - USTED DEBER EXPLICARLE QUE ESTA REVERSIN INCREMENTA SU RIESGO DE PRESENTAR:

HIDROSALPINX La anticoncepcin tubrica procedimiento en principio irreversible es sin embargo
potencialmente factible de reversin pero las posibilidades de un embarazo
intrauterino posterior a la restitucin son extremadamente variables La tcnica de
eleccin para la recanalizacin tubaria es la endoscpica con un m nimo de
complicaciones tras el procedimiento De haber complicaciones son de tipo
infeccioso aqu una revisin de las siguientes definiciones de infeccin tubarea
H DROSALP NX Es una alteracin de la trompa de Falopio en la que sta se
encuentra bloqueada dilatada y con l quido en su interior generalmente debido a
una infeccin previa en las trompas
SALPINGITIS SALP NG T S Es la inflamacin aislada de las trompas de Falopio La causa ms
frecuente es el ascenso de gonococos o clamidias (pero tambin otros grmenes
aerobios y anaerobios) a partir de focos del tramo genital bajo con ocasin de la
menstruacin enfriamiento coito etc
PIOSALPINX P OSALP NX Es la acumulacin de pus en la trompa por salpingitis Generalmente
secundaria a una enfermedad plvica inflamatoria
EMBARAZO
ECTPICO
Las mujeres que se han sometido a una cirug a de recanalizacin tubrica con el fin
de volver a quedar embarazadas tambin se exponen a un alto riesgo de tener un
embarazo ectpico EL EMBARAZO ECTP CO CONST TUYE LA PR NC PAL
COMPL CAC N AL NTENTAR UN EMBARAZO EN LA MUJER CON
RECANAL ZAC N TUBAR A

Bibliografa:DIAGNOSTICOS Y TRATAMIENTOS GINECO OBSTETRICOS. DE CHERNEY ALAN H., PERNOLL MARTIN L. MANUAL MODERNO. 7 ED. 1997. PAG.
1130



21/05/13 18:07 Simulador Proedumed
Pgina 1 de 1 http://www.proedumed.com.mx/simulador/pages/examen/resultadoPregunta.faces
Anlisis del Caso Clnico
Identificacin del reactivo
Area: GINECOLOGA Y OBSTETRICIA
Especialidad: OBSTETRICIA
Tema: HEMORRAGIA POSTPARTO Y PUERPERIO ANORMAL
Subtema: HEMORRAGIA POSTPARTO
CASO CLNICO SERIADO

MUJER DE 31 AOS DE EDAD, QUE CURSA PRIMERAS HORAS DE PUERPERIO POST-EUTCICO INMEDIATO DE UN PRODUCTO DE 4500 GRAMOS OBTENIDO MEDIANTE LA
APLICACIN DE FRCEPS POSTERIOR A UN PERIODO EXPULSIVO PROLONGADO. DURANTE SU VALORACIN LA ENCUENTRA CON SANGRADO TRANSVAGINAL EN
MODERADA CANTIDAD Y TERO A NIVEL DE CICATRIZ UMBILICAL.

ELEMENTOS CLAVE A CONSIDERAR EN EL CASO CLNICO:
Edad: -
Antecedentes: PUERPERIO INMEDIATO PARTO EUTCICO PRODUCTO
MACROSMICO USO DE FRCEPS
Sintomatologa: -
Exploracin: SANGRADO TRASVAGINAL MODERADO TERO INVOLUCIONADO
Laboratorio y/o gabinete: -

9 - EL DIAGNSTICO CLNICO MS PROBABLE ES:

ATONA
UTERINA
La atona uter na const tuye una as causa de hemorrag a puerpera precoz se
debe a a fa ta de contracc n uter na Norma mente tras e a umbramento ocurre
una contracc n uter na ntensa que s rve como pr nc pa mecan smo hemostt co
postparto que en conjunto con os factores de coagu ac n prev enen a hemorrag a
puerpera Los factores de r esgo para atona uter na son: Po h dramn os gestac n
m t p e macrosoma parto rp do parto pro ongado a ta par dad nfecc ones
a terac ones anatmcas de tero INMEDIATAMENTE DESPUS DE LA
EXPULSIN PLACENTARIA EL FONDO DEL TERO CONTRADO SE SITA
CASI AL NIVEL DE LA CICATRIZ UMBILICAL CIRCUNSTANCIA QUE NO
SUCEDE EN LA ATONA UTERINA LA PACIENTE PRESENTA UNA ADECUADA
INVOLUCIN UTERINA AL MOMENTO DE LA EXPLORACIN LO QUE
DESCARTA STA POSIBILIDAD DIAGNSTICA
DESGARROS
DEL CANAL
DEL PARTO
Todos os desgarros per nea es sa vo os ms superf c a es se acompaan de
grados var ab es de es n de a parte nfer or de a vag na dando ugar a
hemorrag as mportantes Los pr nc pa es factores de r esgo para desgarro de cana
b ando: parto prec p tado parto nstrumentado (uso de forceps) macrosoma feta
ma pos c n feta LA HEMORRAGIA MIENTRAS EL TERO EST FIRMEMENTE
CONTRADO ES UNA FUERTE PRUEBA DE DESGARRO DEL APARATO
GENITAL ASOCIADO A LA MACROSOMA FETAL Y EL USO DE FRCEPS
INVERSIN
UTERINA
La nvers n uter na es e g ro "a revs" de fondo uter no; cas s empre se da como
consecuenc a de tracc n fuerte sobre un cordn umb ca f jo a una p acenta
mp antada en e fondo un cordn grueso que no se desprende con fac dad de a
p acenta p acenta acreta y atona uter na aunque puede presentarse s n que
necesar amente haya adhes n f rme de a p acenta Se re ac ona con una
hemorrag a nmed ata que pone en r esgo a v da ES UNA COMPLICACIN
INMEDIATA DURANTE EL PARTO GENERALMENTE ANTES DEL
ALUMBRAMIENTO O DURANTE STE NO COINCIDE CON EL CUADRO
CLNICO POR ESTAR LA PACIENTE EN EL POSTPARTO ADEMS DE
PALPARSE EL FONDO UTERINO LO QUE DESCARTA STA OPCIN
DIAGNSTICA
RETENCIN DE
RESTOS
PLACENTARIOS
La retenc n de restos p acentar os y/o ovu ares en a cav dad uter na es a
expu s n ncomp eta de a p acenta A examen de a p acenta se observa ausenc a
de uno o ms cot edones Se asoc a a una nadecuada as stenc a de
a umbramento p acentas con mayor adhes v dad Es a pr nc pa causa de
hemorrag a postparto tarda Se caracter za por sangrado gen ta ntermtente
oqu os ft dos endometr t s y sub nvo uc n uter na NO COINCIDE CON EL
CUADRO CLNICO YA QUE LA RETENCIN DE RESTOS PLACENTARIOS ES
UNA COMPLICACIN TARDA CON PRESENCIA DE LOQUIOS FTIDOS

Bibliografa: WILLIAM OBSTETRICIA. PRITCHMAN, JACK A. SALVAT. EDICIN 3A. 1992. PAG. 685-696.



10 - EN ESTE MOMENTO EL TRATAMIENTO INDICADO ES:

REVISIN INSTRUMENTADA
DEL CANAL DEL PARTO Y
REPARACIN
Una vez que se sospecha desgarro de partes b andas se requ ere un
examen met cu oso de cana de parto ya que e examen d g ta no es
suf c ente La v sua zac n se ogra rea zando tracc n en os ab os
de cue o uter no con p nzas de an os a menudo son necesar os os
retractores de a pared vag na para mejorar a v s b dad EL
DIAGNSTICO ES CLNICO Y CONSISTE EN LA INSPECCIN
METICULOSA DEL CANAL DEL PARTE PARA LOCALIZAR LA
ZONA DE HEMORRAGIA Y REALIZAR UNA REPARACIN
QUIRRGICA RESULTANTE
HISTERECTOMA
ABDOMINAL
La h sterectoma en s tuac n de hemorrag a mas va es t por su
capac dad para e mnar e sangrado su desventaja rad ca en a
prd da de tero ste proced mento so o est reservado para casos
muy graves de hemorrag a RECUERDA QUE LA HISTERECTOMA
ELIMINA EL SANGRADO CUANDO STE ES DE ORIGEN
UTERINO Y EST RESERVADO A CASOS GRAVES DE
HEMORRAGIA DADO QUE LA PACIENTE PRESENTA SANGRADO
MODERADO CON BUENA CONTRACCIN UTERINA NO ES UNA
POSIBILIDAD TERAPUTICA ADECUADA
REVERSIN UTERINA Y
OXITOCINA EN BOLO
Ante a presenc a de nvers n uter na se recomenda a repos c n de
tero (man obra de Johnson) con a mano en a vag na se eva e
fondo de tero hac a arr ba manten endo e pu gar y e nd ce a n ve
de a un n crv co-uter na Pr mero separar a p acenta y uego se
repone e tero Poster ormente admn strar ox toc na o retractores
uter nos para favorecer a contracc n uter na S con esto no es
efect vo se debe rea zar h sterectoma ESTA OPCIN DESCRIBE
EL MANEJO DE LA INVERSIN UTERINA DIAGNSTICO QUE NO
COINCIDE CON EL CUADRO CLNICO DE LA PACIENTE
LEGRADOINSTRUMENTADO E egrado nstrumentado como tratamento de a hemorrag a uter na
postparto est encamnado a a extracc n de restos p acentar os
como tratamento de e ecc n Debe rea zarse en qu rfano y bajo
anestes a genera Con ayuda de una egra roma no cortante se vaca
a cav dad uter na y tras e proced mento se comprueba med ante
una ecografa a pers stenc a de restos NO EST INDICADO EN LA
PACIENTE PUES NO HAY DATOS QUE SUGIERAN QUE LA
HEMORRAGIA ES SECUNDARIA A LA PERSISTENCIA DE
RESTOS PLACENTARIOS

Bibliografa: WILLIAM OBSTETRICIA. PRITCHMAN, JACK A. SALVAT. EDICIN 3A. 1992. PAG. 689.


FIN DEL CASO CLNICO SERIADO

21/05/13 18:07 Simulador Proedumed
Pgina 1 de 1 http://www.proedumed.com.mx/simulador/pages/examen/resultadoPregunta.faces
Anlisis del Caso Clnico
Identificacin del reactivo
Area: GINECOLOGA Y OBSTETRICIA
Especialidad: GINECOLOGA
Tema: LESIONES BENIGNAS Y MALIGNAS DE LA MAMA
Subtema: CNCER DE MAMA
CASO CLNICO SERIADO

MUJER DE 40 AOS DE EDAD, HERMANA FINADA POR CNCER DE MAMA. TABAQUISMO POSITIVO DESDE LOS 20 AOS, ALCOHOLISMO OCASIONAL, NULPARA. ACUDE
SOLICITANDO SE LE REALICE MASTOGRAFA PARA DETECCIN DE CNCER DE MAMA.

ELEMENTOS CLAVE A CONSIDERAR EN EL CASO CLNICO:
Edad: MUJER DE 40 aos
Antecedentes: hermana con CA DE MAMA
Sintomatologa: -
Exploracin: -
Laboratorio y/o gabinete: -

11 - LA SIGUIENTE RECOMENDACIN SE ASOCIARA CON UNA MAYOR DISMINUCIN DEL RIESGO DE CNCER DE MAMA EN LA PACIENTE:

EJERCICIO
FSICO
EL EJERCICIO FSICO MODERADO SE ASOCIA CON UNA DISMINUCIN DEL
RIESGO DE CNCER DE MAMA EN LA POBLACIN GENERAL Es mportante
nformar a as mujeres sobre os benef c os potenc a es de ejerc c o fs co en a
reducc n de r esgo de padecer cncer de mama En mujeres postmenopas cas
con un IMC >30 aumenta 2 veces ms e r esgo de cncer de mam y es aqu
donde e ejerc c o ejerce una acc n muy fuerte en e caso EL EJERICIO Y
CONTROL DE PESO EN LA ACTUALIDAD REPRESENTAN LOS CAMBIOS DE
ESTILO DE VIDA MS EFICIENTES PARA REDUCIR EL RIESGO DE CNCER
DE MAMA
ABANDONAR
HBITO
ALCOHLICO
E r esgo de padecer Cncer de Mama aumenta con e consumo de a coho Se
reporta un ncremento de r esgo re at vo de 7 1% por cada 10 gramos ad c ona es
en a ngesta de a coho por da Ex ste poca ev denc a de que e r esgo re at vo
asoc ado a ncremento en e consumo de a coho sea d ferente para mujeres con
h stor a fam ar de cncer de mama SI BIEN ABANDONAR EL HBITO
TABQUICO ES UN FACTOR BENFICO EN LA INCIDENCIA DE CAMA NO
ESTA BIEN ESTUDIADO
ABANDONO
DEL HBITO
TABAQUICO
Hay nd c os de que e tabaqu smo se puede asoc ar con un pequeo aumento en e
r esgo de cncer de mama; s n embargo deb do a que os resu tados de os
estud os c entf cos no han s do cons stentes esta re ac n ha s do re ac onada
actua mente como especu at va (GPC Prevenc n Tamzaje y Referenc a
Oportuna de Casos sospechosos de Cncer de Mama en e Pr mer N ve de
Atenc n Ev denc as y Recomendac ones Actua zac n 2012) En mujeres que
reportan nunca haber beb do o fumado e r esgo re at vo es cercano a 1
comparado con as mujeres fumadoras Un meta an s s rec ente conc uye que
fumar c garr os aumenta e r esgo de cncer de mama con un r esgo mayor en as
mujeres postmenopus cas y en as que n c aron e tabaqu smo a una edad
temprana LA EVIDENCIA CIENTFICA NO ES CLARA ACTUALMENTE A CERCA
DE LA ASOCIACIN DIRECTA DE TABAQUISMO Y CNCER DE MAMA Y EL
ABANDONO NO PARECE REDUCIR EL RIESGO SI ES QUE YA SE TUVO STE
HBITO
NO
EMBARAZARSE
La actanc a conf ere un efecto protector sobre e r esgo de cncer de mama E
efecto protector de a actanc a se suma a efecto protector de embarazo La
reducc n en e r esgo de cncer de mama est re ac onado con a durac n tota de
a actanc a materna: por doce meses de actanc a hay una reducc n de 4% La
d smnuc n de r esgo re at vo es s m ar con h stor a fam ar La edad avanzada en
e pr mer nac mento v vo o en e pr mer nac mento est asoc ada con un
ncremento s gn f cat vo ene r esgo de cncer de mama E ncremento en a
par dad se ha encontrado que se asoc a con una d smnuc n con e r esgo de
cncer de mama: - Reducc n de 38% de r esgo en mujeres que reportaron 5 o
ms nac dos v vos - D smnuc n de 32% en e r esgo de mujeres que nformaron
de 3 nac mentos en comparac n con mujeres que reportaron 1 nac mento EL
INCREMENTO EN LA PARIDAD Y LA LACTANCIA SON FACTORES
PROTECTORES PARA CANCER DE MAMA

Bibliografa:- WILLIAMS. GINECOLOGA. MCGRAW-HILL INTERAMERICANA EDITORES. TRADUCCIN DE LA PRIMERA EDICIN. - GUA DE PRCTICA CLNICA.
PREVENCIN Y DIAGNSTICO OPORTUNO DE CNCER DE MAMA EN EL PRIMER NIVEL DE ATENCIN. EVIDENCIAS Y RECOMENDACIONES. CATLOGO
MAESTRO DE GUAS DE PRCTICA CLNICA: SS-001-08. - GUA DE REFERENCIA RPIDA. PREVENCIN Y DIAGNSTICO OPORTUNO DEL CNCER DE
MAMA EN EL PRIMER NIVEL DE ATENCIN GPC: CATLOGO MAESTRO: GO-00L-55-2008. - PREVENCIN, TAMIZAJE Y REFERENCIA OPORTUNA DE CASOS
SOSPECHOSOS DE CNCER DE MAMA EN EL PRIMER NIVEL DE ATENCIN. EVIDENCIAS Y RECOMENDACIONES. ACTUALIZACIN 2012. CATLOGO
MAESTRO DE GUAS DE PRCTICA CLNICA: S-001-08.

http://www.facmed.unam.mx/sg/css/GPC/SIDSS-GPC/gpc/docs/SS-001-08-RR.pdf


12 - EL REALIZAR LA MASTOGRAFA EN LA PACIENTE REDUCIRA SU RIESGO DE MORIR POR CNCER DE MAMA EN EL SIGUIENTE PORCENTAJE:

10-
20%
La mamografa es t en a eva uac n de as mujeres que sospechan que t enen una masa en
a mama que e md co no puede pa par En ta es casos as mgenes de u trason do enfocadas
a s t o de a masa pueden ser coadyuvante d agnst co a cua qu er edad Una masa pa pab e
puede requer r una b ops a con aguja de oca zac n o b ops a estereotct ca
30-
40%
La mamografa es a n ca moda dad de detecc n que ha demostrado d smnu r a morta dad
de cncer de mama en un 26 a 39%
50-
60%
La mastografa est nd cada 1 En mujeres con r esgo med o de cncer mamar o: tamzaje
anua a part r de os 40 aos 2 Mujeres con a to r esgo de cncer de mama: anua mente
empezando desde os 30 aos (pero no antes de os 25) o 10 aos antes de a edad de
d agnst co de fam ar afectado ms joven o que resu te ms tardo
70-
80%
Debe rea zarse anua mente desde e d agnst co de a gn prob ema mamar o
ndepend entemente de a edad

Bibliografa:- WILLIAMS. GINECOLOGA. MCGRAW-HILL INTERAMERICANA EDITORES. TRADUCCIN DE LA PRIMERA EDICIN. - GUA DE PRCTICA CLNICA.
PREVENCIN Y DIAGNSTICO OPORTUNO DE CNCER DE MAMA EN EL PRIMER NIVEL DE ATENCIN. EVIDENCIAS Y RECOMENDACIONES. CATLOGO
MAESTRO DE GUAS DE PRCTICA CLNICA: SS-001-08. - GUA DE REFERENCIA RPIDA. PREVENCIN Y DIAGNSTICO OPORTUNO DEL CNCER DE
MAMA EN EL PRIMER NIVEL DE ATENCIN GPC: CATLOGO MAESTRO: GO-00L-55-2008. - PREVENCIN, TAMIZAJE Y REFERENCIA OPORTUNA DE CASOS
SOSPECHOSOS DE CNCER DE MAMA EN EL PRIMER NIVEL DE ATENCIN. EVIDENCIAS Y RECOMENDACIONES. ACTUALIZACIN 2012. CATLOGO
MAESTRO DE GUAS DE PRCTICA CLNICA: S-001-08.

http://www.imss.gob.mx/profesionales/guiasclinicas/Documents/001GER.pdf

FIN DEL CASO CLNICO SERIADO

21/05/13 18:08 Simulador Proedumed
Pgina 1 de 1 http://www.proedumed.com.mx/simulador/pages/examen/resultadoPregunta.faces
Anlisis del Caso Clnico
Identificacin del reactivo
Area: GINECOLOGA Y OBSTETRICIA
Especialidad: OBSTETRICIA
Tema: COMPLICACIONES DEL EMBARAZO Y DEL PARTO
Subtema: PARTO PREMATURO
CASO CLNICO CON UNA PREGUNTA

MUJER DE 26 AOS DE EDAD GESTA 1, CON EMBARAZO DE 39 SEG Y EN FASE ACTIVA DE TRABAJO DE PARTO.

ELEMENTOS CLAVE A CONSIDERAR EN EL CASO CLNICO:
Edad:
Antecedentes:
Sintomatologa: fase activa trajo de parto
Exploracin:
Laboratorio y/o gabinete:

13 - EN ESTE MOMENTO LA OXITOCINA ES LIBERADA POR:

LA PLACENTA La placenta es el rgano de transporte entre la madre y el feto La placenta
humana tambin sintetiza una enorme cantidad de hormonas prote nicas y
peptidicas que incluyen Lactgeno Placentario (hpL) hormna gonadotropina
corinica (hCG) adenocorticotropina (ACTH) que es una variante de la hormona
de crecimiento prote na relacionada con la hormona paratiroidea (PTH rP)
calcitonina relaxina inhibinas activinas y pptido natriurtico auricular Adems
de otras hormonas similares a las liberadoras e inhibidoras hipotalmicas LA
PLACENTA T ENE FUNC ONES ENDCR NAS MUY D VERSAS Y
EXCLUS VAS DEL EMBARAZO PERO NO ACTA EN EL PROCESO DE
FORMAC N Y L BERAC N DE LA OX TOC NA
LA
NEUROHIPOFISIS
La hipfisis posterior o neurohipfisis es una prolongacin del hipotlamo ventral
La oxitocina es una hormona producida en el hipotlamo y secretada en la
neurohipfisis Aumentan sus niveles en el embarazo y durante la lactancia Los
factores que estimulan su secrecin son est mulos mecnicos en el tero
(presin ejercida por el feto) cuando el embarazo est a trmino aumenta cada
vez ms esta estimulacin y se desencadena el trabajo de parto la succin del
pezn Tiene como efectos aumentar la sensibilidad del tero a despolarizarse y
contraerse aumentar el ritmo e intensidad de las contracciones uterinas y
favorecer eyeccin de la leche materna LA NEUROH PF S S SLO
ALMACENA Y L BERA LA OX TOC NA RECUERDA QUE NO SE PRODUCE
AH
EL ENDOMETRIO El endometrio es un tejido altamente sensible a los cambios hormonales se
regenera en cada ciclo ovrico y endometrial siendo el nico tejido que se
descama y recrece completamente Contiene glndulas que producen moco
Carece de funciones endcrinas EL ENDOMETR O RESPONDE DE FORMA
S NGULAR A LOS EFECTOR HORMONALES FEMEN NOS PERO NO POSEE
FUNC ONES ENDOCR NAS
EL HIPOTALAMO El hipotlamo se encuentra en la zona ms anterior e inferior del diencfalo
produce hormonas "controladoras" Estas hormonas regulan procesos corporales
tales como el metabolismo y controlan la liberacin de hormonas de glndulas
como la tiroides las suprarrenales y las gnadas El hipotlamo secreta la
hormona liberadora de la tirotropina (TRH) que tambin estimula la prolactina la
hormona liberadora de las gonadotropinas (GnRH) la hormona liberadora de la
corticotropina (CRH) la hormona inhibidora de la prolactina (dopamina P H) la
hormona liberadora de la hormona del crecimiento (GHRH) la hormona inhibidora
de la hormona del crecimiento (somatostatina GH H) que tambin puede inhibir
la prolactina y la tirotropina y la hormona liberadora e inhibidora de la hormona
melanocitoestimulante Adems produce oxitocina y hormona antidiurtica EL
H POTLAMO PRODUCE LA OX TOC NA Y SE ALMACENA EN LOS
NCREOS PARAVENTR CULARES PERO NO LA L BERA

Bibliografa:TRATADO DE FISIOLOGIA MDICA. ARTHUR C. GUYTON, M.D. MC. GRAW HILL. INTERAMERICANA. EDICIN 10. 2001. PAG.
1145.



21/05/13 18:09 Simulador Proedumed
Pgina 1 de 1 http://www.proedumed.com.mx/simulador/pages/examen/resultadoPregunta.faces
Anlisis del Caso Clnico
Identificacin del reactivo
Area: GINECOLOGA Y OBSTETRICIA
Especialidad: OBSTETRICIA
Tema: COMPLICACIONES DEL EMBARAZO Y DEL PARTO
Subtema: ATENCIN DEL PARTO Y DISTOSIAS
CASO CLNICO CON UNA PREGUNTA

MUJER DE 38 AOS DE EDAD, GESTA 1. ACUDE A SU PRIMERA CONSULTA PRENATAL. DESEA ORIENTACIN CON RESPECTO A LA CULMINACIN DE SU EMBARAZO.

ELEMENTOS CLAVE A CONSIDERAR EN EL CASO CLNICO:
Edad:
Antecedentes: GESTA 1
Sintomatologa:
Exploracin:
Laboratorio y/o gabinete:

14 - DURANTE SU PLTICA USTED DEBER SEALARLE QUE LA COMPLICACIN MAS FRECUENTE ENCONTRADA DURANTE LA ATENCIN DEL PARTO ES:

DISTOCIA DE
HOMBROS
LA D STOC A MS FRECUENTE DE TODAS ES LA D STOC A DE HOMBROS La
distocia de hombros se produce cuando tras la salida de la cabeza fetal se detiene
la progresin del parto Algunas veces se llega a completar en mayor o menor grado
la rotacin externa pero nunca se produce el descenso con el desprendimiento de los
hombros aun si se utilizan las maniobras habituales de asistencia Y var a entre un
0 6 a 1 6% de los partos
PROLAPSO
DE CORDON
UMBILICAL
El prolapso del cordn umbilical es una complicacin en el parto que ocurre durante
la etapa final del embarazo consiste en un descenso prematuro del cordn al canal
de la vagina frecuentemente cuando se ha roto ya la membrana y el beb se prepara
para nacer Se calcula que este episodio lo padecen una de cada 300 mujeres en
trabajo de parto ES UNA COMPL CAC N POCO FRECUENTE SE PRESENTA EN
EL 0 3% DE LOS PARTOS
RETENCION
DE CABEZA
La retencin de cabeza fetal es una complicacin que se presenta principalmente en
la presentacin de nalgas Por fortuna suele presentarse con poca frecuencia Se han
ideado muchas maniobras para la extraccin de la cabeza El frceps de Piper que es
el ideal para estos casos La frecuencia global de embarazos con presentacin
plvica se encuentra entre el 3 y 5% De stos la frecuencia de retencin de cabeza
ltima es del 4% (solo el 0 2% de los partos plvicos) LA RETENC N DE CABEZA
ES UNA COMPL CAC N MUY RARA Y SE ASOC A FRECUENTEMENTE AL
EMBARAZO PLV CO
PARTO
PRECIPITADO
El parto precipitad es la expulsin del feto en menos de tres horas una actividad
uterina exageradadilatacin muy rpida con contracciones uterinas intensas y
frecuentes que producen un descenso precipitado fetal Se presenta con mayor
frecuencia en pacientes mult paras LA PAC ENTE ES NUL PARA LO QUE
D SM NUYE EL R ESGO DE PARTO PREC P TADO

Bibliografa:DIAGNOSTICOS Y TRATAMIENTOS GINECOOBSTETRICOS. DE CHERNEY ALAN., PERNOLL MARTIN L. MANUAL MODERNO. EDICIN 7. PAG. 434-
437.



21/05/13 18:10 Simulador Proedumed
Pgina 1 de 1 http://www.proedumed.com.mx/simulador/pages/examen/resultadoPregunta.faces
Anlisis del Caso Clnico
Identificacin del reactivo
Area: GINECOLOGA Y OBSTETRICIA
Especialidad: GINECOLOGA
Tema: DOLOR PELVICO
Subtema: DISMENORREA
CASO CLNICO CON UNA PREGUNTA

MUJER DE 32 AOS DE EDAD, QUE ACUDE A LA CONSULTA CON CUADRO CLNICO COMPATIBLE CON DISMENORREA. USTED CONSIDERA QUE SE TRATA DE UNA
DISMENORREA LEVE.

ELEMENTOS CLAVE A CONSIDERAR EN EL CASO CLNICO:
Edad: Mujer de 32 aos de edad
Antecedentes:
Sintomatologa: Dismenorrea es muy importante considerar que se trata de dismenorrea
leve
Exploracin:
Laboratorio y/o gabinete:

15 - EL TRATAMIENTO DE ELECCIN CON EL CUAL SE DEBER TRATAR ESTA PACIENTE ES CON:

MELOXICAM. NH B DORES PREFERENC ALES COX 2 La ciclooxigenasa 2 (COX 2)
Producen prostaglandinas responsables del proceso inflamatorio La mayor a de
los A NES inhibe la actividad de ambas isoformas pero en mayor medida la de la
COX 1 Por esta razn el A NES ideal deber a inhibir solamente el COX 2 lo que
evitar a los efectos adversos como son la hemorragia digestiva una de las ms
peligrosas y frecuentes En diferentes estudios se a visto un beneficio significativo
con el uso de A NES comparados con placebo con un alivio al menos moderado
del dolor en las primeras 8 24 hrs durante 3 5 d as con inicio del tratamiento 24 a
48 hrs antes de la menstruacin en mujeres con ciclos regulares en estos
mismos estudios se vio que no hay diferencia entre los diferentes A NES en el
cuadro N 1 se menciona los diferentes A NES y las dosis La de esta eficacia
llega a un 70 % Estos medicamentos se pueden usar durante 6 meses y hacer
una nueva valoracin para ver la evolucin si han ido bien se pueden mantener
un tiempo ms si fracasan se debe realizar un estudio ms intenso en busca de
otras enfermedades y en caso de que se descarten se pueden usar otros
frmacos Ejemplos Meloxicam Nimesulida Celocoxib Meloxicam es un frmaco
inhibidor selectivo de la Ciclooxigenasa 2 COX 2 del grupo de los antinflamatorios
no esteroideos en este caso no lo utilizamos porque se trata de dismenorrea leve
ACETAMINOFEN. El acetaminofen es un analgsico antipirtico con efecto dbil como inhibidor de la
ciclooxigenasa Produce analgesia disminuyendo el umbral al dolor teniendo
buena tolerancia gastrointestinal indicado en casos de dismenorrea leve
Paracetamol 500mg c/ 8hrs su uso se limita a Dismenorrea leve que no
responde a tratamiento no farmacolgico Cuando esta contraindicada el uso de
A NES por sus efectos adversos
IBUPROFENO. A NES El tratamiento esta fundamentalmente dirigido inhibir la s ntesis de
prostaglandinas a nivel endometrial El mecanismo de accin de los A NES se
basa en la inhibicin de la enzima ciclooxigenasa bloqueando la produccin de
prostaglandinas mediadoras del mecanismo de la inflamacin La ciclooxigenasa
tiene 2 tipos o isoformas que son 1 La ciclooxigenasa 1 (COX 1) producen las
prostaglandinas protectoras de la microcirculacin gstrica heptica y renal
A NES no espec ficos inhibidores de COX 1 COX 2 buprofeno naproxeno
diclofenaco indometacina piroxican El tratamiento con A NES de primera
intencin para disminuir el dolor pero en este caso utilizamos acetaminofen por
tratarse de dismenorrea leve
CIDO
MEFENMICO.
El cido mefenmico es un medicamento antinflamatorio no asteroideo derivado
del cido fenmico o fenamato indicado para el tratamiento del dolor leve o
moderado incluyendo el dolor menstrual por lo general en presentacin oral de
250 mg Aunque no se emplean con tanta frecuencia como otros A NES

Bibliografa: GPC. DIAGNSTICO Y TRATAMIENTO DE DISMENORREA EN EL PRIMER, SEGUNDO Y TERCER NIVEL DE ATENCIN. IMSS-183-09.

http://www.cenetec.salud.gob.mx/descargas/gpc/CatalogoMaestro/183 GPC DISMENORREA/ MSS 183 09 EyR Dismenorrea.pdf


21/05/13 18:10 Simulador Proedumed
Pgina 1 de 1 http://www.proedumed.com.mx/simulador/pages/examen/resultadoPregunta.faces
Anlisis del Caso Clnico
Identificacin del reactivo
Area: GINECOLOGA Y OBSTETRICIA
Especialidad: GINECOLOGA
Tema: LEUCORREA
Subtema: TRICOMONIASIS VAGINAL
CASO CLNICO CON UNA PREGUNTA

MUJER DE 19 AOS DE EDAD, ES LLEVADA POR SU MADRE CON USTED, CON RESULTADO DE LABORATORIO POSITIVO PARA CHLAMYDIA TRACHOMATIS.

ELEMENTOS CLAVE A CONSIDERAR EN EL CASO CLNICO:
Edad: Mujer de 19 aos de edad
Antecedentes: Vida sexual activa
Sintomatologa: asintomatica
Exploracin:
Laboratorio y/o gabinete: Laboratorios positivos para chlamydia

16 - EN ESTE CASO USTED DEBER INDICAR COMO TRATAMIENTO:

AZITROMICINA. La azitromicina es un antibitico macrlido semisinttico activo in Vitro frente a una
amplia gama de microorganismos Gram positivos y Gram negativos El Center for
Disease Control CDC recomienda Azitromicina 1 gr oral dosis nica
Doxiciclina 100 mg oral dos veces al d a por siete d as
DICLOXACILINA. La principal indicacin es el tratamiento de infecciones por estafilococo productor
de penicilinasa neumococo grupo A beta estreptococo hemol tico y penicilina G
resistentes y penicilina G estafilococo sensible Ninguna utilidad en este tipo de
infecciones
AMIKACINA. La amikacina es un antibitico semisinttico del grupo de los aminoglucsidos
derivado de la Kanamicina de accin bactericida Se manifiesta activa "in Vitro"
frente a los siguientes microorganismos Gram negativos Especies de
Pseudomonas Escherich a coli especies de Proteus ( ndol positivos e ndol
negativos) especies de providencia especies de Klebsiella Enterobacter Serrana
especies de Acinetobacter (Mima Herellea) y Citrobacter freundii Gram positivos
Especies de Estafilococos productores y no productores de penicilinaza
incluyendo cepas resistentes a la meticilina No obstante la amikacina es poco
activa frente a otros Gram positivos Streptococcus pyogenes Enterococos y
Streptococcus pneumoniae (Diplococcus pneumoniae) con ninguna utilidad en
este tipo de infecciones
ERITROMICINA. Esta es una opcin de tratamiento tambin estando indicados 500 mg cada 6 hors
por v a oral durante 21 d as Se recomienda que las parejas sexuales en los 60
d as previos se sometan a prueba de infeccin uretral o cervical y reciban
tratamiento tradicional contra Chlamydia

Bibliografa: THE PHARMACOLOGICAL BASIS OF THERAPEUTICS . BRUNTON IL, LAZO JS, PARKER KL. MC. GRAW HILL. 11TH. 2005. PG. 1173.



21/05/13 18:10 Simulador Proedumed
Pgina 1 de 1 http://www.proedumed.com.mx/simulador/pages/examen/resultadoPregunta.faces
Anlisis del Caso Clnico
Identificacin del reactivo
Area: GINECOLOGA Y OBSTETRICIA
Especialidad: OBSTETRICIA
Tema: COMPLICACIONES DEL EMBARAZO Y DEL PARTO
Subtema: ATENCIN DEL PARTO Y DISTOSIAS
CASO CLNICO CON UNA PREGUNTA

FEMENINA DE 25 AOS DE EDAD, GESTA 3, PARTOS 1, CSAREA 1. CURSA CON EMBARAZO DE 38 SEMANAS DE GESTACIN Y TRABAJO DE PARTO. DURANTE LA
VALORACIN EN PLVICA EN ADMISIN SE LE CONSIDERA PLVIS LMITE.

ELEMENTOS CLAVE A CONSIDERAR EN EL CASO CLNICO:
Edad:
Antecedentes: g3 p1 c1
Sintomatologa:
Exploracin: PLV S L M TROFE
Laboratorio y/o gabinete:

17 - EL PROCEDIMIENTO QUE DEBER INDICARSE PARA INTEGRAR EL DIAGNSTICO ES:

PELVIMETRA
SEA
La pelvimetr a ha sido utilizada para pronosticar la necesidad de cesrea en mujeres
cuyos fetos tienen presentacin ceflica por lo que puede influir en la atencin
cl nica No se puede establecer el pronstico de parto vaginal exitoso en algn
embarazo slo con base en la pelvimetr a radiogrfica ya que su sensibilidad y
especificidad es muy baja SE CONS DERA DE UT L DAD L M TADA PARA LA
ATENC N DEL TRABAJO DE PARTO EN LAS PRESENTAC ONES CEFL CAS
POR LO QUE HA CA DO EN DESUSO
TOMOGRAFA
AXIAL
COMPUTADA
DE PELVIS
Las ventajas de la pelvimetr a por TAC en comparacin con la pelvimetr a sea
incluyen menor exposicin a radiacin mayor presicin y mayor desempeo
AUNQUE ES UN MTODO MUY CONF ABLE PARA D AGNOST CAR LA
DESPROPORC N CEFALOPLV CA EL COSTO LA DESCARTA COMO MED DA
D AGNST CA PR MAR A
PRUEBA DE
TRABAJO DE
PARTO
La prueba de trabajo de parto es el procedimiento obsttrico a que se somete una
parturienta con relacin cefaloplvica l mite y mediante su vigilancia y conduccin
sin riesgo materno tiene por objeto conseguir la evolucin del trabajo de parto
venciendo obstculos previstos y presumiblemente franqueables LA PRUEBA DE
TRABAJO DE PARTO ESTA ND CADA EN LA PRESENC A DE PELV S
L M TROFE EST ND CADA EN STA PAC ENTE DEB DO A QUE T ENE UN
PARTO PREV O
ULTRASONIDO
OBSTTRICO
Dados los resultados poco claros de la pelvimetria sea se creo un mtodo para
descubrir la desproporcin cefaloplvica al comparar la circunferencia de la cabeza y
abdominal fetales con los planos de entrada y medio de la pelvis materna a lo que
se le llamo ndice fetoplvico Para los clculos de las dimensiones fetales se utiliza
la ecosonograf a EL ULTRASON DO OBSTTR CO ES DE UT L DAD
N CAMENTE PARA DETERM NAR EL D METRO CRANEAL DEL FETO
REQU ERE DE UNA RAD OGRAF A MATERNA PARA REAL ZAR LA
COMPARAC N MATERNO FETAL ESTE MTODO NO HA RESULTADO SER
PRED CT VO DE DESPROPORC N CEFALOPLV CA

Bibliografa: DIAGNSTICO Y TRATAMIENTO GINECOOBSTTRICOS. ALAN H. DE CHERNEY. MANUAL MODERNO. EDICIN 8A. 2003. PAG. 576-589.



21/05/13 18:11 Simulador Proedumed
Pgina 1 de 1 http://www.proedumed.com.mx/simulador/pages/examen/resultadoPregunta.faces
Anlisis del Caso Clnico
Identificacin del reactivo
Area: GINECOLOGA Y OBSTETRICIA
Especialidad: OBSTETRICIA
Tema: HEMORRAGIA POSTPARTO Y PUERPERIO ANORMAL
Subtema: INFECCIONES POSTPARTO Y PUERPERALES
CASO CLNICO SERIADO

MUJER DE 32 AOS DE EDAD, GESTA 6, PARTOS 5, LTIMO EMBARAZO RESUELTO POR CESAREA HACE 5 DAS. ACUDE A LA CONSULTA POR PRESENTAR DESDE EL DA
DE HOY FIEBRE DE 38.5 GRADOS. A LA EXPLORACIN PRESENTA DISTENSIN ABDOMINAL Y TERO DOLOROSO A LA PALPACIN.

ELEMENTOS CLAVE A CONSIDERAR EN EL CASO CLNICO:
Edad: -
Antecedentes: CESREA HACE 5 DAS
Sintomatologa: FIEBRE DE 38 5C
Exploracin: DISTENSIN ABDOMINAL Y TERO DOLOROSO
Laboratorio y/o gabinete: -

18 - EL DIAGNSTICO CLNICO DE LA PACIENTE ES:

ABSCESO
PLVICO
En a gunas mujeres que presentan matr t s ( nf amac n de tero) despus de a
cesrea a ce u t s parametr a es pronunc ada y forma un rea de ndurac n
que rec be e nombre de f emon dentro de as hojas de gamento ancho En
raras ocas nes un f emon puede ser supurat vo y formar un tumor de gamento
ancho que sobresa e por arr ba de gamento ngu na cons tuyendo un
ABCESO PLVICO Se puede sospechar de sta nfecc n cuando a f erbe
pers ste durante ms de 72hrs pese a med camentos ant mcrob anos ES UNA
COMPLICACIN PRECEDIDA DE ENDOMETRITIS GENERALMENTE Y ES
POCO FRECUENTE
PELVIPERITONITIS La per ton t s y pe v per ton t s es un cuadro grave que se nsta a cuando a
nfecc n se propaga a a pared p v ca y/o per toneo se presenta a f na es de a
pr mera semana de puerper o Los sntomas y s gnos ms frecuentes son:
f ebre e evada genera mente mayor de 40C compromso de estado genera
nuseas y vmtos en cant dad var ab e do or en h pogastr o con oca zac n
preferente hac a una u otra reg n anex a A a exp orac n se pueden detectar
a presenc a de co ecc ones qu das genera mente puru entas y de masas
anex a es f jas o mv es en hemabdomen nfer or y saco de Doug as Es
nfrecuente que ocurra per ton t s despus de a cesrea pero cas de manera
nvar ab e a anteceden metr t s ( nf amac n de tero) necros s de a nc s on
uter na y deh scenc a Otras causas pueden ser por es n de rganos
abdomna es durante a cesrea pero es menos nfrecuente ES UNA
COMPLICACIN GRAVE CON ATAQUE AL ESTADO GENERAL NO HAY
DATOS DE ATAQUE GENERAL Y LA FIEBRE NO COINCIDE CON STA
CONDICIN
CORIOAMNIOITIS E trmno cor oamn on t s def n do estr ctamente se ref ere a a nf amac n o
nfecc n de a p acenta y de cor on y e amn os (membranas feta es) La
mayora de casos de cor oamn on t s son de or gen ascendente con organ smos
de a vag na que ganan acceso en e tracto vag na super or a travs de crv x
por d semnac n hematgena (muy raro) o por proced mentos nvas vos como
a amn ocentes s La cor oamn on t s puede der var en nfecc n o nf amac n de
otros tej dos gestac ona es stos nc uyen a dec dua (dec du t s) as
ve os dades p acentar as (v t s) y e cordn umb ca (fun s t s) La
cor oamn on t s tamb n puede dar ugar a bacter ema materna o feta
RECUERDA QUE LA CORIOAMNIOITIS ES UNA ENFERMEDAD PROPIA DEL
EMBARAZO QUE CONSTITUYE UN FACTOR DE RIESGO EN LA INFECCIN
PUERPERAL PERO QUE NO PERSISTE PROPIAMENTE AS SINO COMO
ENDOMETRITIS EN EL PUERPERIO PUES YA NO EXISTE LAS
MEMBRANAS FETALES
ENDOMETRITIS La endometr t s es a nf amac n de endometr o Se man f esta entre e segundo
y qu nto das de puerper o y const tuye acausa ms frecuente de nfecc n
puerpera acompaada de f ebre en este per odo Los sntomas y s gnos ms
frecuentes son: f ebre e evada de 38-40C ca ofros tero sub nvo uc onado
b ando y do oroso oqu os abundantes oscuros (achoco atados o
seropuru entos) La nfecc n por anaerob os es causa de ntensa fet dez La
endometr t s postparto es 10 veces ms frecuente despus de una cesrea LA
ENDOMETRITIS PUERPERAL CONSTITUYE LA CAUSA MS FRECUENTE
DE INFECCIN PUERPERAL QUE COINCIDE CON LOS DATOS CLNICOS
DE LA PACIENTE

Bibliografa:OBSTETRICIA Y MEDICINA PERINATAL. SAMUEL KARCHMER K. COLEGIO MEXICANO DE ESPECIALISTAS EN GINECOLOGA Y OBSTETRICIA.
EDICIN 1A. 2006. PAG. 511-519.



19 - EL FACTOR PRECIPITANTE MS COMN DEL PADECIMIENTO DE LA PACIENTE ES:

TRABAJO DE
PARTO
PROLONGADO
Los pr nc pa es factores de r esgo para que se desarro e una nfecc n puerpera
son: proced mentos nvas vos de a cav dad uter na con f nes d agnst cos o
teraput cos rotura prematura de membranas pro ongada trabajo de parto
pro ongado tactos m t p es (>5) parto traumt co hemorrag a postparto retenc n
de restos p acentar os y man obras obsttr cas EL TRABAJO DE PARTO ES UN
FACTOR DE RIESGO EN LA GNESIS DE INFECCIN PUERPERAL SIN
EMBARGO NO EST DESCRITO EN EL CUADRO DE LA PACIENTE
LA CESAREA E factor de r esgo ms hab tua para a seps s materna es a cesrea razn por a
cua se nd ca manejo ant mcrob ano prof ct co en todos os casos LA CESREA
ES EL NICO FACTOR DE RIESGO ASOCIADO EN STA PACIENTE ADEMS
DE CONSTITUIR EL ANTECEDENTE PRINCIPAL QUE DESARROLLA
ENDOMETRITIS PUERPERAL
TACTO
VAGINAL
REPETIDO
La rea zac n de tactos repet t vos (>5) E mecan smo de auto nfecc n en a
nfecc n puerpera es a nfecc n de tracto gen ta por grmenes que hab tan en e
organ smo de a mujer de un modo hab tua E mecan smo ms comn esta dado
por e transporte de grmenes de a vag na rea zado por as manos de operador
hac a a cav dad uter na provocado sobre todo con os tactos manua es rea zados
repet t vamente o con def c entes cond c ones de aseps a EL TACTO VAGINAL
REPETITIVO ES CLARO EN LA ETIOPATOGENIA DE LA INFECCIN
PUERPERAL PERO NO ES UN ANTECEDENTE MENCIONADO EN LA
PACIENTE
FLORA
COMN
VAGINAL
La co on zac n bacter ana de a porc n nfer or de aparato gen ta con c ertos
mcroorgan smos como estreptococo de grupo B Ch amyd a trachomat s
Mycop asma homn s Ureap asma urea yt cum y Gardne a vag na s; se ha v ncu ado
con mayor r esgo de nfecc ones puerpera es La mayor parte de as nfecc ones
p v cas en a mujer se producen por mcrof ora endgena de aparato gen ta
femen no EN EL CASO DE CESREA DEBES TENER EN CUENTA EL
ANTECEDENTE DE RUPTURA DE MEMBRANAS CORIOAMNIOITIS Y/O
MANIPULACIN VAGINAL FRECUENTE PARA PODER INCLUIR STA OPCIN
COMO ACERTADA

Bibliografa:OBSTETRICIA Y MEDICINA PERINATAL. SAMUEL KARCHMER K. COLEGIO MEXICANO DE ESPECIALISTAS EN GINECOLOGA Y OBSTETRICIA.
EDICIN 1A. 2006. PAG. 511-519.



20 - EL TRATAMIENTO ANTIBITICO DE PRIMERA ELECCIN INDICADO EN ESTA PACIENTE ES CON:

AMIKACINA Y
CEFTAZIDIMA
Las cefa ospor nas estn nd cadas como monoterap a cuando sta se n c a
emp r camente hasta tener os resu tados de ant b ograma A gua que otros
ant b t cos de amp o espectro se ut zarn acorde a resu tado de ant b ograma o
cuando e proceso no haya remt do con os esquemas bs cos LA ASOCIACIN
DE UNA CEFALOSPORINA DE TERCERA GENERACIN A UN
AMINOGLUCSIDO ES CORRECTA PARA EL MANEJO DE INFECCIN
PUERPERAL SIN EMBARGO DEBES TOMAR EN CUENTA LA PRESENCIA DE
GRMENES ANAEROBIOS ASOCIADOS AL PUERPERIO POSTCESREA POR
LO QUE NO EST INDICADO EN STE MOMENTO
METRONIDAZOL
Y
CEFTRIAXONA
Las cefa ospor nas son de gran ut dad en a nfecc n puerpera s empre y cuando
se admn stren con otros med camentos E metron dazo t enen una buena
act v dad contra anaerob os comb nado con amp c na o una cefa ospor nas y un
amnog ucs do ofrece cobertura contra a mayor parte de os mcroorgan smos
encontrados en as nfecc ones p v cas graves LA ASOCIACIN
METRONIDAZOL-CEFTRIAXONA ES MUY BUENA EN ESTOS CASOS SIN
EMBARGO NO CONSTITUYEN EL MANEJO DE PRIMERA ELECCIN
RECUERDA QUE UN TRIPLE ESQUEMA QUE INCLUYA UNA
CEFALOSPORINA UN AMINOGLUCSIDO Y METRONIDAZOL ES IDEAL
PARA LOGRAR BUENA RESPUESTA TERAPUTICA
CEFOTAXIMA E
IMIPENEN
E mpenem es un carbapenem con cobertura de amp o espectro cntra a mayor
parte de os patgenos que causan metr t s Se admn stra comb nado con
c astat na que nh be e metabo smo rena de mpenem s b en esta comb nac n
es efect va en a mayor arde de os casos de metr t s conv ene reservar a para as
nfecc ones ms graves desde e punto de v sta md co y econmco EL
IMIPENEM EST BIEN INDICADO EN INFECCIONES PUERPERALES DEBE
COMBINARSE CON CILASTATINA PERO SIEMPRE TMALO EN CUENTA
SLO EN CASOS GRAVES
CLINDAMICINA
Y GENTAMICINA
E tratamento de as nfecc ones puerpera es es empr co a n c o y debe
reva orarse a as 72hrs E tratamento n c a de una nfecc n puerpera
postcesarea debe enfocarse a a f ora mxta ten endo en cuenta una cobertura
contra anaerb os La C ndamc na es " a base" de tratamento de as nfecc ones
puerpera es La asoc ac n C ndamc na-gentamc na t enen un nd ce de
reso uc n de 95% LA CLINDAMICINA CONSTITUYE EL MEDICAMENTO IDEAL
PARA LAS INFECCIONES PUERPERALES Y DEBE ASOCIARSE CON
GENTAMICINA PARA AMPLIAR EL ESPECTRO EN INFECCIN
POSTCESREA

Bibliografa:WILLIAMS OBSTETRICIA. F. GARY CUNNINGHAM. PANAMERICANA. EDICIN 21A. 2002. PAG. 577-591.


FIN DEL CASO CLNICO SERIADO

21/05/13 18:11 Simulador Proedumed
Pgina 1 de 1 http://www.proedumed.com.mx/simulador/pages/examen/resultadoPregunta.faces
Anlisis del Caso Clnico
Identificacin del reactivo
Area: GINECOLOGA Y OBSTETRICIA
Especialidad: OBSTETRICIA
Tema: ENFERMEDAD HIPERTENSIVA DEL EMBARAZO
Subtema: PREECLAMPSIA
CASO CLNICO CON UNA PREGUNTA

MUJER DE 26 AOS DE EDAD QUE ACUDE A CONSULTA SOLICITANDO MTODO DE PLANIFICACIN FAMILIAR TEMPORAL.

ELEMENTOS CLAVE A CONSIDERAR EN EL CASO CLNICO:
Edad:
Antecedentes:
Sintomatologa:
Exploracin:
Laboratorio y/o gabinete:


21 - CON BASE EN SU VALORACIN CONSIDERA USTED RECOMENDARLE ANTICONCEPCIN CON HORMONALES ORALES. EL MECANISMO DE ACCIN
PRINCIPAL DE STOS ES:

INHIBICIN DEL
DESARROLLO DEL
ENDOMETRIO
El dispositivo intrauterino liberador de hormonas tiene efectos progestagnicos
esencialmente locales en la cavidad uterina La alta concentracin de
levonorgestrel en el endometrio inhibe la s ntesis endome trial de los receptores
estrgenos insensibilizando el en dometrio al estradiol circulante e induciendo
un potente efecto antiproliferativo NO CORRESPONDE AL MECAN SMO DE
ACC N DE LOS HORMONALES ORALES
SUPRESIN DE LA
SECRECIN DE
GONADOTROPINAS
Los anticonceptivos orales combinados corresponden al mtodo
anticoncepcional hormonal ms utilizado El efecto ms importante de stos es
prevenir la ovulacin al SUPR M R LOS FACTORES H POTALM COS DE LA
HORMONA EST MULANTE DEL FOL CULO Y HORMONA LEUT N ZANTE
INHIBICIN DEL
CUERPO
AMARILLO
Corresponde a uno de los mecanismos de accin del levonorgestrel (p ldora de
emergencia) el cual inhibe la formacin de progesterona a partir del cuerpo
lteo NO CORRESPONDE AL MECAN SMO DE ACC N DE LOS
HORMONALES ORALES
ALTERACIN DEL
MOCO CERVICAL
El aumento de la viscosidad del moco cervical es un efecto producido por los
progestgenos en general por lo que se encuentra presente en todos los
hormonales que lo contienen LOS ANT CONCEPT VOS ORALES
COMB NADOS COMPARTEN ESTA ACC N PERO NO ES EL MECAN SMO
DE ACC N PR NC PAL DE ELLOS

Bibliografa:GOODMAN & GILMAN. THE PHARMACOLOGIC BASIS OF THERAPEUTICS. BRUNTON LL, LAZO JZ, PARKER KL. MCGRAW-HILL. EDICIN 11. PAG.
1564.



21/05/13 18:12 Simulador Proedumed
Pgina 1 de 2 http://www.proedumed.com.mx/simulador/pages/examen/resultadoPregunta.faces
Anlisis del Caso Clnico
Identificacin del reactivo
Area: GINECOLOGA Y OBSTETRICIA
Especialidad: GINECOLOGA
Tema: LESIONES BENIGNAS Y MALIGNAS DE LA MAMA
Subtema: CNCER DE MAMA
CASO CLNICO CON UNA PREGUNTA

MUJER DE 29 AOS DE EDAD, QUE ACUDE A LA CONSULTA PRESENTANDO TELORREA Y RETRACCIN DEL PEZN DE LA MAMA DERECHA. A LA EXPLORACIN SE PALPA
UNA TUMORACIN DE BORDES IRREGULARES DE APROXIMADAMENTE 3 CENTMETROS DE DIMETRO, EN EL CUADRANTE SUPERIOR EXTERNO DE LA MAMA DERECHA.
LA PALPACIN GANGLIONAR ES NEGATIVA.

ELEMENTOS CLAVE A CONSIDERAR EN EL CASO CLNICO:
Edad: Femenina de 29 aos
Antecedentes: No se mencionan factores de riesgo
Sintomatologa:
Exploracin: Masa de 3 cm cuadrante superior externo (ms frecuente en esta
regin) con caracter sticas de malignidad
Laboratorio y/o gabinete:

22 - PARA REALIZAR EL DIAGNSTICO DE ESTA PACIENTE DEBER REALIZARSE:

UNA
MASTOGRAFA.
Los estudios de imagen de un tumor sospechoso comienzan con una mastograf a
que comprende amplificacin compresin o la toma de proyecciones adicionales a
la oblicua lateral y craneal caudal que son las habituales A diferencia de la
mastograf a de deteccin la mastograf a diagnstica se realiza en mujeres de
cualquier edad Los resultados de las imgenes diagnsticas se deben resumir
segn la clasificacin del Breast maging Reporting and Data System (B RADS)
Las lesiones que se clasifican como B RADS 5 son altamente sugestivas de
cncer y > 95% de los casos al final se diagnstica cncer Una clasificacin baja
reduce la posibilidad de cncer Debe realizarse una mastograf a en la paciente
como parte del abordaje diagnstico sin embargo el diagnstico de la masa ya
detectada requiere de una biopsia para determinar el diagnstico con precisin
Debes recordar que si realiza la biopsia antes la mastograf a debe indicarse dos
semanas despus no antes ante la posibilidad de detectar cambios mamarios
secundarios al proceso inflamatorio resultante de la biopsia Adems de que sta
est indicada en mujeres mayores de 40 aos donde la densidad mamaria permite
visualizar mejor lesiones en mama
UN
ULTRASONIDO
MAMARIO.
La ecograf a es de gran utilidad para definir si una masa es qu stica o slida y
forma parte de casi todos los algoritmos para el diagnstico Ciertas caracter sticas
de los tumores slidos como los bordes irregulares los ecos internos o una relacin
entre espesor y altura menor de 1 7cm sugieren cncer En la prctica habitual el
ultrasonido se indica posterior a la solicitud de una mastograf a y muchas veces en
funcin de los hallazgos de esta ltima dada la edad de la paciente como
excepcin puede indicarse antes de la mastograf a sin embargo este estudio no nos
permite hacer un diagnstico definitivo
BAF DE LA
LESIN.
Es muy importante que reconozcas que en la paciente ya se tiene detectada la
masa si bien ser necesario complementar su estudio con mastograf a y
ultrasonido en bsqueda de otras masas no percibidas el diagnstico de la masa
ya detectada en la paciente deber ser patolgico a travs de la biopsia con aguja
fina o cortante La valoracin de un tumor slido siempre concluye con una biopsia
con aguja fina o cortante Como puedes ver todas las respuestas son correctas sin
embargo nuestra recomendacin es siempre considerar el estudio patolgico como
las ms correcta
CITOLOGA DE
SECRECIN
MAMARIA.
La secrecin patolgica del pezn se define como la salida espontnea de material
de un solo conducto que es de aspecto seroso o hemtico El ndice de cncer de
fondo var a de 2 % en mujeres jvenes sin otros datos concomitantes en los
estudios de imagen y la exploracin f sica a 20 % en las mujeres mayores con
otros datos concomitantes La mayor parte de las secreciones patolgicas del
pezn es causada por papilomas intraductales benignos que son plipos simples
de los conductos lact feros La valoracin de la secrecin patolgica del pezn
comienza con la exploracin mamaria que ya se realizo en la paciente Una
valoracin detallada a menudo permite ubicar el punto desencadenante en el borde
de la areola que incita la secrecin al comprimirlo La presencia de sangre oculta y
el examen microscpico de la secrecin ofrecen informacin adicional Tambin se
puede realizar el estudio citolgico de una muestra en una laminilla de vidrio con
una gota de secrecin que se fija de inmediato con alcohol a 95 % Las muestras
de l quido del pezn son acelulares en 25 % de los casos y por lo tanto no permiten
excluir un cncer de fondo No obstante cuando se observan clulas malignas
significa que existe cncer Sin duda en la paciente debe realizarse una citolog a

21/05/13 18:12 Simulador Proedumed
Pgina 2 de 2 http://www.proedumed.com.mx/simulador/pages/examen/resultadoPregunta.faces
del l quido de la secrecin mamaria sin embargo el diagnstico lo dar la biopsia
de la lesin Amabas ser an correctas como parte del abordaje diagnstico sin
embargo el diagnstico de la lesin slo lo puede dar la biopsia de la lesin

Bibliografa: GINECOLOGA DE NOVAK. JONATHAN S. BEREK. MC GRAW HILL. EDICIN 13. 2005. PG. 1099-1117.



21/05/13 18:12 Simulador Proedumed
Pgina 1 de 1 http://www.proedumed.com.mx/simulador/pages/examen/resultadoPregunta.faces
Anlisis del Caso Clnico
Identificacin del reactivo
Area: GINECOLOGA Y OBSTETRICIA
Especialidad: GINECOLOGA
Tema: TRANSTORNOS MENSTRUALES
Subtema: HEMORRAGIA UTERINA ANORMAL
CASO CLNICO SERIADO

MUJER DE 25 AOS DE EDAD, QUE EN LTIMO AO HA PRESENTADO RETRASOS MENSTRUALES DE HASTA 70 DAS Y HEMORRAGIA UTERINA DE HASTA 20 DAS DE
DURACIN. A LA EXPLORACIN SE ENCUENTRA CON PESO DE 85 KILOS, PUBARQUA Y TELARQUA DE ACUERDO A SU EDAD, CON DISTRIBUCIN ANDROIDE DEL VELLO
GENITAL E HIRSUTISMO. SE SOLICITA PERFIL HORMONAL QUE REPORTA ELEVACIN DE LA RELACIN LUTEINIZANTE /FOLCULO ESTIMULANTE (LH/FSH).

ELEMENTOS CLAVE A CONSIDERAR EN EL CASO CLNICO:
Edad: Mujer de 25 aos de edad
Antecedentes: Retrasos menstrua es de hasta 70 das
Sintomatologa: Hemorrag a uter na de 20 das de durac n
Exploracin: Peso 85 Kg d str buc n andro de de ve o gen ta e h rsut smo
Laboratorio y/o gabinete: E evac n de a re ac n LH/FSH

23 - CON BASE EN SUS CICLOS MENSTRUALES LA PACIENTE DEBER CLASIFICARSE COMO:

HIPERMENORREICA. TRASTORNOS DEL CICLO MENSTRUAL: Se ref ere a cua qu er t po de
hemorrag a d ferente de patrn observado en os c c os menstrua es
norma es en su cant dad o durac n E c c o menstrua t ene un rango de
norma dad muy amp o s presenta cada 21- 35 das (28 de med a) dura de
2 a 7das (4 de med a) y se p erden 30-80m de sangre HIPERMENORREA:
Menstruac n abundante y que puede durar os das hab tua es La mayora
de as veces es secundar a a trastornos func ona es hormona es
(H perestron smo por c c o anovu ator o) As msmo puede ser secundar a a
momas p pos endometr a es o trastornos genera es (H pertens n y
coagu opatas)
PROIOMENORREICA. PROIOMENORREA: Presenc a de c c os de menos de 25 das o ade anto de
ms de 5 das en a apar c n de sangrado menstrua
OLIGOMENORREICA. OLIGOMENORREA: Reducc n de nmero de menstruac ones (Con un
nterva o super or a 34 das entre dos perodos de reg a) o de a cant dad de
f ujo menstrua menor de 3 das de durac n Genera mente secundar o a
trastornos fo cu ares
MENORRAGICA. MENORRAGIA: La menorrag a es e trmno md co para e sangrado
exces vo que ocurre durante e per odo menstrua ya sea en nmero de das
en cant dad de sangre o ambos Se ha ocupado en muchas ocas ones como
s nn mo de h permenorrea s n embargo debes tomar en cuenta que ste
t mo trmno s o hace referenc a a a cant dad y a menorrag a nc uye a
durac n Dado que a pac ente t ene una durac n menstrua aumentada (20
das) e trmno correcto es menorrag a

Bibliografa:GINECOLOGA Y OBSTETRICIA APLICADAS. ROBERTO AHUED / CARLOS FERNNDEZ DEL CASTILLO. EL MANUAL MODERNO. EDICIN 2DA. 2003.
PG. 830-831.

http://www.cenetec.salud.gob.mx/descargas/gpc/CatalogoMaestro/082 GPC Miomatosisuterina/MIOMATOSIS RR CENETEC.pdf


24 - EL DIAGNSTICO CLNICO MS PROBABLE ES:

SNDROME DE
TESTCULO
FEMINZATE.
E Sndrome de Testcu o Femnzate o Femn zac n Tota ; es una var edad de
pseudohermafrod t smo mascu no con fenot po femen no se trata de nd v duos
portadores de testcu os con gen ta es extremos de t po femen no en a mayora de
os casos con una vag na que termna en forma c ega y genera mente ausenc a de
tero y anexos Los testcu os se encuentran oca zados ya sea en pos c n
ntrabdomna ngu na o en os ab os mayores E car ot po caracterst co es 46XY
Ante a ausenc a de gen ta es nternos femen nos no hay sangrado menstrua
ENFERMEDAD
POLIQUSTICA
DEL OVARIO.
E sndrome de ovar o po qust co (SOP) es a endocr nopata ms frecuente en
mujeres en edad reproduct va; e d agnst co se hace ante a presenc a de por o
menos dos componentes de a tr ada: O goovu ac n h perandrogen smo y ovar os
po qust cos T ene una comprobada asoc ac n a una res stenc a aumentada a a
acc n de a nsu na que produce h per nsu nema y como consecuenc a una
secrec n anorma de andrgenos Estos camb os endocr nos se ref ejan en
desordenes en e c c o menstrua anovu ac n y exceso de andrgenos La
d str buc n andro de de ve o gen ta e h rsut smo son datos de androgen smo en
nuestra pac ente que acompaados de a terac ones menstrua es fundamenta sta
pos b dad d agnost ca
ADENOMA
HIPOFISIARIO.
Los adenomas de h pf s s son neop as as ben gnas der vadas de as c u as
parenqu matosas de a adenoh pf s s Pueden ser func ona es o no func ona es; os
no func ona es son aque os que no se acompaan de n nguna man festac n de
h persecrec n hormona y en os func ona es ex ste una h persecrec n hormona y
es causante de un sndrome c n co especf co E pro act noma es e tumor ben gno
h pof s ar o asoc ado a a terac ones menstrua es se caracter za por amenorrea y
secrec n ctea No co nc de con e cuadro c n co menstrua y no hay ev denc a de
secrec n mamar a
HIPERPLASIA
ENDOMETRIAL
ATPICA.
La H perp as a Endometr a es una pro ferac n de g ndu as endometr a es de
tamao y forma rregu ar con un aumento de a razn g ndu a-estroma que se
desarro a a consecuenc a de una exces va expos c n a os estrgenos s n que
ex sta opos c n a su efecto pro ferat vo La at p a c to g ca est caracter zada por
un aumento en a estrat f cac n con d spo ar dad ce u ar nc eos h percromt cos
nuc o o promnente cromat na de aspecto grumoso e ncremento de a razn
nc eo-c top asma La presenc a de at p a ce u ar es e factor pronst co ms
mportante para a progres n a carc noma endometr a E s gno pr nc pa es a
hemorrag a uter na anorma aunque puede cursar de manera as ntomt ca La
h perp as a endometr a con at pas es ms frecuente en mujeres mayores de 60
aos o que a descarta como sospecha d agnst ca en nuestra pac ente

Bibliografa:GINECOLOGA Y OBSTETRICIA APLICADAS. ROBERTO AHUED / CARLOS FERNNDEZ DEL CASTILLO. EL MANUAL MODERNO. EDICIN 2DA. 2003.
PG. 830-831.

http://www.cenetec.salud.gob.mx/descargas/gpc/CatalogoMaestro/082 GPC Miomatosisuterina/MIOMATOSIS RR CENETEC.pdf


25 - LA PACIENTE AN ES SOLTERA Y, NO TIENE PLANES DE EMBARAZO A CORTO PLAZO. LA CONDUCTA TERAPUTICA INDICADA SER:

CITRATO DE
CLOMIFENO.
E med camento ms comn que se ut za para nduc r a ovu ac n es e c trato
de c omfeno bajo un rg men de un compr mdo durante c nco das a part r de
segundo da de a menstruac n En c c os anovu ator os e cuerpo teo no se
desarro a e ovar o p erde su capac dad para produc r progesterona y hay
producc n estrogn ca cont nua o que produce una hemorrag a no cc ca
mpredec b e o ncons stente E c trato de c omfeno es e tratamento especf co
para a anovu ac n puede ut zarse como coadyuvante en a madurac n
ovr ca en mujeres que desean embarazarse
PROGESTINA
SINTTICA.
E ant andrgeno ms amp amente ut zado es e acetato de c proterona
(Progesterona s ntt ca con efecto tanto ant gonadotropo como ant androgn co)
La comb nac n de este con et n estrad o ogra a supres n de
h perandrogen smo con a mejora de sntomas c n cos y a norma zac n de as
a terac ones hormona es que caracter zan a sndrome de ovar o po qust co
Dado que a pac ente no desea embarazarse e tratamento debe d r g rse a
reduc r e efecto androgn co y a regu ac n menstrua por o que a
progesterona es a opc n en sta pac ente
BROMOCRIPTINA. La bromocr pt na es un der vado de a ergo na c as f cado dentro de os agon stas
D2 dopamnrg cos que se usa para e tratamento de trastornos h pof sar os y a
Enfermedad de Park nson Uno de os efectos dopamnrg cos sobre a h pf s s
es e antagon smo de a producc n de pro act na por os actotrofos No est
nd cada ya que a pac ente no presenta pro act nema
ESTRGENOS. La fa ta de ovu ac n en e Sndrome de Ovar o Po qust co; ocas ona e estmu o
exces vo y pro ongado de estrgenos sobre e endometr o favorec endo su
crec mento sosten do y s n espec a zac n bajo estas cond c ones e endometr o
puede desarro ar es ones prema gnas y ma gnas dcadas despus Cuando
ex ste ovu ac n cc ca se produce progesterona que estab za e endometr o y
ev ta a apar c n de cncer a a vez que permte a menstruac n mensua mente
en os c c os en os que no hubo embarazo La admn strac n de estrgenos
s mp es "No" est nd cada ante e r esgo de h perp as a o cncer endometr a
secundar o a ya de por s estmu o estrogn co endomentr a sosten do

Bibliografa:GINECOLOGA Y OBSTETRICIA APLICADAS. ROBERTO AHUED / CARLOS FERNNDEZ DEL CASTILLO. EL MANUAL MODERNO. EDICIN 2DA. 2003.
PG. 830-831.

http://www.cenetec.salud.gob.mx/descargas/gpc/CatalogoMaestro/082 GPC Miomatosisuterina/MIOMATOSIS RR CENETEC.pdf

FIN DEL CASO CLNICO SERIADO

21/05/13 18:13 Simulador Proedumed
Pgina 1 de 1 http://www.proedumed.com.mx/simulador/pages/examen/resultadoPregunta.faces
Anlisis del Caso Clnico
Identificacin del reactivo
Area: GINECOLOGA Y OBSTETRICIA
Especialidad: GINECOLOGA
Tema: LEUCORREA
Subtema: CANDIDIASIS VAGINAL
CASO CLNICO CON UNA PREGUNTA

MUJER DE 45 AOS DE EDAD, CON ANTECEDENTE DE GESTA 4, PARA 4, FECHA DE LTIMA MENSTRUACIN HACE 2 AOS, OBESA Y CON DIAGNSTICO RECIENTE DE
DIABETES. ACUDE POR PRESENTAR LEUCORREA DESDE HACE UN MES. A LA EXPLORACIN TOMA USTED UNA MUESTRA DE SECRECIN, EVIDENCIANDO LA
PRESENCIA DE GRUMOS Y UN PH MENOR DE 4.5. USTED CONSIDERA LA PRESENCIA DE VAGINOSIS POR CANDIDA, PERO DURANTE EL INTERROGATORIO COMPRUEBA
QUE LA PACIENTE YA HA RECIBIDO TRATAMIENTOS PREVIO, LOGRNDOSE EVIDENCIAR AL MENOS 4 CUADROS SIMILARES DURANTE EL LTIMO AO.

ELEMENTOS CLAVE A CONSIDERAR EN EL CASO CLNICO:
Edad: Mujer de 45 aos de edad
Antecedentes: G4 P4 FUM hace dos aos d agnst co rec ente de d abetes
Tratamentos prev os con recurrenc a de a enfermedad Muy mportante
Sintomatologa: Leucorrea de un mes de evo uc n
Exploracin: SECRECIN CON GRUMOS
Laboratorio y/o gabinete: PH MENOR DE 4 5

26 - CON BASE EN ESTOS DATOS, EL TRATAMIENTO INDICADO EN ESTE CASO SERA:

CREMA DE
BUTOCONAZOL
MS TABLETAS
VAGINALES DE
CLOTRIMAZOL.
Repaso vag nos s por cnd da Factores de r esgo: Los factores usua mente
dent f cados como asoc ados a a nfecc n de a vag na por hongos son e uso de
ant b t cos de amp o espectro embarazo uso de ant concept vos ora es o nc uso
en a gunos trabajos se menc ona a uso de d spos t vos ntrauter nos como factor
asoc ado As msmo a d abetes y as nfecc ones por VIH deben de ser
cons deradas dentro de este grupo La razn de esta asoc ac n est b en
determnada en e caso de a d abetes ya que es b en conoc do que a g ucosa en
as secrec ones vag na es se encuentra ncrementada y esta cond c n pred spone
e crec mento y a adhes n de cnd da sobrev n endo a nfecc n
CREMA DE
MICONAZOL
MS TABLETAS
VAGINALES DE
NISTATINA.
CUADRO CLNICO Los sntomas tp cos de a CVV cons sten en prur to vu var
acompaado de secrec n vag na que se parece por sus caracterst cas a
requesn S n embargo a secrec n puede var ar entre acuosa y densa de
manera homognea Puede haber do or vag na d spareun a ardor vu var e
rr tac n Puede ocurr r d sur a externa cuando a mcc n produce expos c n de
ep te o vu var y vest bu ar nf amado a a or na E examen reve a er tema y edema
de os ab os y a p e vu var Puede encontrarse es ones pstu opapu osas
per fr cas def n das Qu zs a vag na se encuentre er tematosa y con una
secrec n b anquec na adherente E pH vag na sue e ser norma y a prueba de
o or o de Wh ff es negat va ETIOPATOGENIA Las espec es comnmente
asoc adas a a nfecc n vag na por cnd da son: C a b cans C g abrata C
trop ca s C kruse y C gu ermond Hay que tener en cuenta a ex stenc a de ms
de 100 d ferentes espec es de cnd da e nc uso e C a b cans a espec e ms
frecuente t ene ms de 200 cepas d st ntas La gama es bastante amp a
Aprox madamente 85 % a 90 % de as CVV es deb do a C a b cans La nc denc a
de no-a b cans pr nc pa mente C g abrata est en auge y puede ser a causa de
CVV recurrente y res stente a drogas La vag n t s s ntomt ca ocurre cuando e
hongo o os factores amb enta es vag na es camb an permt endo a pro ferac n
de cnd da Tamb n a transformac n h fa y e aborac n de os factores
patogn cos mct cos (Proteasas pasas mcotox nas etc) E husped desarro a
sntomas y s gnos con berac n de c toqu nas y med adores de a nf amac n
POMADA DE
TIOCONAZOL
MS VULOS
VAGINALES DE
TERCONAZOL.
DIAGNSTICO E d agnst co de cand d as s se puede hacer med ante nspecc n
v sua determnac n de pH vag na mcroscopa Papan co aou prueba de tex y
cu t vo de secrec n cerv covag na Inspecc n v sua E f ujo vag na de una rea
nfecc n por hongos puede tener d ferentes apar enc as Puede estar ausente o
muy d screto o muy f u do b anco con presenc a de p acas en a pared vag na
tp camente como requesn Se debe de sospechar de cnd da s a pac ente t ene
un rash geogrf co s mtr co en a vu va o en e rea per nea Una forma a go
atp ca de presentac n de cnd da es aque a pac ente que t ene una rr tac n
nexp cab e y d sconfort o aque a s n h stor a de d spareun a que n c a mo est as de
quemazn ntra o posco ta rr tac n d sconfort Este prob ema sue e presentarse
en mujeres per y posmenopus cas pH vag na La determnac n de pH vag na
s n duda es de ut dad Un pH norma (<4 5) esenc a mente descarta a pos b dad
de vag nos s y debe de buscarse a presenc a de hongos en a secrec n o
cons derar que todo se encuentra dentro de mtes norma es Un pH mayor a 4 5
sug ere vag nos s bacter ana tr comon as s o endocerv c t s mucopuru enta
Mcroscopa La mcroscopa con KOH o NaOH sue e ser de nva orab e ayuda a
ev denc ar a presenc a de pseudoh fas o a presenc a de a fase mce a para e
d agnst co mct co S so o a evadura est presente es pos b e que so o se trate
de a presenc a de a cnd da como comensa F jac n por tex La f jac n por
tex de a cnd da es otro mtodo de ayuda d agnst ca que t ene una buena
corre ac n de su pos t v dad con nfecc n Es una prueba que se puede hacer en
consu tor o amentab emente no d spon b e en nuestro med o que t ene una
sens b dad versus cu t vo de 82 % para e caso de cnd da 95 % para Gardnere a
y 92 % para Tr chomonas Hay que tener cu dado que esta prueba no debe ser
usada en mujeres as ntomt cas ya que puede dar resu tados fa sos pos t vos de
enfermedad Papan co aou E cnd da tamb n puede ser ev denc ado en frot s de
as secrec ones cerv covag na es con a co orac n de Papan co aou Cu t vo Por
t mo y no por eso menos mportante es e cu t vo de a secrec n vag na que
necesar amente se t ene que hacer en Agar Sabouraud med o de N ckerson E
cu t vo sue e ser espec a mente nd cado en aque as pac entes que han ten do
a guna fa a teraput ca Las espec es d ferentes a C a b cans (C g abrata C
trop ca s) sue en ser de ms d fc reso uc n teraput ca S a nfecc n es mxta
(15 % de os casos) deber de tratarse a a pareja sexua
CLOTRIMAZOL
CREMA MS
FLUCONAZOL
VA ORAL.
Las pac entes que padecen cuatro o ms ep sod os de cand dos s en un ao se
c as f can dentro de grupo de nfecc n comp cada s endo necesar o en estos
casos tomar cu t vos para determnar a et o oga especf ca E tratamento pr mar o
para preven r as recurrenc as de as nfecc ones es e f uconazo por va ora de
100 a 200mg semana es durante se s meses Muchas veces ser necesar o aad r
a tratamento tp co pro ongado va ora

Bibliografa: GINECOLOGIA DE NOVAK . BEREK S. JONATHAN. MC GRAW-HILL. INTERMERICANA. EDICIN 12. 1996. PG. 862, 863, 860.

http://www.cenetec.salud.gob.mx/descargas/gpc/CatalogoMaestro/081 GPC Vaginitisinfec1NA/Vaginitis RR CENETEC.pdf


21/05/13 18:14 Simulador Proedumed
Pgina 1 de 1 http://www.proedumed.com.mx/simulador/pages/examen/resultadoPregunta.faces
Anlisis del Caso Clnico
Identificacin del reactivo
Area: GINECOLOGA Y OBSTETRICIA
Especialidad: OBSTETRICIA
Tema: COMPLICACIONES DEL EMBARAZO Y DEL PARTO
Subtema: CESAREA Y DETERIORO FETAL
CASO CLNICO SERIADO

MUJER DE 23 AOS, GESTA 1, EMBARAZO DE 38 SEG QUE ACUDE A CONSULTA POR PRESENTAR DISMINUCIN DE LOS MOVIMIENTOS FETALES. A LA EXPLORACIN SE
ENCUENTRA BIEN HIDRATADA, ABDOMEN CON FONDO UTERINO A 33 CM POR ARRIBA DE LA SNFISIS DEL PUBIS, FCF 140X'. ABOCADO. TACTO VAGINAL CON CRVIX
CERRADO Y FORMADO.

ELEMENTOS CLAVE A CONSIDERAR EN EL CASO CLNICO:
Edad: --
Antecedentes: gesta 1 embarazo 38sdg
Sintomatologa: d smnuc n de os mov mentos feta es
Exploracin: fondo uter no 33cm fcf 140x abocado cerv x cerrado y formado
Laboratorio y/o gabinete: --

27 - PARA VERIFICAR LAS CONDICIONES FETALES EL ESTUDIO INDICADO EN ESTE MOMENTO ES:

PERFIL BIOFSICO E perf b ofs co (PBF) nc uye a mon tor zac n ecogrf ca de os
mov mentos feta es e tono feta y a resp rac n feta as como a eva uac n
ecogrf ca de vo umen de qu do con o s n ev denc a de a frecuenc a
cardaca feta E PBF se rea za con e objet vo de dent f car a os neonatos
que puedan presentar r esgo de tener un resu tado def c ente de embarazo y
poder rea zar eva uac ones ad c ona es de b enestar nduc r trabajo de parto
o rea zar una cesrea para fac tar e nac mento ES UNA PRUEBA MUY
COMPLETA PARA DETERMINAR EL ESTADO FETAL CUANDO SE
SOSPECHA DETERIORO DEL MISMO LA FRECUENCIA CARDACA
FETAL SE ENCUENTRA DENTRO DE LOS PARMETROS NORMALES Y
NO HAY MAS DATOS DE COMPROMISO MATERNO- FETAL POR LO QUE
NO ES DE ELECCIN EN STE MOMENTO
ULTRASONOGRAFA
PARA MEDIR LA
MADUREZ
PLACENTARIA
C as f cac n U trasonogrf ca de madurez p acentar a/ c as f cac n de
Grannum - Grado 0: Estructura homognea p aca cor a y p aca basa cas no
v s b e - Grado 1: D screto aumento de a ecogen c dad de forma d spersa
p aca cor a un poco ondu ada - Grado 2: P aca cor a con a gunas formas
dentadas: a gunas mgenes ca c f cadas que emp ezan a d bujar os
cot edones - Grado 3: Marcadas formas dentadas que van desde a p aca
cor a a a basa estab ec endo tab ques comp etos; apar c n o aumento de
agunas anecogn cas en e nter or de os cot edones Ter camente no
debera observarse una p acenta grado 3 pasadas as 35 semanas de
gestac n Genera mente est presente en embarazos postrmno LA
VALORACIN DE LA PLACENTA ES DE UTILIDAD EN EMBARAZOS MS
TEMPRANOS DONDE PUDIERA DIAGNOSTICARSE UNA INSUFICIENCIA
PLACENTARIA POR MADUREZ PLACENTARIA PRECOZ AL SER UN
EMBARAZO DE TRMINO SIN DATOS DE SUFRIMIENTO FETAL STE
HALLAZGO NO ES PREDICTIVO AL 100%
PRUEBA SIN
ESTRES
La prueba s n estrs se rea za con un card otocogrfo reg stra
s mu tneamente a frecuenc a card aca feta os mov mentos feta es y as
contracc ones uter nas E trazo se rea za en 20 mnutos en gestas mayores a
28 semanas Se cons dera norma cuando hay 2 o ms per odos de
ace erac n sobre a nea basa de 15 o ms at dos y por ms de 15
segundos Se ap ca cuando se pone en duda e b enestar feta con as
comp cac ones de embarazo que nc uyen embarazo postrmno
mov mentos feta es d smnu dos enfermedad h pertens va restr cc n de
crec mento y hemorrag a en e embarazo ES EL ESTUDIO BSICO A
REALIZAR ANTE UNA HIPOMOTILIDAD FETAL SIN OTROS DATOS QUE
SUGIERAN COMPROMISO FETAL DADO QUE LAS CONDICIONES
MATERNAS SON BUENAS Y LA FCF DENTRO DEL PARMETRO
NORMAL
PRUEBA DE
TOLERANCIA A LA
OXITOCINA
La prueba de to eranc a a a ox toc na prueba con estrs o test de ox toc na
Est basada en e conoc mento de que cada contracc n uter na produce una
d smnuc n de gasto sanguneo en e espac o nterve oso de a p acenta con
un descenso trans tor o en a pres n parc a de oxgeno feta Es una prueba
exce ente para eva uar a reserva func ona feta en sufr mento feta crn co
La prueba est nd cada en todos os trastornos en os que ex ste una
probab dad de que a func n p acentar a est a terada como sera d abetes
gestac ona toxema h pertens n crn ca amenorrea pro ongada sospecha
c n ca o ecogrf ca de retardo en e crec mento ntrauter no NO ES UNA
PRUEBA DE RUTINA Y NO ES DE UTILIDAD PARA VALORAR
HIPOMOTILIDAD FETAL POR LO QUE NO EST INDICADA EN STE
CASO

Bibliografa: DIAGNSTICO Y TRATAMIENTO GINECOOBSTTRICOS. DECHERNEY. MANUAL MODERNO. EDICIN 8A. 2003. PAG. 530.




28 - DURANTE SU VALORACIN LA PACIENTE PRESENTA RUPTURA ESPONTNEA DE MEMBRANAS, SALIENDO LQUIDO TRANSVAGINAL AMARILLENTO. EN
ESTE MOMENTO ESTARA INDICADO:

INICIO DE
ANTIBITICOS
En caso de que se sospeche de nfecc n con ruptura prematura de membranas
(como es e caso ya que e mecon o es factor potenc a de nfecc n) debe tomarse
una muestra de qu do amn t co para cu t vo y ant b ograma y estab ecer un
esquema con ant b ot coterap a adems est nd cado reso ver nmed atamente e
embarazo ndepend entemente de a edad gestac ona y de grado de madurez
pu monar feta LA APLICACIN DE ANTIBITICOS EST INDICADA COMO
MEDIDA COADYUVANTE PARA DISMINUIR EL RIESGO DE INFECCIN ANTE LA
PRESENCIA DE MECONIO SIN EMBARGO NO CONSTITUYE LA ESTRATEGIA
PRIMARIA POR SI SOLA
INDUCTO
CONDUCCIN
DEL PARTO
La Inducc n de Parto es un proced mento d r g do a desencadenar contracc ones
uter nas antes de comenzo de parto espontneo en un ntento de que e parto
tenga ugar por va vag na cuando hay Ind cac n de f na zar a gestac n Est
nd cada a Inducc n de Parto cuando os benef c os de f na zar a gestac n para a
sa ud de a madre y e feto son mayores que os de permt r que e embarazo
cont ne Un mtodo cuant tat vo para predec r resu tado ex toso de a nducc n es
e descr to por B shop en a que hay que puntuar: La d atac n e borramento a
a tura de a presentac n a cons stenc a y a pos c n de crv x DADO QUE HAY
DATOS DE MECONIO QUE SUGIEREN SUFRIMIENTO FETAL Y NO SE
ENCUENTRAN DATOS CERVICALES FAVORABLES PARA LA INDUCCIN DEL
PARTO NO EST JUSTIFICADA ESTA MEDIDA OBSERVACIONES: Recuerda
que e trabajo de parto puede favorecer a asp rac n de mecon o por e feto y que a
nducto conducc n en pr mgestas aumenta a nc denc a de comp cac ones durante
e parto
APLICACIN
DE FORCEPS
E frceps obsttr co es un nstrumento d seado para ayudar a que nazca a cabeza
de feto So o se ut za para ace erar e parto o para correg r anorma dades en a
re ac n cabeza de producto y a pe v s materna y que nterf eran en e descenso de
a cabeza de producto EL USO DE LOS FRCEPS SE RESTRINGE A LAS
ANORMALIDADES DEL PERIODO EXPULSIVO DEL PARTO DADO QUE LA
PACIENTE NO HA INICIADO TRABAJO DE PARTO NO SE JUSTIFICA SU USO
CESREA E d agnst co de sufr mento feta durante e trabajo de parto se sustenta
fundamenta mente en: a terac ones en a frecuenc a cardaca feta pud endo
acompaarse de a presenc a o no de mecon o y a terac ones de pH feta La so a
presenc a de mecon o no es una nd cac n para a termnac n nmed ata de
embarazo este s gno t ene va or para cons derar a nd cac n de cesrea cuando se
acompaa de a terac ones de a frecuenc a cardaca feta h pomot dad feta LA
PRESENCIA DE MECONIO ASOCIADA E HIPOMOTILIDAD FETAL SIN
DESARROLLO DE TRABAJO DE PARTO ESPONTNEO Y CERVIX NO TIL SON
ASPECTOS IMPORTANTES QUE JUSTIFICAN LA INDICACIN DE CESREA

Bibliografa: DIAGNSTICO Y TRATAMIENTO GINECOOBSTTRICOS. DECHERNEY. MANUAL MODERNO. EDICIN 8A. 2003. PAG. 530-532.


FIN DEL CASO CLNICO SERIADO

21/05/13 18:14 Simulador Proedumed
Pgina 1 de 1 http://www.proedumed.com.mx/simulador/pages/examen/resultadoPregunta.faces
Anlisis del Caso Clnico
Identificacin del reactivo
Area: GINECOLOGA Y OBSTETRICIA
Especialidad: OBSTETRICIA
Tema: HEMORRAGIAS DE LA SEGUNDA MITAD DEL EMBARAZO
Subtema: PLACENTA PREVIA
CASO CLNICO SERIADO

MUJER DE 29 AOS DE EDAD, GESTA 4, PARTOS 1. CESREA 2, CURSA CON EMBARAZO DE 33 SEG, SIN CONTROL PRENATAL. ACUDE AL SERVICIO DE URGENCIAS AL
PRESENTAR SALIDA DE LQUIDO TRANSVAGINAL CLARO Y POSTERIORMENTE SANGRADO GENITAL NO MUY ABUNDANTE, INDOLORO E INTERMITENTE.

ELEMENTOS CLAVE A CONSIDERAR EN EL CASO CLNICO:
Edad: -
Antecedentes: MULTIPARA CESREAS PREVIAS EMBARAZO DE 33SDG
Sintomatologa: SANGRADO GENITAL NO MUY ABUNDANTE INDOLORO E
INTERMITENTE
Exploracin: -
Laboratorio y/o gabinete: -

29 - EL DIAGNSTICO CLNICO MS PROBABLE ES:

PLACENTA PREVIA La p acenta prev a cons ste en a mp antac n anorma p acetar a a cua
ocurre a n ve de segmento uter no y que en ocas ones cubre parc a o
tota mente e or f c o cerv ca nterno Es una causa mportante de hemorrag a
en e tercer tr mestre de embarazo se presenta con sangrado vag na rojo
rut ante ndo oro e ntermtente No asoc ado con act v dad uter na Son
factores de r esgo a mu t par dad cesreas prev as o c catr ces uter nas
aborto prev o EL SANGRADO ES CARACTERSTICO ROJO RUTILANTE
INDOLORO INTERMITENTE Y NO ASOCIADO A ACTIVIDAD UTERINA
CORIOAMNIONITIS La cor oamn on t s es un sndrome c n co que se produce por a nvas n
mcrob ana de a cav dad amn t ca Se caracter za por f ebre super or a 37 8C
taqu card a materna y feta h persens b dad uter na y qu do amn t co ft do
LA HEMORRAGIA TRASVAGINAL NO ES UN FACTOR DIAGNSTICO
CLNICO DE CORIOAMNOITIS DEBES CONSIDERAR QUE NO TODOS
LOS CASOS DE CORIOAMNIONITIS PRESENTAN EL CUADRO CLNICO
COMPLETO SIN EMBARGO LA FIEBRE Y TAQUICARDIA FETAL
PERSISTEN EN TODOS LOS CASOS Y NO SON HALLAZGOS
MENCIONADO EN STE CASO CLNICO
DESPRENDIMIENTO
PREMATURO DE
PLACENTA
E desprend mento prematuro de p acenta es a separac n de a p acenta de
su s t o de mp antac n antes de parto C n camente se caracter za por
h pertona uter na o po s sto a y sangrado transvag na profuso que se
acompaa de do or abdomna y repercus n feta en grado var ab e con o s n
trabajo de parto as como presenc a de factores de r esgo (madre mayor a 35
aos mu t par dad c catr ces uter nas prev as y tabaqu smo) STA OPCIN
SE DESCARTA YA QUE A DIFERENCIA DE LAS CARACTERSTICAS DEL
SANGRADO EN LA PACIENTE EL DESPRENDIMIENTO DE PLACENTA DA
UN SANGRADO PROFUSO Y DOLOROSO
CRVICOVAGINITIS La cerv covag n t s es e proceso nf amator o de a vag na y e crv x uter no
acompaados de nfecc n mcrob ana y f ujo vag na E d agnst co
genera mente es c n co y puede conf rmarse con un cu t vo cerv ca LA
PRESENCIA DE SANGRADO TRASVAGINAL NO CONSTITUYE UN DATO
DIAGNSTICO DE CERVICOVAGINITIS

Bibliografa: DIAGNSTICO Y TRATAMIENTO GINECOBSTTRICOS. ALAN H. DE CHERNEY. MANUAL MODERNO. EDICIN 8. 2003. PAG. 395-409.



30 - LA SIGUIENTE OPCIN TERAPUTICA DISMINUIRA DE MANERA IMPORTANTE LA MORBIMORTALIDAD DEL PRODUCTO:

INDUCCIN DE
MADUREZ
PULMONAR
En pac entes con p acenta prev a y/ acreta desprend mento prematuro de
p acenta en forma parc a con 28 a 34 semanas de gestac n y s n nd cac n
de urgenc a qu rrg ca se e deber admn strar esquema de nductores de
madurez pu monar (dexametasona 6mg IM c/12 hrs 4 dos s betametasona
12 mg IM c/24 hrs 2 dos s) EL CASO CLNICO NO NOS OTORGA DATOS
DE COMPROMISO HEMODINMICO MATERNO O SUFRIMIENTO FETAL
POR LO QUE EL PRIMER MANEJO DEBE ESTAR ENCAMINADO A
DISMINUIR LA MORTALIDAD FETAL EN CASO DE SER NECESARIA LA
INTERRUPCIN DEL EMBARAZO TOMANDO EN CUENTA LA EDAD
GESTACIONAL
TOCOLISIS Y
ANTIBITICOS
La toco s s est encamnada a nh b r e trabajo de parto prematuro LA
PACIENTE NO MUESTRA DATOS DE ACTIVIDAD UTERINA POR LO QUE
NO ESTA INDICADA LA TOCOLISIS
INTERRUPCIN DEL
EMBARAZO
En pac entes con prd da sangunea mportante y/ pers stente que ponga en
r esgo a v da est nd cado nterrump r e embarazo a a brevedad pos b e
Datos de descontro hemod nmco materno o compromso feta son
gua mente nd cac ones de nterrupc n de embarazo S empre debe
cons derarse a madurez pu monar feta para mejorar e pronst co feta a
menos que a urgenc a qu rrg ca no o permta LA INTERRUPCIN DEL
EMBARAZO EN PACIENTE HEMODIMICAMENTE ESTABLE SIN DATOS
DE URGENCIA QUIRRGICA EST INDICADA DESPUS DE LA SEMANA
37 DE GESTACIN
HEMOTRANSFUSIN Se recomenda ut zar transfus n de paquete g obu ar cundo se ha perd do
un vo umen sanguneo de 30 a 40 % y en forma urgente s es de ms de 40%
o cundo as concentrac ones de hemog ob na sean <6 g/dL o menores a 10
g/dL y ex sta una perd da rp da de sangre LA PACIENTE NO TIENE DATOS
DE PRDIDA SANGUNEA ABUNDANTE QUE JUSTIFIQUE LA
HEMOTRANSFUSIN EN STE MOMENTO

Bibliografa: DIAGNSTICO Y TRATAMIENTO GINECOBSTTRICOS. ALAN H. DE CHERNEY. MANUAL MODERNO. EDICIN 8. 2003. PAG. 395-409.


FIN DEL CASO CLNICO SERIADO

21/05/13 18:14 Simulador Proedumed
Pgina 1 de 1 http://www.proedumed.com.mx/simulador/pages/examen/resultadoPregunta.faces
Anlisis del Caso Clnico
Identificacin del reactivo
Area: GINECOLOGA Y OBSTETRICIA
Especialidad: OBSTETRICIA
Tema: ENFERMEDAD HIPERTENSIVA DEL EMBARAZO
Subtema: HIPERTENSIN ARTERIAL DURANTE EL EMBARAZO
CASO CLNICO CON UNA PREGUNTA

MUJER DE 35 AOS DE EDAD, QUE ES REFERIDA DE CENTRO DE SALUD POR PRESENTAR CIFRAS TENSIONALES DE 140/95 MMHG Y SINTOMATOLOGA COMPATIBLE
CON VASOESPASMO.

ELEMENTOS CLAVE A CONSIDERAR EN EL CASO CLNICO:
Edad:
Antecedentes: ta 140/95
Sintomatologa: VASOESPASMO
Exploracin:
Laboratorio y/o gabinete:

31 - LO MS PROBABLE ES QUE LA HIPERTENSIN ARTERIAL EN ESTA PACIENTE SEA SECUNDARIA A:

INVASIN
INCOMPLETA
DE LAS
ARTERIAS
ESPIRALES POR
TROFOBLASTO
La invasin defectuosa no logra transformar a las arterias espirales de tubos
reactivos a espacios saculares sin capa muscular y con gran capacidad de
sintetizar vasodilatadores El defecto de placentacin con la consiguiente isquemia
placentaria constituye un aspecto importante en la gnesis de la hipertensin en el
embarazo LA NVAS N DE LAS ARTER AS ESP RALES ES DEFECTUOSA EN
LA H PERTENS N DURANTE EL EMBARAZO
DISMINUCION
DE LA
SENSIBILIDAD A
LA RESPUESTA
DE LA
ANGIOTENSINA
II
A diferencia del embarazo normal las mujeres con preeclamsia eclampsia
PRESENTAN UNA RESPUESTA EXAGERADA A LA ANG OTENS NA Lo
anterior no puede atribuirse a los componentes del sistema Renina Angiotensina
Aldosterona ya que las concentraciones circulantes de la enzima convertidora de
angiotensina se encuentran disminuidas con respecto a mujeres embarazadas
normotensas
AUMENTO DE
LA
PROSTACICLINA
ENDOTELIAL
La prostaciclina derivada del endotelio es un potente vasodilatador antiagregante
plaquetario y estimula la secrecin de renina La deficiencia de prostaciclina
encontrada en la preeclampsia puede resultar en la sensibilizacin de la
angiotensina con una vasoconstriccin secundaria importante EN LA
ENFERMEDAD H PERTENS VA DEL EMBARAZO LA PROSTAC CL NA SE
ENCUENTRA D SM NU DA
BAJA
RESISTENCIA
AL FLUJO EN
LAS ARTERIAS
ESPIRALES
La afeccin de arterias espirales en la preeclampsia supone un deficiente aporte
sangu neo a placenta con efectos de isquemia placentrea e incremento de
deportacin del trofoblasto Esto CONV ERTE AL S STEMA PLACENTAR O
NORMAL DE ALTO FLUJO Y BAJA RES STENC A EN UN S STEMA DE BAJO
FLUJO Y ALTA RES STENC A que resulta en isquemia placentaria que se cree
es el desencadenante de este cuadro cl nico a travs de sustancias liberadas por
el tero o la placenta isqumica que afecta la funcin endotelial ya sea por
liberacin de sustancias vasoconstrictoras o inhibicin de las influencias
vasodilatadoras RECUERDA QUE EL ALTO FLUJO Y BAJA RES STENC A
CORRESPONDEN AL ESTADO NORMA PLACENTAR O

Bibliografa: GINECOLOGIA Y OBSTETRICIA APLICADAS. ROBERTO AHUED. MANUAL MODERNO. EDICIN 2A. 2003. PAG. 463-480.



21/05/13 18:15 Simulador Proedumed
Pgina 1 de 1 http://www.proedumed.com.mx/simulador/pages/examen/resultadoPregunta.faces
Anlisis del Caso Clnico
Identificacin del reactivo
Area: GINECOLOGA Y OBSTETRICIA
Especialidad: OBSTETRICIA
Tema: ENFERMEDADES QUE COMPLICAN EL EMBARAZO
Subtema: INFECCIN DE VAS URINARIAS Y CERVICOVAGINITIS DURANTE
EL EMBARAZO
CASO CLNICO CON UNA PREGUNTA

MUJER DE 32 AOS DE EDAD CON EMBARAZO DE 33 SEG. ES ENVIADA DEL CENTRO DE SALUD POR PRESENTAR INFECCIN DE VAS URINARIAS QUE NO HA
RESPONDIDO AL MANEJO. DESPUS DE TRATAMIENTO ANTIBITICO POR DOS SEMANAS SE REPORTA UROCULTIVO NEGATIVO.

ELEMENTOS CLAVE A CONSIDERAR EN EL CASO CLNICO:
Edad:
Antecedentes: EMBARAZO 33SDG NFECC N DE V AS UR NAR AS DE
REPET C N
Sintomatologa:
Exploracin:
Laboratorio y/o gabinete: UROCULT VO NEGAT VO DESPUS DEL TRATAM ENTO

32 - EL ANTIBITICO INDICADO A PARTIR DE ESTE MOMENTO Y UTILIZADO PARA EVITAR RECADAS ES:

NITROFURANTONA La infeccin urinaria constituye una de las infecciones bacterianas ms
frecuentes durante la gestacin El compromiso infeccioso del tracto urinario
durante el embarazo puede asociarse a complicaciones maternas y
perinatales El principal agente patgeno es la E Coli la nitrofuranto na y
amoxicilina tienen una sensibilidad a sta del 97 y 92% respectivamente El
tratamiento de eleccin para las infecciones de v as urinarias en el embarazo
es la nitrofuranto na oral a 100mg cada 6hrs por 7 d as LA
N TROFURANTO NA ES EL FRMACO DE ELECC N EN EL MANEJO DE
LAS NFECC ONES DE V AS UR NAR AS EN EL EMBARAZO CON UNA
SENS B L DAD AL PR NC PAL PATGENO DEL 97%
CEFTRIAXONA Las cefalosporinas son bien toleradas y seguras durante el embarazo La
cefalexina es la cefalosporina que se usa con mayor frecuencia Las
cefalosporinas de 1 generacin cefalexina cefadroxilo se desaconsejan por
las altas tasas de resistencia Las cefalosporinas de 2 3 generacin
constituyen una alternativa vlida aunque incidencia de recidivas es mayor y
es preciso un tratamiento ms prolongado que otras opciones para lograr la
misma eficacia erradicadora LAS CEFALOSPOR NAS CON UN GRUPO DE
FRMACOS ND CADOS DURANTE EL EMBARAZO NO CORRESPONDEN
A LA PR MERA L NEA DE TRATAM ENTO DADO QUE T ENEN ALTA
RES STENC A BACTER ANA EN NFECC ONES DE V AS UR NAR AS
OFLOXACINA La ofloxacina es un antibitico sinttico del grupo de las fluoroquinolonas se
utiliza para infecciones de v as urinarias agudas y crnicas La ofloxacina se
clasifica en la categor a C del embarazo ya que en estudios con animales
resulto ser embriotxica lo que contraindica su uso en el embarazo LAS
FLUOROQU NOLONAS SON MED CAMENTOS CONTRA ND CADOS
DURANTE EL EMBARAZO
TMP+SMZ El trimetropim con sulfametoazol tiene un efecto inhibidor sobre el metabolismo
de los folatos (su uso en el primer trimestre del embarazo est asociado a
defectos del tubo neural en el recin nacido) Su uso en el tercer trimestre se
ha asociado con ictericia en el recin nacido (desplaza bilirrubina de su unin
con la albmina) EL USO DEL TR METROP M CON SULFAMETOXAZOL
EST CONTRA ND CADO EN EL PR MER Y TERCER TR MESTRE DEL
EMBARAZO Y POR TANTO EN NUESTRA PAC ENTE

Bibliografa: THE PHARMACOLOGICAL BASIS OF THERAPEUTICS. BRUNTON IL, LAZO JS, PARKER KL. MC. GRAW HILL. EDICIN 11TH. PAG. 1122.



21/05/13 18:15 Simulador Proedumed
Pgina 1 de 2 http://www.proedumed.com.mx/simulador/pages/examen/resultadoPregunta.faces
Anlisis del Caso Clnico
Identificacin del reactivo
Area: GINECOLOGA Y OBSTETRICIA
Especialidad: OBSTETRICIA
Tema: ENFERMEDADES QUE COMPLICAN EL EMBARAZO
Subtema: INFECCIN DE VAS URINARIAS Y CERVICOVAGINITIS DURANTE
EL EMBARAZO
CASO CLNICO SERIADO

MUJER DE 19 AOS GESTA 2, PARA 1 CON EMBARAZO DE 33 SEG QUE ACUDE AL SERVICIO DE URGENCIAS, POR PRESENTAR EVACUACIONES DIARREICAS, NAUSEAS,
VOMITOS, DISURIA Y TENESMO VESICAL A LA EXPLORACIN DOLOR EN FOSA ILIACA DERECHA CON IRRADIACION A REGION LUBAR, FIEBRE DE 38 GRADOS. PRESENTA
UNA CONTRACCIN CADA 10 MINUTOS, CERVIX INTERMEDIO BLANDO, CON 40% DE BORRAMIENTO. AMNIOS INTEGRO.

ELEMENTOS CLAVE A CONSIDERAR EN EL CASO CLNICO:
Edad:
Antecedentes: EMBARAZO 33 SDG
Sintomatologa: EVACUAC ONES D ARRE CAS NAUSEAS VOM TOS D SUR A Y
TENESMO VES CAL F EBRE DE 38 GRADOS
Exploracin: DOLOR EN FOSA L ACA DERECHA CON RRAD AC ON A REG ON
LUBAR UNA CONTRACC N CADA 10 M NUTOS CERV X
NTERMED O BLANDO CON 40% DE BORRAM ENTO
Laboratorio y/o gabinete:

33 - EL DIAGNSTICO CLNICO MS PROBABLE ES:

PIELONEFRITIS La pielonefritis aguda es la infeccin de la v a excretora urinaria alta y del
parenquima de uno o varios riones que se acompaa de fiebre escalofr o
malestar general dolor costovertebral y en ocasiones naseas vmito y
deshidratacin LAS NFECC ONES DE V AS UR NAR AS SON LAS MS
FRECUENTES EN EL EMBARAZO RECUERDA QUE LA P ELONEFR T S SE
MAN F ESTA CON S NTOMAS S STM COS (COMO SE MUESTRA EN LA
PAC ENTE) ADEMS DE DESENCADENAR EN MUCHOS DE LOS CASOS
TRABAJO DE PARTO PRETRM NO La diarrea se presenta frecuentemente
al desencadenarse el trabajo de parto de forma refleja
COLECISTITIS
AGUDA
La colecistitis constituye la segunda complicacin quirrgica no obsttrica del
embarazo despus de la apendicitis Durante el embarazo la progesterona
funge como relajante del msculo liso e inhibe la colecistoquinina que ocasiona
aumento del volumen de bilis residual dentro de la vesicula y disminucin de la
contraccin de la ves cula biliar La colecistitis aguda se diagnostica en base a
los signos y s ntomas de inflamacin de la ves cula biliar que pueden ser los
de una peritonitis localizada en el hipocondrio derecho se diferencia del clico
biliar por el dolor constante en el hipocondrio y el signo de Murphy en casos
graves con leve ictericia LA COLEC ST T S ES UNA COMPL CAC N
COMN DURANTE EL EMBARAZO QUE SE ENCUENTRA FAVOREC DA
POR EL AUMENTO DE PROGESTERONA DURANTE LA GESTAC N EL
CUADRO CL N CO CLS CO DE DOLOR EN H POCONDR O Y MURPHY
POS T VO NO ESTN PRESENTES EN NUESTRO PAC ENTE POR LO QUE
NO ES UN D AGNST CO PROBABLE
CISTITIS La cistitis aguda es la infeccin bacteriana del tracto urinario bajo que se
acompaa de los siguientes signos y s ntomas urgencia frecuencia disuria
piuria y hemturia sin evidencia de afectacin sistmica LA F EBRE EN LA
PAC ENTE ES UN DATO DE AFECTAC N S STM CA QUE DESCARTA
ESTA POS B L DAD D AGNST CA
GASTROENTERITIS
INFECCIOSA
Se puede definir la gastroenteritis aguda como una inflamacin de la mucosa
gstrica e intestinal habitualmente de causa infecciosa que va a cursar
cl nicamente con un cuadro de deposiciones l quidas en nmero aumentado
que suele acompaarse de vmitos fiebre y dolor abdominal El s ntoma
principal es la diarrea con aparicin de heces de menor consistencia y/o mayor
nmero las cuales pueden contener moco y/o sangre Otros s ntomas que
pueden aparecer son nuseas vmitos dolor abdominal tipo clico y fiebre
C ERTAMENTE LA PAC ENTE MUESTRA DATOS QUE CO NC DEN CON
STE CUADRO CL N CO S N EMBARGO NO JUST F CA LOS S NTOMAS
UR NAR OS N EL TRABAJO DE PARTO PRETRM NO

Bibliografa: OBSTETRICIA. CUNNIHAM F GARY. PANAMERICANA. EDICIN 21. 2004. PAG. 1064-1065.



34 - EL MANEJO MS INDICADO EN ESTE CASO ES:

21/05/13 18:15 Simulador Proedumed
Pgina 2 de 2 http://www.proedumed.com.mx/simulador/pages/examen/resultadoPregunta.faces

HIDRATACION Y
ANTIBIOTICOTERAPIA
ORAL.
Ante la presencia de cualquier tipo de diarrea siempre est indicado iniciar
hidratacin oral sta medida se considera adecuada La antibioticoterapia
oral est indicada en aquellos casos de infecciones de v as urinarias
distintas de pielonefritis (cistitis bacteriuria asintomtica) el medicamento
de eleccin es nitrofuranto na a razn de 100mg cada 6hrs por 7 d as
RECUERDA QUE EL USO DE ANT B OT COTERAP A ORAL SE L M TA A
LAS NFECC ONES DE V AS UR NAR AS NO COMPL CADAS
ANTIBIOTICOTERAPIA
PARENTERAL.
El manejo de la pielonefritis en cualquier trimestre del embarazo debe
realizarse de forma parenteral Los medicamentos indicados son Amikacina
1 gr ( V c/ 24 hs) ms Ceftriaxona 1gr ( V c/ 24 hs) esquema alterno
Claritromicina 500 mg ( V c/ 12 hs) ms Amikacina 1 gr ( V c/ 24 hs) En la
pielonefritis aguda la hospitalizacin de la paciente tiene como objetivo
manejar la infeccin y vigilar la presencia de posibles complicaciones
obsttricas Cuando la paciente est apirtica 48 72 hs se pueden cambiar
los antibiticos intravenosos a v a oral y valorar el alta hospitalaria y
completar en forma ambulatoria el tratamiento durante 14 d as EL MANEJO
DE LA P ELONEFR T S AGUDA DEBE REAL ZARSE DE FORMA
HOSP TALAR A POR V A PARENTERAL
ANALGSICOS Y
ANTIESPASMDICOS
Al ser la pielonefritis una enfermedad infecciosa es necesaria la utilizacin
de antibiticos Los analgsico y antiespasmdicos slo son coadyuvantes
en el manejo de la paciente
HIDRATACION Y
LOPERAMIDA
La loperamida corresponde a un medicamento categor a B durante el
embarazo por lo que puede ser utilizada Algunas bibliograf as refieren
malformaciones congnitas cuando se utiliza en el primer trimestre del
embarazo EL USO DE LA LOPERAM DA DURANTE EL EMBARAZO EST
RESTR NG DO A AQUELLOS CASOS EN LOS QUE EL BENEF C O
TERAPUT CO ES MAYOR A LOS POS BLES EFECTOS ADVERSOS EN
STE CASO NO EST ND CADO

Bibliografa: OBSTETRICIA. CUNNIHAM F. GARY. PANAMERICANA. EDICIN 21. 2004. PAG. 1066.


FIN DEL CASO CLNICO SERIADO

21/05/13 18:16 Simulador Proedumed
Pgina 1 de 1 http://www.proedumed.com.mx/simulador/pages/examen/resultadoPregunta.faces
Anlisis del Caso Clnico
Identificacin del reactivo
Area: GINECOLOGA Y OBSTETRICIA
Especialidad: GINECOLOGA
Tema: DOLOR PELVICO
Subtema: ENDOMETRIOSIS
CASO CLNICO CON UNA PREGUNTA

MUJER DE 32 AOS DE EDAD, CUYO CUADRO CLNICO SE CARACTERIZA POR TENER UN PATRN CCLICO, DOLOR PLVICO INTENSO, DISPAREUNIA, ESTERILIDAD Y
DISMENORREA.

ELEMENTOS CLAVE A CONSIDERAR EN EL CASO CLNICO:
Edad: Mujer de 32 aos de edad
Antecedentes: Patrn c clico
Sintomatologa: Dolor plvico dispareunia esterilidad y dismenorrea
Exploracin:
Laboratorio y/o gabinete:

35 - EL DIAGNSTICO CLNICO MS PROBABLE ES:

ENFERMEDAD
PLVICA
INFLAMATORIA.
La enfermedad plvica inflamatoria aguda consiste en la infeccin del tero
trompas de Falopio y estructuras plvicas adyacentes no asociadas con cirug a o
embarazo Los criterios cl nicos para el diagnostico Dolor abdominal con o sin
rebote Sensibilidad a la movilizacin del crvix Sensibilidad anexial Los tres
criterios anteriores son necesarios para establecer el diagnstico con uno o ms
de los siguientes Extendido de Gram de endocrvix positivo para diplococos
gram negativos intracelulares temperatura mayor de 38C Leucocitosis (mayor
de 10 000 por c c ) Material purulento (Positivo para leucocitos) en la cavidad
peritoneal obtenido por culdocentesis laparoscopia La paciente del caso
cl nico no presenta esta sintomatolog a
OVARIO
POLIQUSTICO.
Los s ntomas comunes del SOP incluyen Oligomenorrea amenorrea
(irregular pocos o ausencia de per odos menstruales) nfertilidad
generalmente como consecuencia de la anovulacin crnica (falta de ovulacin)
Hirsutismo y el aumento excesivo del vello corporal por lo general en un patrn
masculino que afectan a la cara pecho y piernas La ca da del cabello que
aparece como el adelgazamiento del cabello en la parte superior de la cabeza
Acn piel grasa seborrea Obesidad una de cada dos mujeres con SOP son
obesas Depresin Dolor (Parte daada en el ovario ya sea ovario zq Der )
ENDOMETRIOSIS. Los s ntomas que presentan las pacientes son Dolor (Dispareunia y
dismenorrea) Es el s ntoma ms frecuente de la endometriosis Se localiza
principalmente en el abdomen en la regin lumbar rectal dolores radiantes a
ambas piernas pero en especial la derecha y en la pelvis que puede ser de leve
a severo Hipermenorrea Consiste en sangrados menstruales muy abundantes
tanto en el tiempo como en la cantidad de sangre A veces la metrorragia ocurre
fuera de la menstruacin nfertilidad Dificultad o imposibilidad de quedar
embarazada Trastornos intestinales Como diarrea estreimiento dolor al
defecar o proctalgia Astenia Tambin se llama fatiga Amenorrea Consiste
en ausencia menstrual debido a que el sangrado es interno en la cavidad
abdominal La paciente del caso cl nico cumple con los criterios de endometriosis
por tal motivo se integra el diagnstico
CERVICITIS
CRNICA.
Los s ntomas son Sangrado vaginal anormal despus de las relaciones
sexuales despus de la menopausia entre menstruaciones Flujo vaginal
inusual que no desaparece de color gris blanco o amarillo y puede tener olor
Relaciones sexuales dolorosas Dolor vaginal y plvico La paciente del caso
cl nico no presenta esta sintomatolog a por tal motivo se descarta esta patolog a

Bibliografa: MANUAL CLNICO DE GINECOLOGA. THOMAS G. STOVALL. INTERAMERICANA. EDICIN 2. 1995. PG. 683-685.

http://www cenetec.salud.gob.mx/descargas/gpc/CatalogoMaestro/207 SSA 10 ENDOMETRIOSIS/GRR SSA 207 09 pdf


21/05/13 18:20 Simulador Proedumed
Pgina 1 de 1 http://www.proedumed.com.mx/simulador/pages/examen/resultadoPregunta.faces
Anlisis del Caso Clnico
Identificacin del reactivo
Area: GINECOLOGA Y OBSTETRICIA
Especialidad: GINECOLOGA
Tema: ENFERMEDADES DE TRANSMISIN SEXUAL E INFECCIONES
PLVICAS
Subtema: LINFOGRANULOMA VENEREO
CASO CLNICO SERIADO

MUJER DE 32 AOS DE EDAD, QUE ACUDE A CONSULTA AL PRESENTAR DOLOR INGUINAL DERECHO DESDE HACE 2 SEMANAS QUE SE EXACERBA CON LA
DEAMBULACIN. REFIERE QUE AL COMIENZO DEL DOLOR, PRESENTO UNA LESIN EN LABIOS MAYORES, PERO DESAPARECI MUY RPIDAMENTE. A LA EXPLORACIN
PRESENTA UN CONGLOMERADO DE ADENOMEGALIAS INGUINALES DERECHAS, EDEMA VULVAR Y ESTENOSIS DEL INTROITO VAGINAL.

ELEMENTOS CLAVE A CONSIDERAR EN EL CASO CLNICO:
Edad: Sin importancia.
Antecedentes: -
Sintomatologa: Linfadenopata inguinal dolorosa. Es muy importante la referencia de la
paciente de haber presentado una lesin genital, de muy rpida
resolucin, caracterstica del linfogranuloma venreo.
Exploracin: El conglomerado de linfadenopatas inguinales, ms la estenosis vaginal,
son datos caractersticos del linfogranuloma venreo y que nos ayudan a
hacer diagnstico diferencial.
Laboratorio y/o gabinete: -

36 - EL DIAGNSTICO CLNICO MS PROBABLE ES:

GRANULOMA
INGUINAL.
Esta sera la patologa con la que mas difcilmente podra hacerse el
diagnstico diferencial al momento en el que la paciente solicita la consulta. La
presencia de la lesin genital de evolucin rpida nos permite hacer el
diagnstico, adems de que la paciente no presenta las lesiones caractersticas
de esta enfermedad a nivel inguinal. El granuloma inguinal es una enfermedad
ulcerosa genital tambin conocida como donovanosis y su causa es la bacteria
intracelular, Gram. negativa, Calymmatobacterium (Klebsiella) granulomatis. Es
una bacteria encapsulada y tiene un aspecto caracterstico en la biopsia de
tejido o las muestras de citologa. Al parecer esta enfermedad es poco
contagiosa, requiere de varios contactos y tiene un periodo de incubacin
prolongado de semanas a meses. El granuloma inguinal se manifiesta en forma
de ndulos inflamatorios no dolorosos que degeneran en lceras rojas muy
vascularizadas que sangran fcilmente con el contacto. Si se infectan en forma
secundaria son dolorosas. Estas lceras cicatrizan por fibrosis, lo que origina
una cicatrizacin similar a la queloide. Los ganglios linfticos no suelen
aumentar de tamao, pero algunas veces lo hacen y aparecen lesiones nuevas
a lo largo de estas cadenas linfticas.
LINFOGRANULOMA
VENREO.
La paciente presenta un cuadro clnico muy claro de linfogranuloma venreo.
Para hacer el diagnstico, se deber realizar historia clnica completa ya
existen datos en el examen fsico desde la primera consulta que ayudan a
integrar el diagnstico. En el Linfogranuloma venreo la lcera genital o ppula
desaparece rpidamente. Explorar regiones inguinales y/o femorales en
hombres que tienen sexo con hombres, puede verse manifestado por
proctocolitis o lceras rectales; el periodo de incubacin de la chlamydia es de
5 a 7 das. En esta enfermedad el signo predominante es la linfadenopata
inguinal y/o femoral bilateral dolorosa. Al principio de la enfermedad, la
erupcin vesiculopustulosa inicial puede pasar inadvertida; con la ulceracin
inguinal y (vulvar), el linfedema y la invasin bilateral secundaria, produce una
situacin grave. Durante la fase de formacin de bubones inguinales la ingle
esta hipersensible, el endurecimiento cutneo (con la aparicin de un color rojo
a prpura azulado) constituye una caracterstica notoria. Esto por lo regular
ocurre en 10 a 30 das despus de la exposicin y puede ser bilateral. El
linfedema anorrectal ocurre temprano, la defecacin es dolorosa y las heces
pueden estar teidas con estras de sangre. Posteriormente a medida que el
linfedema y la ulceracin atraviesan por la etapa de la cicatrizacin, la
estenosis rectal vuelve difcil o imposible la defecacin. La estenosis vaginal y
la distorsin pueden terminar en dispareunia grave. En la ltima fase pueden
desarrollarse los sntomas generales (Fiebre, cefalea, artralgias, escalofros y
calambres abdominales).
CHANCRO
BLANDO.
La diferencia principal con esta patologa es la lesin genital caracterstica, que
no presenta la paciente. El chancroide presenta un perodo de incubacin que
oscila entre 3 y 7 das, aunque se han observado variaciones de 1 a 40 das.
Posteriormente, sin la aparicin de sntomas prodrmicos, el chancroide
comienza como una ppula eritematosa o vesiculopstula que en 24-48 horas
evoluciona gradualmente hacia una lesin ulcerada. La lcera puede ser nica,
aunque con frecuencia son mltiples (sobre todo en la mujer), no indurada, con
un dimetro que oscila entre 1 mm y 2 cm., redondeada, con unos bordes
irregulares y socavados, cubierta con un exudado de color amarillo-grisceo y
con una base compuesta por tejido de granulacin que sangra al mnimo roce.
En los mrgenes de la lesin suele apreciarse un discreto componente
inflamatorio, clsicamente llamado Doble borde de Petges4. Las lesiones del
chancroide suelen ser dolorosas, a diferencia de la lcera observada en la
sfilis, aunque en la mujer, y segn el lugar de inoculacin, pueden permanecer
asintomticas o dar sntomas atpicos como dispareunia, dolor en la
defecacin, hemorragia rectal o leucorrea vaginal84. La lesin ulcerosa, tras
una evolucin espontnea, cicatriza en 4-6 semanas, y queda una superficie
lisa o ligeramente deprimida y pigmentada. Las localizaciones tpicas en las
mujeres son los labios mayores y menores, horquilla vulvar, vestbulo y cltoris,
aunque tambin se han descrito lceras vaginales, cervicales y perianales.
Raramente tambin se observan formas extragenitales en la mucosa oral,
dedos, muslos y mamas. La aparicin de una adenopata inguinal dolorosa es
tambin un hallazgo caracterstico del chancroide y puede darse hasta en el 50
% de casos. Suele presentarse de 1 a 2 semanas despus de la instauracin
de la lcera. Inicialmente se desarrolla una linfangitis en forma de numerosas
tumoraciones ganglionares, pequeas, en forma de corona de rosas (Chancro
de Nisbet) que, posteriormente, confluyen para formar un gran grupo
adenoptico (Bubn) El bubn suele ser unilateral, doloroso y unilocular; puede
volverse fluctuante y drenar al exterior un lquido purulento, espeso y cremoso,
dejando una ulceracin que curar lentamente con una lesin cicatricial. Los
bubones son menos frecuentes en las mujeres.
CHANCRO
SIFILTICO.
Las caractersticas de la lesin rpida y de las linfadenopatas en
conglomerado y dolorosas, descartan completamente esta posibilidad
diagnstica. Las dems opciones de respuesta son las patologas con las que
debers hacer diagnstico diferencial ante la presencia de una ulcera en la
regin genital y adenopatas. La sfilis es una enfermedad infecciosa causada
por el Treponema Pallidum, microorganismo procariota en forma de espiral
perteneciente al orden de las espiroquetas, familia Treponematacea. Es un
anaerobio facultativo muy sensible a las condiciones fsicas del medio
ambiente, por lo que no es posible cultivarlo in Vitro en ningn medio
bioqumico, pero s mediante inoculacin en testculo de conejo. Puede
observarse mediante examen microscpico en campo oscuro, o por tcnica de
anticuerpos fluorescentes. Las vas de transmisin son: Exposicin sexual:
corresponden a cerca del 90 % de las infecciones. La contagiosidad va
disminuyendo hacia el segundo ao de la infeccin. Besos: por lesiones
sifilticas primarias o secundarias en los labios o en la cavidad oral.
Transmisin prenatal Transfusin: raro hoy en da dado bsqueda en el
donante. Ms frecuente es por compartir jeringas para inyeccin de drogas
intravenosas. Inoculacin directa accidental en laboratorio. Ante la presencia
de factores de riesgo para ETS y en embarazo la prueba de serologa VDRL
con una titulacin mayor 1:8 se considera positiva para sfilis. Ante un paciente
de VIH positivo puede asociarse otra ETS, como sfilis. INTERROGATORIO Y
EXPLORACIN FSICA. Realizar historia clnica completa, existen datos en el
examen fsico en la primera consulta que ayudan a integrar el diagnstico.
Explorar en bsqueda de adenomegalias inguinales; en algunos casos tiene
presentaciones atpicas con lceras dolorosas, mltiples, purulentas y
destructivas, puede causar balanitis sifiltica de Follman; su periodo de
incubacin promedio es de 3 semanas (3-90 das). Sfilis primaria: El chancro
primario tpico suele comenzar con una sola ppula indolora que pronto se
erosiona y endurece, adquiriendo el borde y la base de la lcera una
consistencia cartilaginosa, muy caracterstica con la palpacin. En los varones
heterosexuales, el chancro suele localizarse en el pene, y en los varones
homosexuales suele encontrarse en el conducto anal o en el recto, en la boca,
o en los genitales externos. En las mujeres, las localizaciones ms frecuentes
son el cuello uterino y los labios vulvares. Por consiguiente, la sfilis primaria
pasa inadvertida con mayor frecuencia en las mujeres y los varones
homosexuales que en los varones heterosexuales. La lesin sifiltica primaria
por lo general conlleva adenopatas regionales que aparecen en la primera
semana tras el comienzo de la infeccin. Los ganglios son indoloros, de
consistencia firme y no supuran. Estas adenopatas son bilaterales y pueden
aparecer tanto en el chancro anal como en el chancro de los genitales
externos. El chancro se cura por lo general en cuatro a seis semanas (lmites
de 2 a 12 semanas), pero las adenopatas pueden persistir meses. Sfilis
secundaria: Entre las manifestaciones protenicas de la sfilis secundaria suelen
contarse lesiones mucocutneas simtricas, circunscritas o difusas, y
linfadenopata generalizada no dolorosa. El chancro primario en fase de
cicatrizacin puede persistir incluso en 15% de los enfermos, y muy a menudo
hay superposicin de fases en individuos que tambin tienen infeccin por VIH,
en comparacin con quienes no la tienen. La erupcin cutnea consiste en
lesiones maculosas, papulosas, papuloescamosas, y a veces pustulosas,
llamadas siflides; con frecuencia coexisten varias lesiones de distinta
morfologa. La erupcin puede ser muy sutil. Al principio, las lesiones son
mculas redondas, separadas, de color rosa o rojo plido, no pruriginoso,
bilateral y simtrico, que miden de 5 a 10 mm de dimetro y estn situadas en
el tronco y la parte proximal de los miembros. Unos das o semanas despus
aparecen tambin lesiones papulosas rojizas de 3 a 10 mm de dimetro. Estas
lesiones, que pueden empeorar hasta formar lesiones necrticas (parecidas a
pstulas) junto a una creciente endarteritis con infiltracin perivascular por
mononucleares, se distribuyen muy ampliamente, con frecuencia afectan a las
palmas de las manos y las plantas de los pies, y pueden aparecer en la cara y
el cuero cabelludo. Hasta en 5% de los casos hay ppulas diminutas llamadas
siflides foliculares que afectan a los folculos pilosos y que pueden producir
placas de calvicie (Alopecia areata) con cada del pelo en el cuero cabelludo,
las cejas o la barba. En las zonas calientes, hmedas e intertriginosas del
cuerpo, como la regin perianal, vulva, escroto, parte interna de los muslos,
axilas y la piel situada bajo las mamas pndulas, las ppulas pueden aumentar
de tamao y sufrir erosiones que dan lugar a lesiones extensas, hmedas,
rosadas o blanco grisceas y muy contagiosas llamadas condilomas planos.
Los sntomas generales que pueden preceder o acompaar a la sfilis
secundaria son: dolor de garganta (15 a 30 %), fiebre (5 a 8 %), prdida de
peso (2 a 20 %), malestar general (25 %), anorexia (2 a 10 %), cefalalgia (10
%) y meningismo (5 %). Slo en 1 a 2% de los casos se observa meningitis
aguda, pero en 30 % de ellos existe un mayor nmero de clulas y de la
concentracin de protenas en el LCR. En 30 % de los casos de sfilis primaria
y secundaria en el LCR se asla a T. pallidum, dato que muchas veces, aunque
no siempre, se vincula con otras alteraciones del lquido cefalorraqudeo. Sfilis
latente: El diagnstico de sfilis latente se establece ante el dato de unas
pruebas serolgicas positivas para la sfilis en el examen normal de lquido
cefalorraqudeo (LCR) y ausencia de las manifestaciones clnicas de esta
enfermedad. El proceso suele sospecharse por los antecedentes de lesiones
de la sfilis primaria o secundaria, por el antecedente de una exposicin a la
sfilis o por el alumbramiento de un lactante afectado de sfilis congnita. La
duracin que ha tenido la infeccin latente (Que influye en la seleccin del
tratamiento apropiado) se puede averiguar cuando existe una prueba
serolgica anterior que dio un resultado negativo, o si se conocen las fechas de
aparicin de las lesiones o del contacto sospechoso. Afeccin del sistema
nervioso central. Tradicionalmente se ha considerado que la neurosfilis es una
manifestacin tarda de la sfilis, lo cual constituye un concepto inexacto. La
sfilis del SNC se extiende por una escala que abarca desde la invasin
temprana (por lo comn las primeras semanas o meses de la infeccin),
pasando por una afeccin asintomtica que dura meses o aos, hasta, en
algunos casos, el surgimiento de manifestaciones neurolgicas tempranas o
tardas. Neurosfilis sintomtica. Aunque a menudo compartan algunas de sus
manifestaciones, las principales formas clnicas de la neurosfilis son la sfilis
menngea, la sfilis meningovascular y la sfilis parenquimatosa. Esta ltima
comprende la parlisis general y la tabes dorsal. Los sntomas de la sfilis
menngea suelen comenzar antes de un ao tras la infeccin; los de la sfilis
meningovascular tardan cinco a 10 aos en aparecer, los de la parlisis general
unos 20 aos, y los de la tabes dorsal de 25 a 30 aos. Sin embargo, desde la
introduccin de los antibiticos, la neurosfilis no suele manifestarse con su
cuadro clnico clsico, sino ms bien como un sndrome mixto, sutil o
incompleto. Otras manifestaciones de la sfilis tarda. Aortitis sifiltica, sfilis
cardiovascular y sfilis benigna tarda (Gomas).

Bibliografa: DIAGNSTICO Y TRATAMIENTO GINECO-OBTTRICOS. ALAN H. DE CHERNEY. MANUAL MODERNO. EDICIN 8A. 2003.



37 - EL TRATAMIENTO DE LA PACIENTE DEBE REALIZARSE CON:

AZITROMICINA. Este antibitico sera de primera eleccin para el chancro blando y la uretritis no
gonoccica, adems de utilizarse de manera combinada con ceftriaxona para el
tratamiento de gonorrea. Debers tener muy en cuenta este dato. Su
denominacin como macrlidos proviene de su estructura, ya que se
encuentran constituidos por un anillo de lactosa macrocclico formado por
muchos miembros, al que se van a unir uno o ms desoxiazcares. La
diferencia entre los compuestos de esta familia precisamente va a estar dada
por la cantidad de tomos que componen la molcula. Por ejemplo, la de
eritromicina est constituida por 14 tomos, sin embargo la azitromicina tiene 15
y ubicados en otras posiciones, adems de que es un compuesto
semisinttico.2 Entre otras propiedades qumicas presentan: poca solubilidad en
agua, tienen aspecto cristalino blanco, son bases dbiles que se inactivan en
medio cido, de ah que se presenten en forma de sales o steres que son ms
resistentes a los cidos, as como que en sus presentaciones orales tengan una
cubierta entrica para protegerlos de la accin de los cidos a nivel del
estmago. Ejercen su actividad antimicrobiana al obstaculizar la sntesis de
protenas en la bacteria a nivel ribosmico, se fijan a la unidad 50 S del mismo,
e impiden la reaccin de translocacin en la cual la cadena de pptido en
crecimiento se desplaza del sitio aceptor al donador, por esta particularidad se
proscribe su combinacin con otras drogas que compiten con un sitio similar de
fijacin en el ribosoma como seran la clindamicina y el cloranfenicol. Su efecto
bactericida o bacteriosttico depende de su concentracin, del microorganismo,
del inculo, su sensibilidad, y de la fase de proliferacin en que se encuentren.
Sealaremos la actividad de los macrlidos frente a cada tipo de
microorganismos, se hace referencia en el caso que constituyan la droga de
eleccin. Frente a cocos Gram. positivos aerobios: tienen una buena actividad
contra Streptococos del grupo A, B y C, neumoniae y viridans. El 50 % de los
estreptococos del grupo (Enterococos) son resistentes. Su actividad para
estafilococos es variable. Cocos Gram. positivos anaerobios: cepas de
peptococos y peptoestreptococos son sensibles a macrlidos. Son poco tiles
contra cocos Gram. negativos anaerobios y los aerobios Gram. negativos. No se
utilizan frente a Neisseria meningitidis, pues penetran poco el sistema nervioso
central. Su efectividad es variable ante cepas de Neisseria gonorrhoeae y s son
efectivas contra Bramhanella catharralis. Constituyen la droga de eleccin en
las sepsis por: Bordetella pertusis, Legionella, Haemophylus ducreyi, Chlamydia
trachomatis, Mycoplasma neumoniae y Ureaplasma urealyticum. Son muy
efectivos contra Campylobacter fetus y yeyuni. Haemophylus influenzae,
ricketsias y Mycobacterium sp, tienen una resistencia variable. Por ltimo,
constituyen una alternativa en el tratamiento contra Treponema pallidum y se
han utilizado con buena efectividad frente a Bacillum antracis, Corinebacterium
difteriae, Actinomices israel y Clostridium tetani.
CIPROFLOXACINA. Granuloma inguinal: Azitromicina 1 g VO cada semana por 3 semanas es el
frmaco de eleccin doxiciclina 100 mg VO cada 12 h por 3 semanas, es el
frmaco alternativo. ciprofloxacino 500 mg VO cada 12 h por 3 semanas es otro
frmaco alternativo.
DOXICICLINA. Linfogranuloma venreo: Doxiciclina 100 mg VO cada 12 h por 21 das es el
frmaco de primera lnea para linfogranuloma venreo. Eritromicina 500 mg VO
cada 6 h por 21 das, es el frmaco alternativo para el tratamiento. La alternativa
al tratamiento de eritromicina es la Azitromicina 1 g VO en dosis nica.
PENICILINA G
BENZATNICA.
El tratamiento para sfilis sigue siendo el mismo desde hace muchos aos.
Recuerda, en la sfilis primaria, secundaria, latente precoz (<1 ao) se
recomienda Penicilina G benzatnica 2.4 millones de UI IM en una sola dosis.
Otras opciones orales (persona alrgica a la penicilina, mujeres no
embarazadas) doxiciclina 100mg cada 12 hrs. por dos semanas o tetraciclina
500mg cada 6 horas por dos semanas. Sfilis latente tarda, terciaria y
cardiovascular. Esquema recomendado: Penicilina G benzatnica, 2.4 millones
de UI Im cada semana por tres dosis. Opcin oral (Personas alrgicas a la
penicilina, mujeres no embarazadas), doxiciclina, 100mg cada 12 hrs. durante 4
semanas.

Bibliografa: DIAGNSTICO Y TRATAMIENTO GINECO-OBTTRICOS. ALAN H. DE CHERNEY. MANUAL MODERNO. EDICIN 8A. 2003.


FIN DEL CASO CLNICO SERIADO

21/05/13 18:21 Simulador Proedumed
Pgina 1 de 1 http://www.proedumed.com.mx/simulador/pages/examen/resultadoPregunta.faces
Anlisis del Caso Clnico
Identificacin del reactivo
Area: GINECOLOGA Y OBSTETRICIA
Especialidad: OBSTETRICIA
Tema: ENFERMEDADES QUE COMPLICAN EL EMBARAZO
Subtema: INFECCIN DE VAS URINARIAS Y CERVICOVAGINITIS DURANTE
EL EMBARAZO
CASO CLNICO CON UNA PREGUNTA

DURANTE LA VALORACIN PRENATAL DE UNA PACIENTE CON UN EMBARAZO DE 18 SEG DIAGNOSTICA INFECCIN DE VAS URINARIAS.

ELEMENTOS CLAVE A CONSIDERAR EN EL CASO CLNICO:
Edad:
Antecedentes: EMBARAZO 18 SDG CON NFECC N DE V AS UR NAR AS
Sintomatologa:
Exploracin:
Laboratorio y/o gabinete:

38 - DEBIDO A SU POTENCIAL TERATOGNICO EN ESTA PACIENTE ESTAR CONTRAINDICADO EL EMPLEO DE:

FLUOROQUINOLONAS Las fluoroquinolonas son antibiticos sinttico se utiliza para infecciones de
v as urinarias agudas y crnicas La ofloxacina se clasifica en la categor a C
del embarazo ya que en estudios con animales resulto ser embriotxica lo
que contraindica su uso en el embarazo LAS FLUOROQU NOLONAS SON
MED CAMENTOS "CONTRA ND CADOS" DURANTE EL EMBARAZO
DADO SU ALTO EFECTO TERATOGEN CO
CEFALOSPORINAS Las cefalosporinas son bien toleradas y seguras durante el embarazo La
cefalexina es la cefalosporina que se usa con mayor frecuencia Las
cefalosporinas de 1 generacin cefalexina cefadroxilo se desaconsejan
por las altas tasas de resistencia Las cefalosporinas de 2 3 generacin
constituyen una alternativa vlida aunque incidencia de recidivas es mayor
y es preciso un tratamiento ms prolongado que otras opciones para lograr
la misma eficacia erradicadora LAS CEFALOSPOR NAS CON UN GRUPO
DE FRMACOS " ND CADOS" DURANTE EL EMBARAZO AUNQUE NO
CONST TUYEN LA PR MERA L NEA DE TRATAM ENTO PARA LAS
NFECC ONES DE V AS UR NAR AS EN GESTANTES
BETALACTMICOS La amoxicilina es una aminopenicilina que pertenece al grupo de los
antibiticos beta lactmicos Es el medicamento ideal de segunda l nea
indicado para el manejo de las infecciones de v as urinarias no complicadas
durante el embarazo a dosis de 500mg cada 8hrs por 7 d as LA
AMOX C L NA UN BETALACTM CO ES EL MED CAMENTO DE 2A
ELECC N EN EL MANEJO DE NFECC ONES DE V AS UR NAR AS EN
EL EMBARAZO
NITROFURANOS La nitrofuranto na es un nitrofurano sinttico indicado como el tratamiento de
eleccin para las infecciones de v as urinarias en el embarazo a dosis oral
de 100mg cada 6hrs por 7 d as LA N TROFURANTO NA ES EL FRMACO
DE ELECC N EN EL MANEJO DE LAS NFECC ONES DE V AS
UR NAR AS EN EL EMBARAZO CON UNA SENS B L DAD AL PR NC PAL
PATGENO E COL DEL 97%

Bibliografa:THE PHARMACOLOGICAL BASIS OF THERAPEUTICS. BRUNTON IL, LAZO JS, PARKER KL. MC. GRAW HILL. EDICIN 11TH. 2005. PAG.
1122.



21/05/13 18:22 Simulador Proedumed
Pgina 1 de 1 http://www.proedumed.com.mx/simulador/pages/examen/resultadoPregunta.faces
Anlisis del Caso Clnico
Identificacin del reactivo
Area: GINECOLOGA Y OBSTETRICIA
Especialidad: OBSTETRICIA
Tema: COMPLICACIONES DEL EMBARAZO Y DEL PARTO
Subtema: ISOINMUNIZACIN RH
CASO CLNICO CON UNA PREGUNTA

THE FIRST-BORN INFANT OF AN RH-NEGATIVE 22-YEAR-OLD WOMAN WHO HAD TWO PREVIOUS SECOND TR MESTER ABORTIONS HAS SEVERE HEMOLYSIS AND
CIRCULATORY FAILURE.

ELEMENTOS CLAVE A CONSIDERAR EN EL CASO CLNICO:
Edad: El primer beb nacido de una mujer de 22 aos
Antecedentes: con Rh negativo que tuvo dos abortos previos en el segundo trimestre
Sintomatologa: tiene insuficiencia circulatoria y hemlisis severa
Exploracin:
Laboratorio y/o gabinete:

39 - THIS CONDITION COULD HAVE BEEN PREVENTED BY TREATING THE MOTHER WITH?

ANTI-RHD IGG DURING THE MOST RECENT
PREGNANCY
gG anti RhD durante el embarazo ms
reciente
ANTI-RHD IGG ON TERMINATION OF EACH OF THE
FIRST TWO PREGNANCIES
gG anti RhD al terminar cada uno de los
dos primeros embarazos
ANTI-RHD IGM DURING THE MOST RECENT
PREGNANCY
gM anti RhD durante el embarazo ms
reciente
ANTI-RHD IGM ON TERMINATION OF THE FIRST
PREGNANCY
gM anti RhD al terminar el primer
embarazo

Bibliografa: -



21/05/13 18:22 Simulador Proedumed
Pgina 1 de 1 http://www.proedumed.com.mx/simulador/pages/examen/resultadoPregunta.faces
Anlisis del Caso Clnico
Identificacin del reactivo
Area: GINECOLOGA Y OBSTETRICIA
Especialidad: OBSTETRICIA
Tema: COMPLICACIONES DEL EMBARAZO Y DEL PARTO
Subtema: RUPTURA PREMATURA DE MEMBRANAS
CASO CLNICO SERIADO

MUJER DE 21 AOS GESTA 2 PARA 1, CON AMENORREA NO CONFIABLE DE 36 SEMANAS. ACUDE A SU CONSULTA POR PRESENTAR SALIDA MODERADA DE LIQUIDO
CLARO TRANSVAGINAL Y SENSACIN DE HUMEDAD VULVAR. A LA EXPLORACIN, FONDO UTERINO A 32 CENTMETROS DE LA SNFISIS DEL PUBIS CON PRODUCTO
CEFLICO Y FRECUENCIA CARDIACA FETAL NORMAL.

ELEMENTOS CLAVE A CONSIDERAR EN EL CASO CLNICO:
Edad:
Antecedentes: 36sdg
Sintomatologa: SAL DA DE L QU DO CLARO TRANSVAG NAL SENSAC N DE
HUMEDAD VULVAR
Exploracin:
Laboratorio y/o gabinete:

40 - EL DIAGNSTICO CLNICO MS PROBABLE ES:

INCONTINENCIA
URINARIA DE
ESFUERZO DEL
EMBARAZO
La incontinencia urinaria es la prdida involuntaria de orina por alteracin del
piso plvico y constituye un diagnstico diferencial importante de ruptura de
membranas Normalmente el lquido amnitico suele ser transparente o
blanquesino sin olor aunque a veces est teido de meconio; el olor
suigeneris de la orina puede hacer el diagnstico diferencial LA
NCONT NENC A UR NAR A DURANTE EL EMBARAZO ES MUY
FRECUENTE Y SE RELAC ONA CON ESFUERZOS LA PAC ENTE NO
PRESENTA ANTECEDENTE DE ESFUERZO POR LO QUE SE DESCARTA
COMO PR MERA OPC N
RUPTURA
PREMATURA DE
MEMBRANAS
La ruptura prematura de membranas es la prdida de continuidad de las
membranas corio amniticas antes del inicio del parto independientemente que
se produzca antes del trmino a trmino o despus del trmino del embarazo
Es importante tomar ste diagnstico principal ya que la rotura de membranas
es significante en el embarazo por tres motivos: 1 Si la presentacin no est
fija en la pelvis aumenta la posibilidad de prolapso y compresin del cordn 2
Es posible que el trabajo de parto empiece poco despus de la ruptura de
membranas 3 Si el trabajo de parto se retrasa de la rotura de membranas se
corre el riesgo de una infeccin uterina ANTE LA PRESENC A DE SAL DA DE
LQU DO TRANSVAG NAL S EMPRE DEBES PENSAR QUE PUEDE
TRATARSE DE UNA RUPTURA DE MEMBRANAS COMO PR MER
D AGNST CO POR LAS COMPL CAC ONES QUE SE DESPREDEN DE
ELLA
CERVICOVAGINITIS El flujo vaginal suele ser ms denso o cremoso ftido asociado en muchas
veces a prurito vaginal GENERALMENTE LA LEUCORREA SE UB CA A
N VEL VAG NAL S B EN PUEDE ENCONTRARSE AL EXTER OR DE STA
POCAS VECES ES TAN ABUNDANTE COMO PARA PRODUC R
SENSAC N DE HUMEDAD VULVAR COMO LO REF ERE NUESTRA
PAC ENTE
EXPULSIN DE
TAPN MUCOSO
El aspecto mucoso con o sin sangre se relaciona generalmente con actividad
uterina prodrmica LA PAC ENTE NO PRESENTA ACT V DAD
PRODRM CA UTER NA QUE NOS HAGA SOSPECHAR DE STE
D AGNST CO

Bibliografa: OBSTETRICIA Y MEDICINA PERINATAL. SAMUEL KARCHMER. COMEGO. EDICIN 1RA. 2006. PAG. 178-180.



41 - PARA CONFIRMAR EL DIAGNSTICO DEBER REALIZAR:

UNA MANIOBRA
DE VALSALVA
Apoyado de la especuloscopa las maniobras de valsalva se ocupan como
auxiliares para el diagnstico de ruptura de membranas El procedimiento
consiste en colorar un espejo vaginal y observar si hay prdida de lquido
amnitico a travs del cuello cervical ES UNA PRUEBA POCO SENS BLE
PARA CONF RMAR EL D AGNST CO GENERALMENTE SE TENDR QUE
ACOMPAAR DE OTRA PRUEBA PARA CONF RMARSE
CULTIVO DE
EXUDADO
CRVICOVAGINAL
El cultivo de exudado vaginal es ideal cuando se sospecha de proceso
infeccioso a ste nivel determina el agente causal especfico de la infeccin NO
ES DE UT L DAD YA QUE NDEPEND ENTEMENTE DEL RESULTADO NO
DESCARTA LA POS B L DAD DE RUPTURA DE MEMBRANAS Recuerda
adems que al tratarse de un cultivo tardar al menos 3 das en arrojar
resultados
TACTO VAGINAL EL TACTO VAG NAL ESTA CONTRA ND CADO EN LA RUPTURA
PREMATURA DE MEMBRANAS ya que en el caso de presentarse la ruptura
favorece la infestacin uterina
CRISTOLOGRAFA La cristalografa se realiza tomando una muestra de liquido del fondo del saco y
se extiende en una laminilla a la cual se le da calor con una lmpara y al
observarla con el microscopio Si la prueba es positiva se visualizan formaciones
en forma de hojas de helecho debido a la concentracin relativa de cloruro de
sodio protena y carbohidratos del lquido amnitico Puede ser falso negativo
en ruptura prematura de membranas de muchas horas de evolucin y en
presencia de sangre o meconio LA CR STALORAFA CONST TUYE UN
MTODO D AGNST CO MUY SENS BLE PARA DETERM NAR RUPTURA
DE MEMBRANAS DEAL EN STE CASO

Bibliografa: OBSTETRICIA Y MEDICINA PERINATAL. SAMUEL KARCHMER. COMEGO. EDICIN 1RA. 2006. PAG. 178-179.



42 - UNA VEZ CONFIRMADO SU DIAGNSTICO SE DEBER:

INDICAR
ANTIBITICO
LOCAL
Luego de 6 horas de producida la RPM se debe iniciar antibioticoterapia profilctica ya
que hay evidencia de que disminuye la sepsis neonatal LA ANT B OT CO TERAP A
DEBE SER S STM CA PROCURANDO UN BUEN ESPECTRO TERAPUT CO
DAR DATOS
DE ALARMA
Y ENVIAR A
DOMICILIO
Se recomienda la hospitalizacin en pacientes con ruptura prematura de membranas
ante el riesgo de infeccin neonatal El manejo mdico expectante es el tratamiento
indicado para las pacientes con ruptura prematura de membranas sin signos clnicos
de infeccin materna ni fetal con edades gestacionales entre 24 y 34 semanas EL
MANEJO EN TODOS LOS CASOS DEBE SER HOSP TALAR O Y NO
AMBULATOR O NO EST ND CADA ESTA MED DA PARA LA EDAD
GESTAC ONAL DE NUESTRA PAC ENTE
MEDIDAS
HIGINICO
DIETTICAS
Y ENVIAR A
UROLOGA
UNA VEZ
RESUELTO
EL
EMBARAZO
Esta es una opcin para la incontinencia del embarazo Cabe mencionar que la
incontinencia puede desaparecer tras el embarazo sin que amerite tratamiento
especfico NO CORRESPONDE A LA TERAP A ESPECF CA PARA EL MANEJO DE
LA RUPTURA DE MEMBRANAS
INDUCIR EL
TRABAJO
DE PARTO
De acuerdo con los lineamiento de la Secretaria de Salud en Mxico la ruptura
prematura de membranas en embarazos de 34 o ms semanas la conducta es la
interrupcin del embarazo Ella se justifica considerando que sobre esta edad
gestacional normalmente existe madurez pulmonar fetal; adems la infeccin es la
causa ms frecuente de mortalidad perinatal y por otra parte existe una relacin
directa entre periodo de latencia e infeccin ovular DADO QUE LA GESTAC N SE
ENCUENTRA EN LA SEMANA 36 ESTA ND CADA LA RESOLUC N DEL
EMBARAZO PARA EV TAR NFECC N NEONATAL

Bibliografa: OBSTETRICIA Y MEDICINA PERINATAL. SAMUEL KARCHMER. COMEGO. EDICIN 1RA. 2006. PAG. 179.


FIN DEL CASO CLNICO SERIADO

21/05/13 18:22 Simulador Proedumed
Pgina 1 de 1 http://www.proedumed.com.mx/simulador/pages/examen/resultadoPregunta.faces
Anlisis del Caso Clnico
Identificacin del reactivo
Area: GINECOLOGA Y OBSTETRICIA
Especialidad: GINECOLOGA
Tema: CLIMATERIO Y MENOPAUSIA
Subtema: CLIMATERIO Y MENOPAUSIA
CASO CLNICO CON UNA PREGUNTA

MUJER DE 50 AOS DE EDAD, CON ANTECEDENTE DE TROMBOSIS VENOSA. ACUDE POR PRESENTAR SNTOMAS SEVEREOS VASOMOTORES Y ATROFIA UROGENITAL.

ELEMENTOS CLAVE A CONSIDERAR EN EL CASO CLNICO:
Edad: mujer de 50 aos probablemente en la perimenopusia
Antecedentes: el dato clave es el antecedente de trombosis venosa que contraindica el
uso te terapia hormonal principalmente con estrgenos
Sintomatologa: sintomatolog a de perimenopausia
Exploracin:
Laboratorio y/o gabinete:

43 - CONSIDERANDO EL RIESGO DE TROMBOSIS ESTA INDICADA LA TERAPIA HORMONAL:

CON
PROGESTGENOS
Entre las contraindicaciones para la terapia hormonal se encuentran Cncer
hormono dependiente (endometrial y de mama) Sangrado uterino anormal no
diagnosticado nsuficiencia venosa complicada nsuficiencia heptica Litiasis
vesicular Trombofilias Antecedentes de eventos tromboemblicos
Dislipidemias La paciente presenta antecedente de eventos tromboembol ticos
lo que contraindica la terapia hormonal Sin embargo los progestgenos
aislados pueden estar indicados en pacientes con sintomatolog a vasomotora
intensa y atrofia vaginal severa en las mujeres que tienen contraindicado el uso
de estrgenos por el antecedente de tromboembolismo o cncer mamario Su
administracin pudiera verse limitada por alguno de sus efectos adversos como
hemorragia vaginal y aumento de peso
COMBINADA
CONTNUA
En las mujeres con tero los progestgenos se combinan con estrgenos prara
reducir el riesgo de cncer endometrial De hecho se puede prescribir
diariamente progestgenos con los estrgenos lo que se denomina terapia
cont nua Sin embargo este mtodo genera amenorrea Se indica en pacientes
con tero Al incluir estrgenos estar a contraindicado
CON
ESTRGENOS VA
TRANSDRMICA
Los estrgenos por v a oral estn contraindicados en pacientes con
antecedentes de tromboembolismo porque al ser metabolizados en h gado
estimulan la produccin heptica de factores de la coagulacin Sin embargo los
parches transdrmicos omiten el efecto de primer paso en el h gado y ofrecen
la conveniencia de una administracin menos frecuentes Esta ser a la
explicacin por la hay bibliograf a que considerar a esta la opcin correcta
Nosotros creemos que como mdico general debers considerar que la terapia
hormonal con estrgenosl esta contraindicada en este tipo de pacientes por eso
consideramos la terapia con estrgenos la respuesta correcta
CCLICO
COMBINADO
Otra opcin es administrar estrgenos durante 25 d as de cada mes y algn
progestgeno durante los ltimos 10 d as Los medicamentos se retiran durante
cinco d as despus de los cules se produce descamacin y hemorragia
endometrial Este tratamiento c clico es el que ms se utiliza durante la
transicin menopusica mientras que el tratamiento continuo se prefiere en las
mujeres despus de la menopausia Al incluir estrgenos estar a contraindicado

Bibliografa: GPC. ATENCIN DEL CLIMATERIO Y MENOPAUSIA.

http://www.cenetec salud.gob.mx/descargas/gpc/CatalogoMaestro/019_GPC_ClimatyMenop/SS_019_08_EyR.pdf


21/05/13 18:23 Simulador Proedumed
Pgina 1 de 1 http://www.proedumed.com.mx/simulador/pages/examen/resultadoPregunta.faces
Anlisis del Caso Clnico
Identificacin del reactivo
Area: GINECOLOGA Y OBSTETRICIA
Especialidad: OBSTETRICIA
Tema: COMPLICACIONES DEL EMBARAZO Y DEL PARTO
Subtema: ATENCIN DEL PARTO Y DISTOSIAS
CASO CLNICO CON UNA PREGUNTA

MUJER DE 28 AOS DE EDAD PRIMIGESTA CON EMBARAZO DE 40 SEG. ACTUALMENTE SE ENCUENTRA EN SALA DE LABOR. DESDE HACE 3 HORAS SE ENCUENTRA CON
3CM DE DILATACIN.

ELEMENTOS CLAVE A CONSIDERAR EN EL CASO CLNICO:
Edad:
Antecedentes: primigesta embarazo 40sdg
Sintomatologa: 3horas con dilatacin de 3cm
Exploracin:
Laboratorio y/o gabinete:

44 - EN ESTE CASO DEBERAMOS CONSIDERAR QUE LA PACIENTE MUY PROBABLEMENTE ESTA CURSANDO CON:

DISTOCIA DE
HOMBROS
La distocia de hombros se produce cuando tras la salida de la cabeza fetal se
detiene la progresin del parto Los factores de riesgo para la distocia de hombros
son aquellos que provocan un crecimiento fetal aumentado o una pelvis reducida
Los factores relacionados con la estenosis plvica la baja talla y los defectos de la
marcha de la madre NO CORRESPONDE YA QUE STA SE DA TRAS LA
SAL DA DE LA CABEZA FETAL Y LA PAC ENTE AUN NO HA D LATADO
COMPLETO PARA QUE STO SUCEDA
DISTOCIA DE
TRABAJO DE
PARTO
La detencin de la fase activa se define como la ausencia de dilatacin durante
2hrs o ms se presenta en el 5% de las nul paras a trmino LA DETENC N DE
LA FASE ACT VA ES UN T PO DE D STOC A DEL TRABAJO DE PARTO
DISTOCIA DE
PARTES
BLANDAS
La distocia de partes blandas es la dificultad del trabajo de parto originado por
alguna de las estructuras q componen al canal de parto o las causadas por la
presencia de rganos o tejidos q al encontrarse en la pelvis materna obstruyen y
desplazan la presentacin Los principales factores de riesgo en stas distocias
son malformaciones uterinas tumores obstructivos del canal del parto cicatrices
uterinas y vaginales ES UNA D STOC A MS FRECUENTE DE MULT PARAS
CON LES ONES CERV CALES QUE NO CORRESPONDEN A LOS
ANTECEDENTES DE NUESTRA PAC ENTE
DISTOCIA DE
PRESENTACIN
La distocia de presentacin se presenta cuando la presentacin es distinta a la
cefalica La mas frecuente de stas distocias se da en la presentacin plvica
cuando las nalgas o las extremidades inferioresdel feto se relacionan directamente
con elestrecho superior de la pelvis materna El parto pretrmino es el factor de
riesgo mas importante en ste caso seguido de multiparidad y alteraciones
uterinas EL SER LA PAC ENTE NUL PARA Y PRESENTAR UN EMBARAZO DE
TRM NO D SM NUYE EL R ESGO PARA STE T PO DE D STOC A

Bibliografa:GINECOLOGIA Y OBSTETRICIA APLICADAS. ROBERTO AHUED / CARLOS FERNANDEZ DEL CASTILLO. MANUAL MODERNO. EDICIN 2. 2003. PAG.
260.



21/05/13 18:23 Simulador Proedumed
Pgina 1 de 1 http://www.proedumed.com.mx/simulador/pages/examen/resultadoPregunta.faces
Anlisis del Caso Clnico
Identificacin del reactivo
Area: GINECOLOGA Y OBSTETRICIA
Especialidad: GINECOLOGA
Tema: LESIONES BENIGNAS Y MALIGNAS DE LA MAMA
Subtema: CNCER DE MAMA
CASO CLNICO CON UNA PREGUNTA

MUJER DE 48 AOS DE EDAD QUE CUENTA CON DIAGNSTICO RECIENTE DE CNCER DE MAMA DUCTAL INFILTRANTE.

ELEMENTOS CLAVE A CONSIDERAR EN EL CASO CLNICO:
Edad: nico factor de riesgo, la edad.
Antecedentes: -
Sintomatologa: Se refiere ya el diagnstico de cncer de mam ductal infiltrante.
Exploracin: -
Laboratorio y/o gabinete: -

45 - ESTA PACIENTE DEBER SER TRATADA MEDIANTE:

RADIOTERAPIA. El CARCINOMA DUCTAL INFILTRANTE O INVASIVO. Se origina en un
conducto lcteo, pero luego las clulas invaden la pared del conducto y se
propagan al tejido adiposo del seno. Entonces puede propagarse a los canales
linfticos, a los vasos sanguneos del seno y a otras partes del cuerpo. Este
tipo de cncer es responsable del 80 % de los casos de cncer de mama. La
finalidad de la intervencin quirrgica y la radioterapia es eliminar el tumor
local o regional resaltando los aspectos estticos y reduciendo al mnimo el
riesgo de una recurrencia circunscrita o regional. Se ha demostrado que estos
mtodos reducen el riesgo de metstasis subsecuentes y por lo tanto
repercuten en la supervivencia. Sin embargo, gran parte de las pacientes con
cncer aparentemente circunscrito posee clulas tumorales detectables en la
sangre o mdula sea en el momento del diagnstico, por lo que el tratamiento
generalizado con quimioterapia, manipulacin hormonal o tratamientos
dirigidos constituye el mtodo principal para reducir el riesgo de metstasis y
muerte. La radioterapia debe utilizarse con manejo coadyuvante posquirrgico
y no como medida nica de tratamiento.
QUIMIOTERAPIA. Cuando se utiliza la quimioterapia complementaria debe administrarse
despus de la ciruga primaria pero antes de la radioterapia. La quimioterapia
neocomplementaria se administra antes de la intervencin quirrgica definitiva
y cada vez es ms popular. Este tipo de quimioterapia permite valorar la
sensibilidad, determinado temor a ciertos frmacos y la reduccin en el
tamao del tumor que stos provocan, permiten llevar a cabo una ciruga
menos agresiva. La quimioterapia moderna del cncer comprende por lo
general a alguna antraciclina como doxorrubicina combinada con
ciclofosfamida.
MASTECTOMA. Esta pregunta es muy sencilla puesto que el cncer de mama siempre
requiere como manejo principal el tratamiento quirrgico. Como puedes ver
todas las respuestas podran ser correctas dado que dependiendo la
estadificacin del tumor se determinar el tratamiento definitivo pero
precisamente para poder estadificar correctamente el cncer de mama es
necesario realizar el tratamiento quirrgico y con base en el tamao e
involucramiento de ganglios o metastasis se determinara cual de los dems
tratamientos (Radioterapia, quimioterapia y hormonoterapia) sern necesarios.
Es importante reconocer que el cncer de mama se debe tratar en un
ambiente multidisciplinario que comprende cirujanos, radioterapeutas y
onclogos mdicos. 1. La Mastectomia: consiste en la extirpacin del seno por
completo incluyendo el pezn. - Mastectoma Simple (Total): Se extirpa todo el
seno pero no se corta ningn ganglio linftico. - Mastectoma Radical
Modificada: Se extirpa todo el seno y algunos ganglios linfticos axilares. -
Mastectoma Radical: Adems se extirpa todo el msculo pectoral. 2.
Tumorectoma: se extirpa slo la masa en el seno y un borde de tejido normal
del seno circundante. "El manejo inicial de todo tumor mamario es quirrgico".
HORMONOTERAPIA. La hormonoterapia complementaria se utiliza para los tumores con receptores
estrognicos. Una opcin es el tamoxifeno, un modificador selectivo de los
receptores estrognicos en mujeres premenopusicas posmenopusicas o
bien, un inhibidor de la aromatasa en mujeres posmenopusicas.

Bibliografa: GINECOLOGA DE NOVAK. JONATHAN S. BEREK. MC GRAW HILL. EDICIN 13. 2005. PG. 1099-1117.



21/05/13 18:24 Simulador Proedumed
Pgina 1 de 1 http://www.proedumed.com.mx/simulador/pages/examen/resultadoPregunta.faces
Anlisis del Caso Clnico
Identificacin del reactivo
Area: GINECOLOGA Y OBSTETRICIA
Especialidad: GINECOLOGA
Tema: AFECCIONES BENIGNAS Y MALIGNAS DEL UTERO
Subtema: CERVISITIS AGUDA Y CRNICA
CASO CLNICO CON UNA PREGUNTA

FEMENINA DE 42 AOS DE EDAD, QUE ACUDE A LA CONSULTA CON DIAGNSTICO DE CERVICITIS CRNICA Y QUE HA SIDO MULTITRATADA DESDE HACE 1 AO.

ELEMENTOS CLAVE A CONSIDERAR EN EL CASO CLNICO:
Edad: Femenina de 42 aos de edad.
Antecedentes: Cervicitis crnica multitratada desde hace 1 ao, recuerda que las
pacientes pueden ser clasificadas en funcin del riesgo en: 1. Mujeres de
bajo riesgo son aqullas sin relaciones sexuales previas. 2. Mujeres de
riesgo moderado son aqullas con relaciones sexuales despus de los
20 aos de edad, y que tienen relaciones habituales con dos parejas
sexuales. 3. Mujeres con alto riesgo inician sus relaciones antes de los
20 aos y tienen ms de dos parejas sexuales.
Sintomatologa: -
Exploracin: -
Laboratorio y/o gabinete: -

46 - UNA VEZ CONCLUIDO EL TRATAMIENTO ETIOLGICO ESPECFICO, LA PACIENTE DEBER SER TRATADA MS PROBABLEMENTE CON:

ELECTROFULGURACIN. La cervicitis crnica se caracteriza por clulas inflamatorias
habitualmente mono nucleares (linfocitos, clulas plasmticas y
macrfagos); es una de las causas ms comunes de leucorrea, aunque
algunas veces puede ser asintomtica u oligoasintomtica y ser
diagnosticada por un exudado endocervical. Tras la constante
estimulacin del cuello uterino por agentes infecciosos el epitelio
responde con una eversin glandular (Ectropin) caracterstica de sta
patologa. El tratamiento de la cervicitis se puede hacer por diferentes
mtodos: 1. Crio-congelado: Cuya tasa de curacin es de un 60 a 70 %.
2. Cono: Este consiste en la extirpacin a travs del asa diatsica
(Electrofulguracin), o el bistur extirpando la lesin completa para ser
biopsiada inmediatamente, por eso es llamada cono biopsia. Su tasa de
curacin va de un 80 a 90%. 3. Histerectoma: La cual cura en un 100%
pero con el inconveniente de que en pacientes que desean embarazos
posteriores, ya no es posible ste. La electrofulguracin es un mtodo
barato y seguro para el manejo de la cervicitis crnica con buenas tasas
de curacin.
ANTI-INFLAMATORIOS,
NO ESTEROIDEOS
SISTMICOS.
Se recomiendan en el tratamiento local de vaginitis inespecfica, vaginitis
post-menopusica (Senil), vaginitis debida a Trichomonas vaginales y
vaginitis por hongos fungus (Monilias) y bacterias sensibles a su accin.
El tratamiento de la vaginitis senil con estos vulos puede necesitar de la
administracin concomitante de estrgenos. Los vulos dexametasona
proporcionan en la embarazada (Cuando se considera necesario) y en la
no embarazada, un control eficaz e indoloro de los desagradables signos
y sntomas que corrientemente se asocian al cuadro de las vaginitis,
como flujo, picazn e inflamacin. En los casos obsttricos, los vulos
han demostrado su utilidad para el control de la vaginitis antes del parto
vaginal. Los AINES pueden ser utilizados de forma concomitante para
disminuir la inflamacin y dolor pero no son el tratamiento especfico.
VULOS DE
FLUOCINOLONA.
Es un corticoesteroide sinttico fluorinado de aplicacin tpica y de
potencia intermedia, comparable a la alclometasona, betametasona,
benzoato, triamcinolona, acetnido de flurandrenlido y dipropionato de
betametasona. El acetnido de fluocinolona se usa como antiinflamatorio
y coadyuvante en el tratamiento de la infeccin en cervico vaginitis.
CREMAS Y DUCHAS
VAGINALES.
Ya no se recomiendan, en muchos casos las duchas vaginales;
modifican el pH de la vagina lo que favorece las infecciones e incrementa
el riesgo de cervicitis crnica.

Bibliografa: DIAGNSTICO Y TRATAMIENTO GINECO-OBTTRICOS. ALAN H. DE CHERNEY. MANUAL MODERNO. EDICIN SPTIMA. 2000. PG. 904-912.



21/05/13 18:24 Simulador Proedumed
Pgina 1 de 1 http://www.proedumed.com.mx/simulador/pages/examen/resultadoPregunta.faces
Anlisis del Caso Clnico
Identificacin del reactivo
Area: GINECOLOGA Y OBSTETRICIA
Especialidad: GINECOLOGA
Tema: AFECCIONES BENIGNAS Y MALIGNAS DEL UTERO
Subtema: HIPERPLASIA ENDOMETRIAL Y CNCER DE ENDOMETRIO
CASO CLNICO SERIADO

MUJER DE 49 AOS DE EDAD NULPARA, DIABTICA E HIPERTENSA DE 7 AOS DE EVOLUCIN. MADRE FINADA POR CNCER UTERINO. MENARCA A LOS 8 AOS, INICIA
VIDA SEXUAL A LOS 19 AOS, ACTUALMETE ACTIVA SEXUALMENTE, NIEGA USO MTODO DE PLANIFICACIN FAMILIAR. HACE 8 AOS FUE SOMETIDA A TRATAMIENTO
POR CNCER DE MAM CON RESOLUCIN EXITOSA, LTIMO CONTROL NEGATIVO. ACUDE A CONSULTA REFIRIENDO CERCA DE 6 MESES CON MENSTRUACIONES
IRREGULARES QUE SE PRESENTAN CON LAPSOS CADA VEZ MS CORTOS DE TIEMPO, EN STE MOMENTO CON SANGRADO ABUNDANTE. A LA EXPLORACIN IMC 30.2,
PALIDEZ EN PIEL, CERVIX EROSIONADO, SANGRADO DE CARACTERSTICAS CATAMENIALES PROFUSO.

ELEMENTOS CLAVE A CONSIDERAR EN EL CASO CLNICO:
Edad: Mujer 45 aos
Antecedentes: CaMa Diabetes Hipertensin Obesidad Menarca temprana
Sintomatologa: Sangrado Uterino Anormal
Exploracin: Sangrado profuso de caractersticas catameniales
Laboratorio y/o gabinete:

47 - EL DIAGNSTICO CLNICO MS PROBABLE ES:

HIPERPLASIA
ENDOMETRIAL
La H PERPLAS A ENDOMETR AL es un crecimiento excesivo del endometrio
que se da por un estmulo mantenido de estrgenos endgenos /o exgenos
que no son contrarrestados por la accin de la progesterona Existen varias
condiciones que pueden predisponer a esto entre las cuales se encuentra la
irregularidad menstrual (por anovulacin obesidad diabetes ovarios
poliqusticos entre otros) tumores productores de estrgenos hiperplasia
adrenocortical mal diseo de la terapia estrognica menopusica etc
FUNDAMENTOS D AGNST COS EN STA PAC ENTE: 1 El uso de
anticonceptivos hormonales reduce el riesgo (la paciente niega uso de medidas
antoconceptivas) 2 La menarca temprana (<12 aos) es un factor de riesgo 3
La mujer nulpara tiene mayor riesgo especialmente si es infertil (todo indica que
la paciente lo es dado que lleva vida sexual sin proteccin niega paridad) 4 El
tejido adiposo puede convertir algunas hormonas en estrgenos y favorecer el
cuadro clnico 5 El tamoxifeno es un medicamento que se usa para prevenir y
tratar el cama antecedente con el que cuenta la paciente 6 La asociacin
diabetes y obesidad AUMENTAN EL R ESGO S B EN PUEDEN SER
FACTORES COMPAT BLES DE H ERPLAS A Y CNCER LA EDAD ES
FUNDAMENTAL PARA ELEG R STE D AGNST CO EL RESULTADO DE
LA B OPS A ES EL ESTNDAR DE ORO PARA CONF RMAR O DESCARTAR
EL D AGNST CO
CNCER
CERVICOUTERINO
El CNCER CERV COUTER NO (CaCu) es una alteracin celular que se origina
en el epitelio del cuello uterino y que se manifiesta inicialmente a travs de
lesiones precursoras de lenta y progresiva evolucin En estadios avanzados
puede producir sangrado intermenstrual sangrado postcoital sangrado
postmenopusico apariencia anormal del crvix (sospecha de malignidad)
descarga vaginal (manchado) y dolor plvico Factores de riesgo: <30 aos sin
control citolgico inmunocompromiso uso anticonceptivos orales infeccin por
virus del papiloma humano mltiples parejas sexuales inicio de vida sexual a
temprana edad deficiencia de cido flico tabaquismo EL SANGRADO
PRODUCTO DEL CACU ES NTERMENSTRUAL O POSTCO TAL LA
PAC ENTE NO T ENE DATOS N ANTECEDENTES CLAROS QUE
FUNDAMENTEN STE D AGNST CO
PREMENOPASIA En los aos antes de la menopausia las mujeres tienen meses cuando no
ovulan Esto puede causar sangrado uterino anormal El engrosamiento del
endometrio es otra cause de sangrado en las mujeres en la cuarta dcada de
vida (puede ser una seal de aviso de cncer del tero importante asegurarse
de que el cncer uterino no es la causa de sangrado anormal) La falta de
estrgeno en el climaterio provoca cese parcial de las menstruaciones poco
sangrado varias menstruaciones al mes o sangrados no relacionados a la
menstruacin es el comienzo de la menopausia En esta etapa se observa la
mayor prevalencia del sndrome climatrico debido a una disminucin gradual
de la funcin ovrica El nmero de folculos primordiales se reduce
presentando una respuesta pobre o nula al estmulo de las gonadotropinas
hipofisiarias y por ende la produccin de estrgenos y progesterona se reduce
El hipoestrogenismo es la causa de la mayora de las alteraciones del sndrome
climatrico que ocurren en este periodo: reduccin de la fertilidad alteraciones
menstruales inestabilidad vasomotora alteraciones del tracto genitourinario
cambios psicolgicos y en la sexualidad EN LA PREMENOPAUS A AL
ALTERAC ONES MENSTRUALES PR NC PALMENTE DE T PO
OL GOMEN RREA C ERTAMENTE LA EDAD DE LA PAC ENTE APUNTA
HAC A STE D AGNST CO PERO SON LOS ANTECEDENTES LOS QUE
PERM TEN DESCARTARLO
CANCER
ENDOMETRIAL
El CNCER DE ENDOMETR O (CE) es una neoplasia glandular maligna que se
origina en la capa interna (endometrio) del cuerpo uterino Es la segunda
neoplasia ginecolgica ms frecuente a nivel mundial El cncer en endometrio
es una neoplasia que produce sntomas an que en sus estadios ms precoces;
el sntoma ms comn es SANGRADO UTER NO ANORMAL Los factores de
riesgo asociados son: Historia de terapia estrognica sustitutiva Tratamiento
con tamoxifeno (de uso en cncer de mama) Menopausia tarda
Nuliparidad nfertilidad o falla teraputica a inductores de ovulacin
Obesidad Diabetes e hipertensin El cncer endometrial se clasifica en tipo y
El tipo es el que se presenta ms frecuentemente la edad de presentacin
es aproximadamente entre los 55 y 65 aos con factores de riesgo antes
mencionados (y que comparte con la hiperplasia endometrial) Son estrgeno
dependientes El tipo estrgeno independiente se presenta en mujeres de
mucha mayor edad que el anterior son pacientes delgadas con hijos es
histolgicamente indiferenciado

Bibliografa:- GUA DE REFERENCIA RPIDA. DIAGNSTICO Y TRATAMIENTO DE LA HIPERPLASIA ENDOMETRIAL EN MUJERES POST-MENOPASICAS EN EL
SEGUNDO NIVEL DE ATENCIN. GPC. CATLOGO MAESTRO: SSA-223-09. - GUA DE PRCTICA CLNICA. DIAGNSTICO Y TRATAMIENTO DEL CNCER DE
ENDOMETRIO. EVIDENCIAS Y RECOMENDACIONES. CATLOGO MAESTRO DE GUAS DE PRCTICA CLNICA: IMSS-478-11. - GUA DE REFERENCIA
RPIDA. DIAGNSTICO Y TRATAMIENTO DEL CNCER DE ENDOMETRIO. GPC. CATLOGO MAESTRO DE PRCTICA CLNICA: IMSS-478-11. - WILLIAMS.
GINECOLOGA. MCGRAW-HILL INTERAMERICANA EDITORES.

http://www.facmed.unam.mx/sg/css/GPC/SIDSS-GPC/gpc/docs/IMSS-478-11-RR.pdf http://www.facmed.unam mx/sg/css/GPC/SIDSS-GPC/gpc/docs/SSA-223-09-RR pdf


48 - UNA VEZ CONFIRMADO EL DIAGNSTICO DEBER INDICARSE:

CONIZACIN
CERVICAL
La CON ZAC N es el procedimiento quirrgico por el que se extrae parte del
crvix uterino en forma de cono para diagnstico histolgico exacto en casos de
C N Ca microinvasivo o lesiones endocervicales En muchos de los casos el
cono es diagnstico y teraputico ste ltimo cuando se extrae por completo la
lesin Debe considerare este tipo de tratamiento para CaCu en forma
excepcional en aquellas pacientes en las cuales sea imperativo la fertilidad
siempre y cuando los mrgenes de la conizacin sean negativos para neoplasia
no haya compromiso de espacios vasculares ni linfticos y sea posible asegurar
un correcto seguimiento de la paciente LA CON ZAC N CERV CAL
CONST TUYE LA TERAP A DE ELECC N PARA EL CNCER
CERV COUTER NO CONF NADO EXCLUS VAMENTE AL CERV X
ADMINISTRACIN
DE ESTRGENOS
CONJUGADOS
Los ETRGENOS CONJUGADOS estn indicados en la mujer en peri o
postmenopasia sin tero para mejorar los sntomas propios de sta etapa La
administracin continua evita la sintomatologa vasomotora que se llega a
presentar con esquemas discontinuos durante el periodo libre de hormonas LOS
ESTRGENOS ESTN CONTRA ND CADOS EN LA H PERPLAS A
ENDOMETR AL ya que sta es una entidad donde la proliferacin de glndulas
endometriales se da por la excesiva exposicin a estrgenos
ABLACIN
ENDOMETRIAL
En pacientes con H PERPLAS A ENDOMETR AL se pueden aplicar los
siguientes tratamientos: Un ciclo de 3 a 6 meses con progestina en dosis bajas
Pastillas anticonceptivas va oral combinadas D U con progesterona
Ablacin endometrial por histeroscopa la cual puede ser curativa en muchos
de los casos Para las mujeres con cavidad endometrial normal la ablacin
histeroscpica endometrial y la ablacin no histeroscpica del sistema
endometrial son equivalentes con respecto al grado de satisfaccin del flujo
menstrual de la paciente en el ao siguiente de ciruga LA ABLAC N
ENDOMETR AL CONST TUYE MANEJO DE PR MERA LNEA EN EL CASO DE
H PERPLAS A ENDOMETR AL
HISTERECTOMA La histerectoma en cuanto a la patologa endometrial tiene su uso en: Cncer
Endometrial (de cualquier tipo) Hiperplasia endometrial atpica Cuando ha
fallado el tratamiento inicial (progesterona anticonceptivos orales ablacin) en la
hiperplasia endometrial no atpica En el caso del cncer endometrial la
histerectoma es til para la estadificacin y el tratamiento primario; esta incluye
lavado peritoneal histerectoma extrafasial con salpingo ooferectoma bilateral
linfadenectoma plvica bilateral y para artica LA H STERECTOMA EST
ND CADA EN EL CASO DE CNCER ENDOMETR AL O H PERPLAS A QUE
NO RESPONDE AL TRATAM ENTO PR MAR O

Bibliografa:- GUA DE REFERENCIA RPIDA. DIAGNSTICO Y TRATAMIENTO DE LA HIPERPLASIA ENDOMETRIAL EN MUJERES POST-MENOPASICAS EN EL
SEGUNDO NIVEL DE ATENCIN. GPC. CATLOGO MAESTRO: SSA-223-09. - GUA DE PRCTICA CLNICA. DIAGNSTICO Y TRATAMIENTO DEL CNCER DE
ENDOMETRIO. EVIDENCIAS Y RECOMENDACIONES. CATLOGO MAESTRO DE GUAS DE PRCTICA CLNICA: IMSS-478-11. - GUA DE REFERENCIA
RPIDA. DIAGNSTICO Y TRATAMIENTO DEL CNCER DE ENDOMETRIO. GPC. CATLOGO MAESTRO DE PRCTICA CLNICA: IMSS-478-11. - WILLIAMS.
GINECOLOGA. MCGRAW-HILL INTERAMERICANA EDITORES.

http://www.cenetec.salud.gob.mx/descargas/gpc/CatalogoMaestro/478 GPC CxncerEndometrio/GER Cxncer de Endometrio.pdf

FIN DEL CASO CLNICO SERIADO

21/05/13 18:25 Simulador Proedumed
Pgina 1 de 1 http://www.proedumed.com.mx/simulador/pages/examen/resultadoPregunta.faces
Anlisis del Caso Clnico
Identificacin del reactivo
Area: GINECOLOGA Y OBSTETRICIA
Especialidad: GINECOLOGA
Tema: TRANSTORNOS MENSTRUALES
Subtema: AMENORREA
CASO CLNICO SERIADO

MUJER DE 24 AOS DE EDAD, CASADA QUE ACUDE A LA CONSULTA PORQUE DESDE HACE 4 AOS DESEA EMBARAZARSE SIN LOGRARLO. DESDE HACE 3 MESES
PRESENTA AMENORREA SECUNDARIA Y GALACTORREA.

ELEMENTOS CLAVE A CONSIDERAR EN EL CASO CLNICO:
Edad: No existe una edad caracterstica
Antecedentes: nfertilidad de 4 aos
Sintomatologa: En la clnica la galactorrea los trastornos menstruales y la esterilidad
sugieren el diagnstico de adenoma hipofisiario
Exploracin:
Laboratorio y/o gabinete: Solicitar siempre niveles de prolactina En caso de encontrarse elevados
debe considerarse siempre la posibilidad de un adenoma hipofisiario de
los cuales slo el 1% degeneran a malignidad Corresponden al 10% de
todos los tumores intracraneales


49 - SOLICITA USTED NIVELES SRICOS DE PROLACTINA, REPORTNDOSE MUY ELEVADOS. PARA CONFIRMAR SU DIAGNSTICO USTED DEBER
SOLICITAR:

CITOLGICO Y
CITOQUMICO DE
LQUIDO
CEFALORRAQUDEO.
Tambin causan hiperprolactinemia ciertos tumores la radioterapia y algunas
enfermedades infiltrantes como sarcoidosis y tuberculosis que lesionan el
tallo hipofisiario e impiden la inhibicin de la secrecin de prolactina
gobernada por la dopamina La paciente no presenta ningn dato clnico para
considerar esta posibilidad diagnstica
PRUEBAS DE
FUNCIN TIROIDEA.
El hipotiroidismo primario tambin se acompaa de elevacin leve de la
prolactina srica La concentracin reducida de hormona tiroidea circulante
provoca una elevacin refleja de la concentracin hipotalmica de TRH por
ausencia de la inhibicin por retroalimentacin La hormona liberadora de
tirotropina se puede unir directamente a los lactotropos de la hipfisis anterior
y estimular la produccin de prolactina En general cuando se confirma un
diagnstico de hiperprolactinemia se deben realizar pruebas de la funcin
tiroidea puesto que algunos pacientes necesitan hormona tiroidea y no la
bsqueda exhaustiva de un adenoma hipofisiario La paciente no cuenta con
datos clnicos para considerar este diagnstico Los datos clnicos seran
cansancio debilidad muscular dolores musculares contracturas musculares
sndrome del tnel del carpo piel seca y escamosa cada de pelo bradilalia
alopecia edema de parpados depresin ansiedad apata etc
TOMOGRAFA DE
CRNEO.
En nuestro medio este podra ser el estudio al que se tendra acceso esta
podra ser la respuesta correcta si no se tuviera la opcin de la resonancia
magntica o si no se estuviera actualizado Ante la presencia de las dos
opciones la ventaja de diagnosticar adenomas en fase ms tempranas hace
a la resonancia magntica la opcin correcta
RESONANCIA
MAGNTICA DE
CRNEO.
Por definicin cualquier paciente con prolactina srica elevada padece
hiperprolactinemia Los adenomas secretores de prolactina tambin
llamados prolactinomas son los adenomas hipofisiarios ms frecuentes y
adems son los que con ms frecuencia diagnostican los gineclogos La
mayora de los pacientes manifiesta micro adenomas y signos de
hiperprolactinemia como galactorrea y amenorrea Actualmente se
recomienda como estudio diagnstico realizar una resonancia magntica en
todo paciente con hiperprolactinemia confirmada Su principal ventaja con
respecto a la tomografa es que es capaz de diagnosticar adenomas muy
pequeos que en la tomografa pasaran desapercibidos con lo que se
mejora el pronstico de la paciente

Bibliografa: GINECOLOGIA DE NOVAK. BEREK S. JONATHAN. MC GRAW-HILL. INTERAMERICANA. EDICIN 12. 1996. PG. 862.



50 - EL TRATAMIENTO DE LA PACIENTE DEBER SER:

LEVOTIROXINA
SDICA.
Slo en caso de que la etiologa fuera hipotiroidismo esta sera la opcin
correcta en nuestra paciente La levotiroxina es absorbida incompleta y de
manera variable desde el tracto gastrointestinal Su vida media es de
aproximadamente 7 das en una persona normal y suele acortarse en estados
hipertiroideos y alargarse en estados hipotiroideos Una vez en sangre la
levotiroxina se une casi por completo a las protenas plasmticas y
principalmente a la globulina fijadora de tiroxina (TBG) aunque tambin lo hace
con la proalbmina y en menor grado con la albmina La fraccin libre aunque
solo representa un 0 03 % de la tiroxina total es la disponible para la accin
perifrica y para la conversin al metabolito ms activo triyodotironina cuya
potencia es casi un 30 % mayor que la correspondiente a la levotiroxina La
levotiroxina se distribuye a todos los lquidos y tejidos del organismo con
mayores concentraciones en hgado y rin La alteracin en los niveles de las
protenas plasmticas puede afectar la concentracin total pero no la fraccin
libre de tiroxina De esta manera el embarazo o el incremento en los niveles
plasmticos de estrgenos incrementa la concentracin de TBG mientras que
el sndrome nefrtico la disminuye La tiroxina es parcialmente convertida a
triyodotironina y otros metabolitos Algunos conjugados con cido glucurnico y
cido sulfrico son eliminados por la bilis Parte del material conjugado es
hidrolizado en el colon y eliminado por las heces al igual que el 20 % de la
tiroxina Parte de la tiroxina libre y los metabolitos desionizados son eliminados
por la orina Las enfermedades renal o heptica no parecen afectar
marcadamente la disponibilidad de tiroxina La dosis se ajustar de acuerdo con
los requerimientos y respuesta individual de cada paciente por lo que la edad
condicin fsica severidad y duracin del hipotiroidismo determinarn la dosis
inicial y de mantenimiento La dosis inicial en pacientes geritricos debe ser
baja en pacientes con padecimientos de larga evolucin y otras
endocrinopatas o evidencia de enfermedad cardio vascular e hipotiroidismo
severo deben ser sujetos a pruebas constantes y apropiadas de laboratorio
Para el tratamiento de hipotiroidismo ligero en adultos la dosis oral inicial de L
tiroxina sdica es de 50 mcg diariamente una sola dosis incrementndose de
25 a 50 mcg ms diariamente en intervalos de 2 a 4 semanas hasta obtener la
dosis deseada sin embargo en pacientes hasta ahora sanos pero con inicio
reciente de hipotiroidismo se indican dosis iniciales de 100 a 200 mcg por da
sin efectos txicos Para el manejo de pacientes adultos con hipotiroidismo
severo la dosis oral inicial es de 12 5 25 mcg por da en una sola toma
incrementndose de 25 a 50 mcg por da a intervalos de 2 a 4 semanas hasta
obtener la respuesta ade cuada En pacientes geritricos la dosis inicial es de
12 5 mcg una vez al da incrementndola a intervalos de 3 a 8 semanas hasta
la obtencin de la respuesta deseada La dosis de mantenimiento en pacientes
adultos es de 100 a 200 mcg diarios aunque algunos pacientes pueden
requerir de dosis mayores
RESECCIN
TRANESFENOIDAL.
Los adenomas resistentes al tratamiento mdico o los que producen sntomas
que empeoran cada vez ms se deben someter a neurociruga Siempre que
sea posible se utiliza la va transesfenoidal para llegar hasta la hipfisis En los
pacientes con tumores que no se pueden operar o que son persistentes se
recurre a la radioterapia
ANTIFIMICOS. Tuberculosis extrapulmonar Las formas extrapulmonares de TB representan
entre 10 y 20 % de los casos totales de TB (Sin incluir a los individuos
inmunodeprimidos) El rgimen de 6 meses es tan eficaz como en las formas
pulmonares aunque hay casos como la TB menngea en los que se aconseja
la pauta de 9 meses El uso de terapias coadyuvantes con ciruga o
corticoides es ms frecuente en las formas extrapulmonares se recomienda el
empleo de corticoides en la pericarditis tuberculosa y en la TB menngea Slo
sera opcin correcta si se tratara de tuberculosis
CON
CARBEGOLINA.
Cuando un adenoma de cualquier tamao se acompaa de amenorrea o
galactorrea se debe contemplar la posibilidad de administrar tratamiento En
general el tratamiento de primera lnea para los micro adenomas y macro
adenomas es mdico Las mujeres reciben algn agonista dopaminrgico como
bromocriptina o carbegolina Esta ltima tiene mucho menos efectos colaterales
que la bromocriptina sin embargo en nuestro medio se prefiere esta ltima
porque es mucho ms econmica La cabergolina es un derivado de la ergolina
dopaminrgica dotada de una potente y prolongada actividad para disminuir los
niveles de prolactina Acta por medio de una estimulacin directa de los
receptores D2 para la dopamina que se encuentra en los lacttrofos pituitarios
por lo tanto inhibe la secrecin de prolactina En resumen la cabergolina ejerce
un efecto central dopaminrgico por medio de la estimulacin de los receptores
D2 a dosis orales ms altas que aqullas efectivas en disminuir los niveles
sricos de prolactina nhibicin/supresin de la lactancia fisiolgica: Para
inhibicin de la lactancia: La dosis recomendada es de 1 mg Para supresin de
la lactancia establecida: La dosis recomendada es de 0 25 mg cada doce horas
por dos das (1 mg de dosis total) Tratamiento de desrdenes
hiperprolactinmicos: La dosis inicial recomendada es de 0 5 mg por semana
administrada en una o dos dosis (Una mitad de una tableta de 0 5 mg) por
semana (Ejemplo lunes y jueves) La dosis semanal deber ser incrementada
gradualmente preferiblemente adicionando 0 5 mg por semana a intervalos
mensuales hasta lograr una respuesta teraputica ptima La dosis teraputica
es usualmente de 1 mg por semana pero con un intervalo de 0 25 a 2 mg por
semana Las dosis de hasta 4 5 mg por semana debern ser utilizadas en
pacientes hiperprolactinmicos La dosis semanal deber ser administrada
como una sola dosis dividida en dos o ms dosis por semana de acuerdo con la
tolerabilidad del paciente

Bibliografa: GINECOLOGIA DE NOVAK . BEREK S JONATHAN. MC GRAW-HILL. INTERAMERICANA. EDICIN 12. 1996. PG. 862, 863.


FIN DEL CASO CLNICO SERIADO

21/05/13 18:26 Simulador Proedumed
Pgina 1 de 1 http://www.proedumed.com.mx/simulador/pages/examen/resultadoPregunta.faces
Anlisis del Caso Clnico
Identificacin del reactivo
Area: GINECOLOGA Y OBSTETRICIA
Especialidad: GINECOLOGA
Tema: LEUCORREA
Subtema: CANDIDIASIS VAGINAL
CASO CLNICO SERIADO

MUJER DE 52 AOS DE EDAD CON FUM DE HACE 7 DAS. ACUDE A CONSULTA POR PRESENTAR PRURITO VULVAR INTENSO QUE SE ACOMPAA DE ARDOR VULVAR
DURANTE LA MICCIN. A LA EXPLORACIN SE ENCUENTRA ERITEMA VULVAR ACENTUADO, HUELLAS DE RASCADO INTENSO Y LESIONES PAPULOSAS PERIFRICAS
BIEN DEFINIDAS, LA SECRECIN ES BLANQUECINA Y GRUMOSA.

ELEMENTOS CLAVE A CONSIDERAR EN EL CASO CLNICO:
Edad: Mujer de 52 aos de edad
Antecedentes: FUM de hace 7 das
Sintomatologa: Prurito vulvar intenso que se acompaa de ardor vulvar durante la
miccin
Exploracin: Eritema vulvar acentuado huellas de rascado intenso y lesiones
papulosas perifricas bien definidas y secrecin blanquecina grumosa
Laboratorio y/o gabinete:

51 - EL AGENTE INFECCIOSO QUE MS PROBABLEMENTE ESTA ASOCIADO AL CUADRO CLNICO DE LA PACIENTE ES:

VIRUS DEL
HERPES
SIMPLE.
El herpes genital es una enfermedad de transmisin sexual (ETS) causada por los
virus del herpes simple tipo 1 (VHS 1) y tipo 2 (VHS 2) La mayora de los herpes
genitales son causados por el tipo VHS 2 La mayora de las personas infectadas
por el VHS 1 VHS 2 no presentan signos ni sntomas de la infeccin o presentan
sntomas mnimos Cuando se manifiestan los signos usualmente lo hacen en
forma de una o ms ampollas en los genitales o el recto o alrededor de los mismos
Las ampollas se rompen formando lceras dolorosas (llagas) que pueden tardar de
dos a cuatro semanas en curarse la primera vez que se presentan Tpicamente
puede presentarse otro brote semanas o meses despus del primero pero casi
siempre es menos intenso y de ms corta duracin A pesar de que la infeccin
puede permanecer en forma indefinida en el organismo la cantidad de brotes tiende
a disminuir a medida que pasan los aos
CHLAMYDIA
TRACHOMATIS.
Cervicitis La cervicitis mucopurulenta es en la mujer el equivalente a la uretritis en
el varn Es muy importante su diagnstico para prevenir complicaciones como la
endometritis y la salpingitis y en la mujer embarazada el parto prematuro la
infeccin puerperal y la iniciacin o promocin de una neoplasia cervical Para
afirmar que existe una CMP el exudado del crvix obtenido con una torunda de
algodn blanco tras una primera limpieza de la mucosidad debe manchar la
torunda de color amarillento o verdoso extendido sobre el portaobjetos visto al
microscopio (x 1 000) debe contener al menos 10 polimorfonucleares por campo en
5 campos no adyacentes observados de forma consecutiva C trachomatis es el
microorganismo aislado con mayor frecuencia en la CMP seguido de N
gonorrhoeae herpes simple y T vaginalis estos dos ltimos producen una
exocervicitis Chlamydia y gonococo infectan el endocrvix El diagnstico etiolgico
se establece mediante cultivos del exudado endocervical que debe obtenerse tras
la limpieza previa del orificio externo del crvix
CANDIDA
ALBICANS.
Los sntomas de CVV incluyen prurito descenso vaginal dolor vaginal dispareunia
y disuria externa El diagnstico de candidiasis se puede hacer mediante inspeccin
visual determinacin del pH vaginal microscopa Papanicolaou prueba de ltex y
cultivo de secrecin cervicovaginal El flujo vaginal de una real infeccin por hongos
puede tener diferentes apariencias Puede estar ausente o muy discreto o muy
fluido blanco con presencia de placas en la pared vaginal tpicamente como
requesn Se debe de sospechar de cndida si la paciente tiene un rash
geogrfico simtrico en la vulva o en el rea perineal Una forma algo atpica de
presentacin de la cndida es aquella paciente que tiene una irritacin inexplicable
y disconfort o aquella sin historia de dispareunia que inicia molestias de quemazn
intra o poscoital irritacin disconfort Este problema suele presentarse en mujeres
peri y posmenopusicas Evidentemente la paciente presenta un cuadro clnico
compatible con este padecimiento
TRICHOMONA
VAGINALIS.
Tricomoniasis T vaginalis constituye una de las enfermedades de transmisin
sexual ms frecuentes en el mundo en la embarazada se ha asociado a parto
prematuro y recin nacido de bajo peso Produce leucorrea profusa espumosa
amarillo verdosa y maloliente con abundantes polimorfonucleares pH alcalino y
prurito vaginal En la exploracin la vagina est inflamada y el crvix enrojecido y
edematoso con aspecto de frambuesa En el varn la infeccin por T vaginalis es
menos frecuente La sintomatologa que produce es de uretritis y con frecuencia la
infeccin es asintomtica El diagnstico por microscopia en fresco es un mtodo
especfico en caso de vaginitis purulenta pero poco sensible para detectar a las
pacientes asintomticas El cultivo es un mtodo especfico y sensible superado
por los mtodos moleculares especialmente por la reaccin en cadena de la
polimerasa (PCR) La citologa especialmente el Papanicolau se considera un
mtodo inadecuado por su baja sensibilidad y pobre valor predictivo positivo

Bibliografa: GINECOLOGIA DE NOVAK . BEREK S. JONATHAN. MC GRAW-HILL. INTERAMERICANA. EDICIN 13. 2004. PG. 374.

http://www.cenetec salud.gob.mx/descargas/gpc/CatalogoMaestro/081 GPC Vaginitisinfec1NA/Vaginitis RR CENETEC.pdf


52 - EL TRATAMIENTO MS INDICADO EN ESTE CASO ES:

AZITROMICINA. Tratamiento de la cervicitis mucopurulenta Est indicada la azitromicina 1 g en
dosis nica o doxiciclina 100 mg cada 12 horas por va oral durante 7 das Como
rgimen alternativo podemos emplear levofloxacino 500 mg al da durante 7 das
Tanto la doxiciclina como el levofloxacino estn contraindicados en la infeccin en
mujeres embarazadas la azitromicina es eficaz y segura Por los riesgos de
infeccin del neonato debe repetirse el cultivo a las tres semanas de finalizado el
tratamiento
METRONIDAZOL. Tratamiento Tricomona Un tratamiento eficaz es el metronidazol que en dosis
nica de 2 g o dosis mltiples de 500 mg cada 12 horas durante 7 das se ha
mostrado eficaz en ms del 90% de las pacientes al igual que los nuevos
compuestos tinidazol y ornidazol Este tratamiento se recomienda tambin en las
mujeres embarazadas en las que no se ha mostrado teratognico
ACICLOVIR. Se desconoce la cura del herpes genital pero la evolucin de los sntomas se
puede modificar si se inicia el tratamiento sistmico con aciclovir o sus anlogos
tan pronto comienzan los sntomas El tratamiento puede reducir la formacin de
nuevas lesiones la duracin del dolor el tiempo necesario hasta la resolucin de
las lesiones y la eliminacin viral Sin embargo no parece modificar la historia
natural de la enfermedad recurrente No se recomienda el tratamiento tpico con
aciclovir ya que slo produce un acortamiento mnimo de la duracin de los
episodios sintomticos nfecciones recurrentes La mayora de los pacientes con
un primer episodio de infeccin por herpes genital presentar episodios
recurrentes de las lesiones genitales El tratamiento antiviral episdico o supresivo
acortar la duracin de las lesiones genitales Muchos pacientes se benefician al
recibir el tratamiento antiviral por lo tanto se deben analizar con todos los
pacientes las opciones de tratamiento antiviral Muchos pacientes con enfermedad
recurrente se benefician al recibir tratamiento episdico si se inicia el tratamiento
durante el perodo prodrmico o dentro del primer da de aparicin de las lesiones
Si se opta por el tratamiento de los episdicos recurrentes se deber suministrar
al paciente el tratamiento antiviral una receta para obtener el medicamento de
manera que se inicie el tratamiento ante el primer signo de lesiones genitales o el
prdromos
MICONAZOL. Todos los azoles tpicos y orales as como la nistatina local tienen una efectividad
alrededor del 80% en el tratamiento de candidiasis vulvovaginal no complicada La
eleccin de azoles ( traconazol ketoconazol fluconazol) * para tratamiento de la
candidiasis vulvovaginal no complicada depender de su disponibilidad y costo
Tratamiento tpico 1) Miconazol** crema 2% una aplicacin (5 gramos) en vulva
y vagina al da durante 7 das 2) Nistatina vulos o tabletas vaginales de 100
000 U una aplicacin vaginal al da durante 14 das Tratamiento oral Fluconazol
capsulas 250 mg en una dosis nica traconazol cpsulas 200 mg cada 12 horas
por 1 da * Contraindicados en el embarazo y lactancia ** Los azoles tpicos
pueden causar irritacin vulvovaginal misma que debe considerarse si persisten
los sntomas ** Daa los condones y diafragmas de ltex El tratamiento de la
pareja masculina asintomtica de pacientes con VC no disminuye la frecuencia
de recurrencia de la VC: No dar tratamiento a la(s) pareja(s) masculina(s) si
sta(s) se encuentra(n) asintomtica(s) Las mujeres embarazadas asintomticas
con VC no requieren tratamiento Use slo tratamientos locales por 14 das en
caso de VC sintomtica durante el embarazo Vaginitis recurrente por Candida sp
Tratamiento de eleccin para VC recurrente: nduccin: traconazol oral 200mg
cada 12 horas por un da (dosis nica) Miconazol crema 2% 1 aplicacin
intravaginal diaria por 14 das Mantenimiento: Ketoconazol* tabletas de 200mg
media tableta al da por 6 meses traconazol* oral 50 a 100 mg diario por 6
meses Fluconazol* cpsulas de 100 mg una vez a la semana por 6 meses *No
se use en embarazo o lactancia

Bibliografa: GINECOLOGIA DE NOVAK . BEREK S. JONATHAN. MC GRAW-HILL. INTERAMERICANA. EDICIN 13. 2004. PG. 374.

http://www.cenetec salud.gob.mx/descargas/gpc/CatalogoMaestro/081 GPC Vaginitisinfec1NA/Vaginitis RR CENETEC.pdf

FIN DEL CASO CLNICO SERIADO

21/05/13 18:26 Simulador Proedumed
Pgina 1 de 1 http://www.proedumed.com.mx/simulador/pages/examen/resultadoPregunta.faces
Anlisis del Caso Clnico
Identificacin del reactivo
Area: GINECOLOGA Y OBSTETRICIA
Especialidad: GINECOLOGA
Tema: LEUCORREA
Subtema: TRICOMONIASIS VAGINAL
CASO CLNICO CON UNA PREGUNTA

MUJER DE 25 AOS DE EDAD, CON VIDA SEXUAL ACTIVA DESDE HACE 5 AOS. ACUDE A CONSULTA POR REFERIR FLUJO POSTCOITO, FTIDO, AMARILLO VERDOSO Y
PRURIGINOSO.

ELEMENTOS CLAVE A CONSIDERAR EN EL CASO CLNICO:
Edad: Mujer de 25 aos de edad.
Antecedentes: Vida sexual activa.
Sintomatologa: Flujo postcoito, con olor ftido, pruriginoso.
Exploracin: Se observa flujo amarillento-verdoso y fetidez.
Laboratorio y/o gabinete: Medicin del pH, la prueba de las aminas y frotis para examen directo al
microscopio.

53 - EL SIGUIENTE ESTUDIO ES EL MS TIL PARA ESTABLECER EL DIAGNSTICO DE CERTEZA:

CULTIVO DE
EXUDADO.
Los cultivos vaginales se reservan para casos especiales de sospecha de trichomona
vaginalis en los siguientes casos: La paciente solicita un diagnstico preciso, existe
un riesgo; alto para presencia de enfermedad de transmisin sexual, hay sntomas de
infeccin de tracto reproductivo alto, no hubo respuesta a un tratamiento previo y si
los sntomas se presentan en las 3 primeras semanas posterior a la insercin de DIU.
Si la pregunta fuera, El diagnstico definitivo?, esta podra ser considerada la
respuesta correcta.
CITOLOGA
EXFOLIATIVA.
La citologa, especialmente el Papanicolau, (Se considera un mtodo inadecuado por
su baja sensibilidad y pobre valor predictivo positivo). La colposcopa es un
procedimiento ginecolgico que consiste en la observacin microscpica del cuello
uterino, las paredes vaginales y la entrada a la vagina. Permite realizar con mayor
exactitud y seguridad el Papanicolau y la toma de biopsias ante la presencia de
lesiones sospechosas en el cuello uterino.
FROTIS EN
FRESCO.
La infeccin por T. vaginalis constituye una de las ETS ms frecuentes en el mundo,
en la embarazada se ha asociado a parto prematuro y recin nacido de bajo peso.
Produce leucorrea profusa, espumosa, amarillo-verdosa y maloliente, con
abundantes polimorfonucleares, pH alcalino y prurito vaginal. En la exploracin, la
vagina est inflamada y el crvix enrojecido y edematoso con aspecto de frambuesa.
En el varn, la infeccin por T. vaginalis es menos frecuente. La sintomatologa que
produce es de uretritis y, con frecuencia, la infeccin es asintomtica. El diagnstico
por microscopia en fresco es un mtodo especfico en caso de vaginitis purulenta,
pero poco sensible para detectar a las pacientes asintomticas. El cultivo es un
mtodo especfico y sensible, superado por los mtodos moleculares, especialmente
por la reaccin en cadena de la polimerasa (PCR). A pesar de que el cultivo y los
mtodos moleculares son ms sensibles y especficos que el frotis en fresco, en el
primer nivel de atencin, se tiene acceso a este estudio prcticamente de manera
exclusiva, por esa razn seguiremos considerando est como la respuesta correcta.
La pregunta habla de utilidad, y lo primero que debes considerar es la disponibilidad
del estudio para que realmente sea til.
PRUEBA DE
HIDRXIDO
DE POTASIO.
Olor a aminas (Pescado), cuando se agrega solucin de hidrxido de potasio a las
secreciones vaginales, utilizado como criterio para vaginosis bacteriana. Vaginosis
Bacteriana. Es la causa ms frecuente de exudado vaginal y de mal olor de vagina.
La VB es una alteracin de la flora vaginal, en la que la flora bacteriana normal,
constituida por bacilos Gram-positivos (Lactobacillus spp.), se halla sustituida por
cocobacilos Gram-negativos (Gardnerella vaginalis) y una flora variada que
comprende diversas especies anaerobias. En su patognesis intervienen
sinrgicamente G. vaginalis y los anaerobios que producen el mal olor. La causa de
esta disbacteriosis es desconocida, y se asocia con la existencia de mltiples parejas
sexuales, duchas vaginales y prdida de Lactobacillus, lo que provoca una elevacin
del pH vaginal. En realidad no est establecido de una forma clara que la VB se
produzca por la adquisicin de un patgeno de transmisin sexual, el tratamiento de
las parejas no resulta eficaz para prevenir las recidivas. Tratamiento. Se recomienda
en la mujer embarazada, pues reduce el riesgo de sufrir complicaciones como parto
prematuro y endometritis puerperal. En la no embarazada tambin se recomienda,
pues elimina los sntomas y signos de VB y reduce las complicaciones en caso de
aborto o histerectoma. El tratamiento. Se realiza con metronidazol en dosis de 500
mg cada 12 horas por va oral, durante 7 das. Existen tratamientos alternativos por
va vaginal como clindamicina al 2 % en crema vaginal metronidazol al 0,75% en
gel, ambos se administran con un aplacador, antes de acostarse, durante 7 das. No
se recomienda el tratamiento de la pareja.

Bibliografa: TRATADO DE GINECOLOGA DE NOVAK. JONES III, HOWARD W. INTERAMERICANA, MC. GRAW HILL 11A ED. 1991. PGS. 507-509



21/05/13 18:26 Simulador Proedumed
Pgina 1 de 1 http://www.proedumed.com.mx/simulador/pages/examen/resultadoPregunta.faces
Anlisis del Caso Clnico
Identificacin del reactivo
Area: GINECOLOGA Y OBSTETRICIA
Especialidad: GINECOLOGA
Tema: CLIMATERIO Y MENOPAUSIA
Subtema: CLIMATERIO Y MENOPAUSIA
CASO CLNICO CON UNA PREGUNTA

MUJER DE 43 AOS DE EDAD, CON ANTECEDENTE DE HISTERECTOMA CON OOFORECTOMA POR MIOMATOSIS UTERINA. ACUDE POR PRESENTAR SNTOMAS
VASOMOTORES INTENSOS Y ATROFIA UROGENITAL. USTED DECIDE INDICAR TERAPIA HORMONAL.

ELEMENTOS CLAVE A CONSIDERAR EN EL CASO CLNICO:
Edad: mujer adulta joven
Antecedentes: elemento clave del caso cl nico histerectom a con ooforectom a
menopausia quirrgica
Sintomatologa: s ntomas propios de perimenopusia
Exploracin:
Laboratorio y/o gabinete:

54 - EN ESTA PACIENTE EL TRATAMIENTO INDICADO SER CON:

TRATAMIENTO
COMBINADO
CONTNUO
En las mujeres con tero los progestgenos se combinan con estrgenos prara
reducir el riesgo de cncer endometrial De hecho se puede prescribir
diariamente progestgenos con los estrgenos lo que se denomina terapia
cont nua Sin embargo este mtodo genera amenorrea Se indica en pacientes
con tero
TRATAMIENTO
CCLICO
COMBINADO
Otra opcin es administrar estrgenos durante 25 d as de cada mes y algn
progestgeno durante los ltimos 10 d as Los medicamentos se retiran durante
cinco d as despus de los cules se produce descamacin y hemorragia
endometrial Este tratamiento c clico es el que ms se utiliza durante la
transicin menopusica mientras que el tratamiento continuo se prefiere en las
mujeres despus de la menopausia
TERAPIA
ESTROGNICA
SIMPLE
La clave en este caso es que se trata de una paciente con menopausia
quirrgica es decir secundaria a la histerctom a y ooforectom a Dado que la
paciente ya no puede menstruar esta es una de las pocas indicaciones de la
terapia estrognica simple Los estrgenos se pueden administrar por v a oral
parenteral tpica o transdrmica con efectos similares En estos pacientes se
recomienda el tratamiento continuo con estrgenos si bien es posible cambiar
la dosis y v a de administracin segn la preferencia de cada paciente En
Estados Unidos los ms comunes son los utilizados por v a oral Los parches
transdrmicos omiten el efecto de primer paso en el h gado y ofrecen la
conveniencia de una administracin menos frecuente (una o dos veces por
semana)
PROGESTGENOS. Los progestagenos slos no estn indicados para el tratamiento de las
manifestaciones genitourinarias de la menopausia Slos son eficaces para el
tratamiento de los bochornos en las mujeres en las que existe alguna
contraindicacin para administrar estrgenos como sucede cuando existen
antecedentes de tromboembolias venosas o cncer mamario Sin embargo su
administracin se ve limitada por alguno de sus efectos adversos como
hemorragia vaginal y aumento de peso

Bibliografa: GPC. ATENCIN AL CLIMATERIO Y MENOPAUSIA.

http://www.cenetec salud.gob.mx/descargas/gpc/CatalogoMaestro/019 GPC ClimatyMenop/SS 019 08 EyR.pdf


21/05/13 18:27 Simulador Proedumed
Pgina 1 de 1 http://www.proedumed.com.mx/simulador/pages/examen/resultadoPregunta.faces
Anlisis del Caso Clnico
Identificacin del reactivo
Area: GINECOLOGA Y OBSTETRICIA
Especialidad: OBSTETRICIA
Tema: ENFERMEDAD HIPERTENSIVA DEL EMBARAZO
Subtema: HIPERTENSIN ARTERIAL DURANTE EL EMBARAZO
CASO CLNICO SERIADO

FEMENINO DE 36 AOS DE EDAD, GESTA 2 PARA 1, PARTO PREVIO HACE 10 AOS. ACTUALMENTE CON EMBARAZO DE 28 SDG. ACUDE A CUARTA CONSULTA PRENATAL,
ENCONTRANDOLA A LA EXPLORACIN CON TENSIN ARTERIAL DE 140/90, FRECUENCIA CARDIACA 90X, FRECUENCIA RESPIRATORIA 22X, CONCIENTE Y ORIENTADA.
FONDO UTERINO A 27 CENTMETROS POR ARRIBA DE LA SNFISIS DEL PUBIS. SE SOLICITA EXAMEN GENERAL DE ORINA QUE SE REPORTA SIN PROTEINURIA.

ELEMENTOS CLAVE A CONSIDERAR EN EL CASO CLNICO:
Edad: 36 aos
Antecedentes: parto previo hace 10 aos embarazo 28SDG
Sintomatologa:
Exploracin: Ta 140 90
Laboratorio y/o gabinete: examen de orina sin proteinuria

55 - EL CUADRO CLNICO DE LA PACIENTE ES DESENCADENADO POR:

HEMOCONCENTRACIN Como resultado del vasoespasmo arteriolar presente en la preeclampsia
se disminuye el volumen plasmtico circulante y da como resultado una
hemoncentracin secundaria LA HEMOCONCENTRAC N ES
CARACTERST CA DE LA PREECLAMPS A PERO DEBES TOMAR EN
CUENTA QUE STA PARTE DE LA D SFUNC N ENDOTEL AL
PR MAR A
ALTERACIN
HEMOSTTICA
Diferentes estudios han demostrado que en la preeclampsia ocurre una
alteracin en la activacin de la coagulacin STE PROCESO NO EST
PRESENTE EN LA H PERTENS N GESTAC ONAL Y SE DESCARTA
DEB DO A QUE LA PAC ENTE NO CUMPLE CON CR TER OS PARA
PREECLAMPS A
VASO ESPASMO
GENERALIZADO
El vasoespasmo generalizado presente en la preeclampsia es secundario
a la disfuncin endotelial por lo que ste no es una etiologa sino
respuesta a ste dao
DISFUNCIN
ENDOTELIAL
La preeclampsia eclampsia constituyen un sndrome que se caracteriza
por: hipoperfusin tisular generalizada relacionada con una respuesta
vascular anormal placentaria que se asocia con un aumento en las
resistencias vasculares sistmicas dao endotelial cambios metablicos
consumo plaquetario aumento en la respuesta inflamatoria y activacin
del sistema de coagulacin Hay distintas teoras en cuanto a la etiologa
de ste sndrome sin embargo la ms aceptada es la lesin endotelial de
la cual se desprenden el resto de ellas

Bibliografa:OBSTETRICIA Y MEDICINA PERINATAL. SAMUEL KORCHMERK. COLEGIO MEXICANO DE ESPECIALISTAS EN GINECOLOGA Y OBSTETRICIA.
EDICIN 1A. 2006. PAG. 213-224.



56 - EL DIAGNSTICO CLNICO MS PROBABLE ES:

PREECLAMPSIA
LEVE
La preeclampsia leve se presenta despus de la semana 20 de gestacin durante
el parto o en las primeras 6 semanas despus de ste se caracteriza por presin
sistlica mayor o igual a 140 mm Hg o presin diastlica mayor o igual a 90 mm Hg
y proteinuria mayor o igual a 300 mg / orina de 24 hrs o su equivalente en tira
reactiva No cumple con el criterio de proteinuria CUMPLE CON C FRAS
TENS ONALES PERO NO HAY PROTE NUR A
HIPERTENSIN
GESTACIONAL
La hipertensin gestacional se caracteriza por la presencia de hipertensin arterial
mayor o igual de 140/90 mm Hg despus de la semana 20 de gestacin y se
diferencia de la preeclamsia por la ausencia de proteinuria RECUERDA QUE EL
D AGNST CO D FERENC AL ENTRE PREECLAMPS A LEVE E
H PERTENS N GESTAC ONAL SE HACE POR LA PRESENC A O AUSENC A
DE PROTE NUAR A
ECLAMPSIA
INMINENTE
La eclampsia se diagnostica cuando adems de los signos de preeclampsia se
agregan convulsiones EL CASO CLN CO NO DESCR BE LA PRESENC A DE
CONVULS ONES
PREECLAMPSIA
SEVERA
La preeclamsia severa se presenta despus de la semana 20 de gestacin
durante el parto o en las primeras 6 semanas despus de ste se caracteriza
entre otros aspectos por presin sistlica mayor o igual a a 160 mm Hg o presin
diastlica mayor o igual a 110 mm Hg y proteinuria mayor o igual a a 2 gr en orina
de 24 horas o su equivalente en tira reactiva No cumple con el criterio de
proteinuria NO CUMPLE CON EL CR TER O DE C FRAS TENS ONALES Y NO
HAY PRESENC A DE PROTE NUR A

Bibliografa:WILLIAMS OBSTETRICIA. F. GARY CUNNINGHAM. PANAMERICANA. EDICIN 21A. 2002. PAG. 489-525.



57 - LO INDICADO EN ESTE CASO ES:

INTERRUPIR EL
EMBARAZO
De acuerdo a las guas clnicas para el manejo de la preeclampsia siempre que
se diagnostica preeclampsia severa el embarazo despus de la semana 34 el
embarazo debe interrumpirse Si ste se encuentra antes de la semana 34 deben
darse inductores de maduracon pulmonar previo a la interrupcin del embarazo
NO CUMPLE CON CR TER OS PARA PREECLAMPS A SEVERA
ADMINISTRAR
SULFATO DE
MAGENESIO
El sulfato de magnesio es el medicamento de eleccin para la prevencin y
tratamiento de las crisis convulsivas en la eclampsia ESTE TRATAM ENTO NO
APL CA PARA EL D AGNST CO DE LA PAC ENTE
ADMINISTRAR
ALFAMETILDOPA
La administracin de antihipertensivos est indicada en todos los tipos de
hipertensin durante el embarazo el manejo de eleccin primario debe realizarse
con alfa metil dopa y debe retirarse una vez que la TA se encuentre menor o igual
a 130/80 EL MANEJO CON ANT H PERTENS VOS ES CORRECTO S N
EMBARGO STE DEBE ADM N STRARSE EN CUALQU ERA DE LOS
SEGU M ENTOS Y T POS DE H PERTENS N LO CUAL LO HACE
NESPECF CO COMO RESPUESTA
VIGILANCIA
ESTRECHA DEL
EMBARAZO
El manejo de la paciente con hipertensin gestacional siempre que el binomio se
encuentre estable debe ser ambulatorio por la consulta externa EN EL CASO DE
LA H PERTENS N GESTAC ONAL EL MANEJO ES CONSERVADOR CON LA
ADM NSTRAC N DE ANT H PERTENS VOS

Bibliografa:WILLIAMS OBSTETRICIA. F. GARY CUNNINGHAM. PANAMERICANA. EDICIN 21A. 2002. PAG. 213-224.


FIN DEL CASO CLNICO SERIADO

21/05/13 18:28 Simulador Proedumed
Pgina 1 de 1 http://www.proedumed.com.mx/simulador/pages/examen/resultadoPregunta.faces
Anlisis del Caso Clnico
Identificacin del reactivo
Area: GINECOLOGA Y OBSTETRICIA
Especialidad: GINECOLOGA
Tema: ANTICONCEPCIN Y PLANIFICACIN FAMILIAR
Subtema: MTODOS DE PLANIFICACIN FAMILIAR, TEMPORALES Y
DEFINITIVOS
CASO CLNICO SERIADO

MUJER DE 35 AOS DE EDAD QUE HA INTENTADO EMBARAZARSE DESDE HACE 2 AOS. CICLOS MENSTRUALES NORMALES. PRESENTA DESDE HACE 12 MESES DOLOR
ARTICULAR Y TRES EPISODIOS DE TROMBOSIS VENOSA EN MIEMBROS PLVICOS CON REMISIN FAVORABLE. PAREJA DE 40 AOS DE EDAD CUYO SEMINOGRAMA
REPORTA MOVILIDAD TIPO "A" SEGN LA ORGANIZACIN MUNDIAL DE LA SALUD.

ELEMENTOS CLAVE A CONSIDERAR EN EL CASO CLNICO:
Edad: 35 AOS
Antecedentes: NCAPAC DAD PARA EMBARAZARSE EN 2 AOS 3 EP SOD OS DE
TROMBOS S VENOSA
Sintomatologa: DOLOR ART CULAR DE 12 MESES DE EVOLUC ON
Exploracin:
Laboratorio y/o gabinete:

58 - EL DIAGNSTICO CLNICO MS PROBABLE ES:

INFERTILIDAD
SECUNDARIA A
ARTRITIS
REUMATOIDE
La Artritis Reumatoide (AR) es una enfermedad inflamatoria crnica
multisistmica predominantemente articular que afecta principalmente a
pequeas articulaciones de las manos y de los pies de forma simtrica
Presenta un curso variable El embarazo altera el estado inmune y esto afecta el
curso de la AR Se ha observado que durante el embarazo disminuyen los
efectos de la AR pero se incrementan despus del parto Parece no afectar la
fertilidad sin embargo los pacientes afectados de artritis reumatoide tienen un
deseo mayor de controlarla NO HAY DATOS CLAROS DE NFECUND DAD EN
PAC ENTES CON ARTR T S REUMATO DE
ESTERILIDAD
SECUNDARIA A
LUPUS
ERITEMATOSO
SISTMICO
El lupus eritematoso sistmico es una enfermedad autoinmune y sistmica que
se presenta con frecuencia en mujeres jvenes y por tanto en su etapa
reproductiva; se asocia a un alto riesgo de morbilidad y mortalidad perinatal Las
complicaciones ms frecuentes son los abortos la muerte fetal la prematurez el
retardo del crecimiento intrauterino y el lupus neonatal HAB TUALMENTE NO
COMPROMETE LA FECUND DAD PERO S SE ASOC A A UN ALTO R ESGO
DE MORB L DAD Y MORTAL DAD PER NATALl
SNDROME
ANTIFOSFOLPIDO
PRIMARIO
El sndrome antifosfolipdico una enfermedad autoinmunitaria de
manifestaciones clnicas diversas (aborto recurrente preeclampsia parto
pretrmino trombosis vascular retraso en el crecimiento intrauterino bitos
desprendimiento de la placenta) Es un padecimiento autoinmunitario distinguido
por trombosis vascular trombocitopenia reaccin serolgica falsa positiva para
sfilis y prdidas fetales recurrentes que aparece en las pacientes (hallazgos
corroborados por laboratorio) con ttulos moderados o altos de anticuerpos
antifosfolipdicos Criterios clnicos mayores: aborto recurrente muerte fetal en el
segundo o tercer trimestre de la gestacin; trombosis venosa arterial o
trombocitopenia Criterios clnicos menores: reaccin serolgica falsa positiva
para sfilis prueba de Coombs positiva anormalidades valvulares cardiacas
livedo reticularis migraa lceras en las piernas mielopata corea hipertensin
pulmonar o necrosis avascular LA NCAPAC DAD PARA EMBARAZARSE Y LA
PRESENC A DE TROMBOS S FUNDAMENTAN EL D AGNST CO
ESTERILIDAD
PRIMARIA
Esterilidad primaria: pareja que tras un ao de coitos no protegidos no ha
conseguido un embarazo (para la Federacin nternacional de Ginecologa y
Obstetricia el periodo es de dos aos) CUMPLE CON LA DEF N C N PERO
EN ELLA NO SE ESTABLECEN LAS CAUSAS

Bibliografa:GINECOLOGA Y OBSTETRICIA. JOHNS HOPKINS. BRANDON J. BANKOWSKI. MARBN LIBROS, S.L. MADRID ESPAA. EDICIN 1RA. 2005. PAG.
377-384.



59 - PARA CONFIRMAR EL DIAGNSTICO DEBER INVESTIGAR:

ANOMALAS
UTERINAS Y DE LA
OVULACIN
La presencia de anomalias uterinas constituye un factor femenino de la
infertilidad y las alteraciones en la ovulacin un factor endocrinolgico
Ninguna de ellas se asocia de manera directa con el Sindrome antifosfolpidos
BSQUEDA DE
ANTICUERPOS
ANTIFOSFOLIPIDOS
Y SEMINOGRAMA
El diagnstico del sndrome de antifosfolpidos consiste en la determinacin de
inmunoglobulinas ( gG gM e gA) dirigidas contra algunos fosfolpidos como
el anticoagulante lpico y el anticardiolipina El estudio inmunolgico segn los
lineamientos de la Sociedad nternacional de Trombosis y Hemostasia debe
ser positivo (al menos en dos ocasiones) para cualquiera de los dos
anticuerpos y con intervalo de seis a ocho semanas de diferencia entre cada
una de las determinaciones
TROMBOCITOSIS Y
OVULACIN
Los criterios clnicos para el diagnstico de Sindrome antofosfolipidos se
dividen en mayores y menores Criterios clnicos mayores: aborto recurrente
muerte fetal en el segundo o tercer trimestre de la gestacin;TROMBOS S
VENOSA arterial o trombocitopenia Criterios clnicos menores: reaccin
serolgica falsa positiva para sfilis prueba de Coombs positiva
anormalidades valvulares cardiacas livedo reticularis migraa lceras en las
piernas mielopata corea hipertensin pulmonar o necrosis avascular LA
TROMBOS S CONST TUYE UN CR TER O MAYOR EL D AGNST CO ES
NDEPEND ENTE DE LA OVULAC N
PRESENCIA DE
ARTRITIS, FACTOR
REUMATOIDE
De acuerdo al Colegio Americano de Reumatologa (ACR American College of
Rheumatology) se establece el diagnstico de Artritis Reumatoidea cuando
estn presentes cuatro de los siete criterios siempre y cuando de los criterios
uno al cuatro estn presentes al menos 6 semanas 1 Rigidez matutina de al
menos una hora de duracin 2 Artritis en 3 o ms reas articulares 3 Artritis
de las articulaciones de la mano 4 Artritis simtrica 5 Ndulos reumatoides
6 Cambios radiolgicos compatibles con AR 7 Factor reumatoide positivo

Bibliografa:GINECOLOGA Y OBSTETRICIA. JOHNS HOPKINS. BRANDON J. BANKOWSKI. MARBN LIBROS, S.L. MADRID ESPAA. EDICIN 1RA. 2005. PAG.
377-384.



60 - EL TRATAMIENTO INICIAL MS ADECUADO CONSISTIRA EN:

REPOSO POSTCOITO
POR 30 MINUTOS Y
CIDO FLICO
Se ha considetado durante mucho tiempo la medida primaria de reposo
postcoido en casos de infertiliad con la finalidad de lograr una mayor
permanencia vaginal del semen posterior al coito
AZATIOPRINA Y
CIDO FOLICO
La azatioprina es un derivado imidazlico de la 6 mercaptopurina (6 MP)
activo por va oral y parenteral con propiedades inmunosupresoras La
azatioprina es utilizada como inmunosupresor en los pacientes
transplantados pero tambin es til en el tratamiento de la artritis
reumatoidea sevara psoriasis artrtica y lupus nefrtico
CIDO
ACETILSALICLICO Y
GLUCOCORTICOIDES
Para la prevencin el tratamiento inicial se basaba en
GLUCOCORT CO DES La combinacin de prednisona (40 a 60 mg/da) y
AC DO ACET LSAL C L CO (80 a 100 mg/da) fue el tratamiento ms
prescrito con el que se logr un margen del 60 al 75% de efectividad para el
sndrome antifosfolipdico pero los efectos secundarios eran frecuentes La
explicacin de administrar cido acetilsaliclico se deba a su efecto inhibidor
de la enzima ciclooxigenasa necesaria para la formacin de tromboxano A2
de efecto procoagulante y vasoconstrictor
AUMENTO DE LA
ACTIVIDAD SEXUAL
Y PREDNISONA
Las causas ms frecuentes que pueden causar alteraciones de la calidad en
el eyaculado y por consecuencia esterilidad en el hombre son: 1 De tipo
general: tabaquismo estrs consumo excesivo de alcohol contaminacin
ambiental 2 De tipo mecnico: impotencia o exceso de actividad sexual
masculina 3 Endocrinas:hper o hipotiroidismo hipopituitarismo sndromes
androgenitales y diabetes severa 4 Enfermedades o infecciones: varicocele
dermatitis y ciertas enfermedades de transmisin sexual EL AUMENTO DE
LA ACT V DAD SEXUAL DETER ORA LA CAL DAD DE LA EYACULAC N

Bibliografa:GINECOLOGA Y OBSTETRICIA. JOHNS HOPKINS. BRANDON J. BANKOWSKI. MARBN LIBROS, S.L. MADRID ESPAA. EDICIN 1RA. 2005. PAG.
110-113.


FIN DEL CASO CLNICO SERIADO

21/05/13 18:28 Simulador Proedumed
Pgina 1 de 1 http://www.proedumed.com.mx/simulador/pages/examen/resultadoPregunta.faces
Anlisis del Caso Clnico
Identificacin del reactivo
Area: GINECOLOGA Y OBSTETRICIA
Especialidad: GINECOLOGA
Tema: LEUCORREA
Subtema: VAGINOSIS BACTERIANA
CASO CLNICO CON UNA PREGUNTA

MUJER DE 34 AOS DE EDAD, QUE ES ENVIADA A SU CONSULTA CON CERVICOVAGINITIS, CARACTERIZADA POR SECRECIN TRANSVAGINAL AMARILLENTA,
ESPUMOSA, FTIDA Y ACOMPAADA DE INTENSO PRURITO.

ELEMENTOS CLAVE A CONSIDERAR EN EL CASO CLNICO:
Edad: Mujer de 34 aos de edad
Antecedentes:
Sintomatologa: ntenso prurito
Exploracin: Secrecin transvaginal amarillenta y espumosa
Laboratorio y/o gabinete:

61 - EL TRATAMIENTO INDICADO EN ESTE CASO ES CON:

BETALACTMICOS. Los betalactamicos como por ejemplo las penicilinas cefalosporinas
carbapenems monobactams e inhibidores de betalactamasa Estn indicados
en profilaxis y tratamiento de infecciones causadas por bacterias Gram
positivas y negativas En ginecolog a su uso se limitar a para el tratamiento de
enfermedad plvica inflamatoria cuando la etiolog a identificada fuera
Neisseria
MACROLIDOS. Macrolidos como por ejemplo claritromicina azitromicina derivados de la
eritromicina La azitromicina se utiliza de primera eleccin en tratamiento de
infeccin por Chlamydia trachomatis
QUINOLONAS. No se conoce a fondo el mecanismo de accin por el cual las nuevas
quinolonas destruyen las bacterias rpidamente donde estas bloquean la
accin del cido nalidixico Pero se ha reportado en la literatura mas reciente
su posible mecanismo de accin El blanco bioqu mico de las quinolonas es la
girasa ADN una enzima bacteriana con 4 subunidades que intervienen en
diversas reacciones de ADN de suma importancia dicha enzima bacteriana
produce giros superhelicoidales negativos de ADN de doble cordn e introduce
roturas en el ADN En ginecolog a se utilizan como rgimen alternativo para el
tratamiento de la Chlamydia trachomatis y en la enfermedad plvica
inflamatoria
IMIDAZOLES. El imidazol es un intermediario de la bios ntesis de la histidina que se forma
desde el imidazol glicerol fosfato con la perdida de agua Su molcula ha
servido de base para el desarrollo de numerosos frmacos Es un ejemplo de
medicamento antibacteriano y antiprotozoario el metronidazol siendo de
primera eleccin en el tratamiento de las pacientes con cervicovaginitis
bacteriana o por Trichomona

Bibliografa: GINECOLOGIA DE NOVAK. BEREK S. JONATHAN. MC GRAW-HILL. INTERAMERICANA. EDICIN 12. 1996. PG. 434.



21/05/13 18:29 Simulador Proedumed
Pgina 1 de 1 http://www.proedumed.com.mx/simulador/pages/examen/resultadoPregunta.faces
Anlisis del Caso Clnico
Identificacin del reactivo
Area: GINECOLOGA Y OBSTETRICIA
Especialidad: OBSTETRICIA
Tema: COMPLICACIONES DEL EMBARAZO Y DEL PARTO
Subtema: RUPTURA PREMATURA DE MEMBRANAS
CASO CLNICO CON UNA PREGUNTA

EN EL SERVICIO DE URGENCIAS ES VALORADA MUJER DE 32 AOS DE EDAD CON EMBARAZO DE 34 SEMANAS DE GESTACIN QUE PRESENTA SALIDA DE LIQUIDO
CLARO TRANSVAGINAL. SE TOMA MUESTRA DEL LQUIDO DE LA VAGINA Y LA CRISTALOGRAFA ES NEGATIVA.

ELEMENTOS CLAVE A CONSIDERAR EN EL CASO CLNICO:
Edad:
Antecedentes:
Sintomatologa: SAL DA DE L QU DO CLARO TRASVAG NAL
Exploracin:
Laboratorio y/o gabinete: CR STALOGRAF A NEGAT VA

62 - PARA DESCARTAR O CONFIRMAR RUPTURA DE MEMBRANAS SE DEBERA REALIZAR:

PRUEBA CON
PAPEL DE
NITRAZINA
La RUPTURA PREMATURA DE MEMBRANAS es la prdida de continuidad de
las membranas corio amniticas antes del inicio del parto independientemente
que se produzca antes del trmino a trmino o despus del trmino del
embarazo El papel nitrazina es una tira de papel absorbente que cambia de color
segn se exponga a soluciones de alcalinidad o acidez variable con ello se
determina el pH Por lo general el pH de las secreciones vaginales var a entre 4 5
y 5 5 en tanto que en el l quido amnitico es casi siempre de 7 0 a 7 5 Una
prueba positiva falsa puede deberse a la presencia de sangre semen o alguna
vaginosis bacteriana mientras que los resultados negativos falsos pueden ser
secundarios a una muestra escasa de l quido LA PRUEBA DE LA N TRAZ NA
MUESTRA MAYOR SENS B L DAD Y ESPEC F C DAD QUE EL RESTO DE LAS
PRUEBAS MENC ONADAS POR LO QUE SER A EL ESTUD O A ELEG R TRAS
UNA CR STALOGRAF A NEGAT VA
PRUEBA DE LA
FLAMA
La prueba de la flama o mtodo de lanneta se basa en el calentamiento de una
muestra del l quido obtenida del fondo del saco vaginal la cual se coloca en una
laminilla y se calienta a fuego directo del lado contrario de la laminilla Si es
positiva aparcera una placa blanquesina por desnaturalizacin de prote nas
Cuando es negativa la coloracin es cafesosa o parduzca COMPARTE LAS
M SMAS CAUSAS DE FALSOS NEGAT VOS CON LA CR STALOGRAF A POR
LO QUE ES PROBABLE QUE SE PRESENTEN DADO QUE TENEMOS UNA
CR STALOGRAF A PREV A NEGAT VA
AMNIOCENTESIS La amniocentesis es el procedimiento diagnstico que consiste en la obtencin de
l quido amnitico por puncin transabdominal En el caso de ruptura de
membranas se realiza de forma inversa inyectando solucin diluida de azul de
Evans o de ndigo carm n en el saco gestacional de forma transabdominal tras
sto se esperan 15 a 30min y en caso de haber ruptura se observa el colorante
en la vagina AL SER UN MTODO NVAS VO DEBE UT L ZARSE COMO
LT MO RECURSO ANTE UNA ALTA SOSPECHA DE RUPTURA DE
MEMBRANAS CON PRUEBAS NO NVAS VAS NEGAT VAS
ULTRASONIDO El ultrasonido es un auxiliar en el estudio de esta pacientes y puede ayudar a
confirmar el diagnstico de ruptura prematura de membranas ante la presencia
de oligohidramnios RECUERDA QUE EL OL GOH DRAMN OS PUEDE
DEBERSE A OTRAS CAUSAS como anomal as fetales de las v as urinarias o
restriccin significativa del crecimiento intrauterino LA OBSERVAC N DE
L QU DO AMN T CO NORMAL NO DESCARTA EL D AGNST CO DE
RUPTURA PREMATURA DE MEMBRANAS

Bibliografa: OBSTETRICIA Y MEDICINA PERINATAL. DR. SAMUEL KARCHMER. COMEGO. EDICIN 1RA. 2006. PAG. 173.



21/05/13 18:29 Simulador Proedumed
Pgina 1 de 1 http://www.proedumed.com.mx/simulador/pages/examen/resultadoPregunta.faces
Anlisis del Caso Clnico
Identificacin del reactivo
Area: GINECOLOGA Y OBSTETRICIA
Especialidad: GINECOLOGA
Tema: DOLOR PELVICO
Subtema: ENDOMETRIOSIS
CASO CLNICO SERIADO

MUJER DE 32 AOS DE EDAD, CASADA DESDE HACE 3 AOS, SIN MTODO DE PLANIFICACIN FAMILIAR, INTENTNDOSE EMBARAZAR DESDE HACE 2 AOS. REFIERE
HIPERMENORREA Y DISPAREUNIA. EN LA EXPLORACIN SE ENCUENTRA DOLOR A LA MOVILIZACIN DEL CUELLO UTERINO. EL ULTRASONIDO REPORTA TERO DE
10.5X8.5X6 CM., ADEMS DE AMBOS OVARIOS CON AUMENTO DISCRETO DE VOLUMEN.

ELEMENTOS CLAVE A CONSIDERAR EN EL CASO CLNICO:
Edad: Mujer de 32 aos de edad.
Antecedentes: Casada, sin mtodo de planificacin familiar. Intentando embarazo desde
hace 2 aos.
Sintomatologa: Hipermenorrea, dispareunia.
Exploracin: Dolor a la movilizacin del cuello uterino.
Laboratorio y/o gabinete: USG reporta tero de 10.5x8.5x6 cm., ovarios con aumento discreto de
volumen.

63 - EL PROCEDIMIENTO DE ELECCIN PARA CONFIRMAR EL DIAGNSTICO Y QUE SE DEBER INDICAR EN LA PACIENTE ES:

UN ULTRASONIDO
TRANSVAGINAL.
La paciente ya tiene ultrasonido, donde reportan tero de
10.5x8.5x6cm y discreto aumento del tamao de los ovarios. El
ultrasonido es una ayuda diagnostica, pero no confirma el diagnostico
de endometriosis.
UNA LAPAROSCOPA. Es una tcnica que nos permite observar la cavidad abdominal y
plvica. Permite intervenciones quirrgicas, por lo que tambin se
considera un sistema de ciruga de mnima invasin. En ginecologa
es til, porque a travs de ste procedimiento se confirma el
diagnstico de endometriosis. Es la tcnica ms til que disponemos
para el diagnstico de la endometriosis. Permite, por una parte,
realizar el diagnstico de las lesiones (No slo de la extensin de las
mismas, sino el diagnstico histolgico por la toma de biopsias) as
como la realizacin de maniobras teraputicas (Adhesilisis,
electrocoagulacin de focos incluso la aspiracin y extirpacin de la
cpsula de endometriomas).
UNA
HISTEROSALPINGOGRAFA.
Es un examen de Rayos X de las trompas de Falopio y tero. Se
utiliza fluoroscopia y un medio de contraste. Es til en protocolo de
infertilidad para determinar presencia de malformaciones mullerianas,
tumores intrauterinos as como para valorar la permeabilidad de las
trompas de Falopio. Sirve especficamente para valorar la anatoma
de los genitales internos.
UNA TOMOGRAFA AXIAL
COMPUTARIZADA.
No es de primera eleccin en el protocolo de estudio, slo lo
realizamos en caso de presentar tumores plvicos.

Bibliografa: OBTETRICIA Y GINECOLOGA. JAIME BOTERO U. CIB. EDICIN 7. 2004. PG. 471.

http://www.cenetec.salud.gob.mx/descargas/gpc/CatalogoMaestro/207 SSA 10 ENDOMETRIOSIS/GRR SSA 207 09.pdf


64 - EL DIAGNSTICO CLNICO MS PROBABLE ES:

ENDOMETRIOSIS. La endometriosis es; presencia de tejido endometrial fuera del tero, ocasiona
dismenorrea incapacitante, adherencias, dispareunia, e infertilidad por lo que se
integra el diagnstico en la paciente.
ENFERMEDAD
PLVICA
INFLAMATORIA.
La enfermedad plvica inflamatoria (EPI), es una enfermedad infecciosa que
afecta a la parte alta del aparato reproductor femenino interno (tero, ovarios y
trompas de Falopio). Aparece por la ascensin de bacterias patgenas desde la
vagina y el cuello uterino. Produce infertilidad; pero no dismenorrea
incapacitante, ni hipermenorrea por lo que se descarta esta patologa en la
paciente.
MIOMATOSIS
UTERINA.
Los leiomiomas uterinos (mioma, miofibroma, fibromioma, leiomiofibroma,
fibroliomioma, fibroma y fibroide), son tumores benignos conformados por
msculo liso del miometrio con cantidades variables de tejido conectivo fibroso.
Ocasiona hipermenorrea, pero no dispareunia ni infertilidad.
OVARIO
POLIQUSTICO.
El sndrome de ovario poliqustico (SOPQ), tambin llamado Sndrome de Stein-
Leventhal, es un trastorno endocrino causando uno de los desbalances
hormonales ms frecuentes en mujeres. Para que se considere a una persona
con posible SOP, debe cumplir dos de estos tres criterios: 1. Oligoovulacin o
anovulacin, 2. Exceso de actividad andrognica, 3. Ovarios poliqusticos
(Visualizados por ultrasonido ginecolgico). Debe excluirse: Hiperplasia
suprarrenal congnita, tumores que puedan secretar andrgenos y la
hiperprolactinemia. La paciente no tienes clnica de esta patologa por tal motivo
se descarta.

Bibliografa: OBTETRICIA Y GINECOLOGA. JAIME BOTERO U. CIB. EDICIN 7. 2004. PG. 471.

http://www.cenetec.salud.gob.mx/descargas/gpc/CatalogoMaestro/207 SSA 10 ENDOMETRIOSIS/GRR SSA 207 09.pdf

FIN DEL CASO CLNICO SERIADO

21/05/13 18:30 Simulador Proedumed
Pgina 1 de 1 http://www.proedumed.com.mx/simulador/pages/examen/resultadoPregunta.faces
Anlisis del Caso Clnico
Identificacin del reactivo
Area: GINECOLOGA Y OBSTETRICIA
Especialidad: OBSTETRICIA
Tema: ENFERMEDADES QUE COMPLICAN EL EMBARAZO
Subtema: ENFERMEDADES VIRALES DURANTE EL EMBARAZO
CASO CLNICO CON UNA PREGUNTA

A 29-YEAR-OLD WOMAN, GRAVIDA 1, PARA 1, SPONTANEOUSLY DELIVERS A 2460-G (5 LB 7OZ) FEMALE NEWBORN AT 38 WEEKS GESTATION. THE NEWBORN HAS
HEPATOSPLENOMEGALY, PATENT DUCTUS ARTERIOSUS, AND CATARACTS. AT 8 WEEKS GESTATION, THE MOTHER DEVELOPED A MACULOPAPULAR RASH, ENLARGED
CERVICAL LYMPH NODES, SORE THROAT, AND ARTHRALGIAS THAT SPONTANEOUSLY RESOLVED IN 1 WEEK. THE SUBSEQUENT PRENATAL COURSE WAS
UNCOMPLICATED.

ELEMENTOS CLAVE A CONSIDERAR EN EL CASO CLNICO:
Edad: Una mujer de 29 aos
Antecedentes:
Sintomatologa:
Exploracin:
Laboratorio y/o gabinete:

65 - WHICH OF THE FOLLOWING TESTS DURING PREGNANCY IS MOST LIKELY TO HAVE PREDICTED THE FINDINGS IN THE FETUS?

AMNIOCENTESIS TO DETERMINE
KARYOTYPE
Amniocentesis para determinar el cariotipo
CULTURE FOR HERPES SIMPLEX VIRUS Cultivo para detectar la presencia del virus del herpes
simple
SERIAL RUBELLA TITERS Valoraciones en serie de rubola
URINALYSIS FOR CYTOMEGALOVIRUS Anlisis de orina para detectar la presencia del
citomegalovirus

Bibliografa: -



21/05/13 18:30 Simulador Proedumed
Pgina 1 de 1 http://www.proedumed.com.mx/simulador/pages/examen/resultadoPregunta.faces
Anlisis del Caso Clnico
Identificacin del reactivo
Area: GINECOLOGA Y OBSTETRICIA
Especialidad: GINECOLOGA
Tema: TRANSTORNOS MENSTRUALES
Subtema: HEMORRAGIA UTERINA DISFUNCIONAL
CASO CLNICO CON UNA PREGUNTA

MUJER DE 41 AOS DE EDAD, SIN ANTECEDENTES DE IMPORTANCIA, CON DIAGNSTICO ACTUAL DE HEMORRAGIA UTERINA DISFUNCIONAL. REFIERE PARIDAD
SATISFECHA.

ELEMENTOS CLAVE A CONSIDERAR EN EL CASO CLNICO:
Edad: Mujer de 41 aos de edad
Antecedentes: Paridad satisfecha diagnostico de hemorragia uterina disfuncional
Sintomatologa:
Exploracin:
Laboratorio y/o gabinete:

66 - EL TRATAMIENTO DE PRIMERA ELECCIN EN ESTA PACIENTE SER CON:

COLOCACIN DE
DIU-L.
EL D U L reduce el sangrado en ms de 95 % de los casos tratados con
beneficio mximo a los 6 meses El uso de D U levonogetren es la opcin que
mejores resultados ofrece en el control de la hemorragia uterina disfuncional de
acuerdo a las Gu as de Prctica Cl nica de la Secretar a de Salud La revisin de
ensayos cl nicos determin que un D U L mejora la calidad de vida de las
mujeres con menorragia tan efectivamente como el tratamiento quirrgico
mientras que una minor a de mujeres prefiere la medicacin oral a largo plazo
Corresponde al mtodo de eleccin en el caso de sangrado uterino disfuncional
es muy efectivo y con buenos resultados a largo plazo
HISTERECTOMA. La finalidad del tratamiento de la hemorragia uterina disfuncional es evitar
episodios agudos de hemorragia prevenir la anemia y mejorar la calidad de vida
de las pacientes La histerectom a es la extirpacin quirrgica del tero Esta
indicada slo cuando ha habido falla de tratamiento mdico en pacientes sin
inters de embarazo La histerectom a debe ser siempre el ltimo recurso en el
manejo de la hemorragia uterina disfuncional Recuerda que "El tratamiento de
eleccin para el sangrad uterino disfuncional es mdico"
NORETISTERONA
CCLICA.
Norestisterona oral dosis de 15mg por d a del 5 al 16 d a del ciclo menstrual
reduce el sangrado en ms de 83 % de los casos Este esquema es el que mejor
para controlar la prdida hemtica a corto plazo Constituye la segunda opcin en
el manejo de la hemorragia uterina disfuncional
PROGESTERONA
POR 21 DAS.
El uso de progesterona por 21 d as parece reducir de manera significativa la
prdida sangu nea en mujeres con hemorragia uterina disfuncional puede
administrarse de manera inmediata para el control de la hemorragia y slo como
un tratamiento temporal Tiene un uso adicional en anovulacin crnica No es el
manejo de primera eleccin y dado que la paciente refiere paridad satisfecha
requiere de manejo prolongado por lo que la progesterona por 21 d as "No" est
indicada

Bibliografa: GPC. DIAGNSTICO Y TRATAMIENTO DE LA HEMORRAGIA UTERINA DISFUNCIONAL. IMSS-322-10

http://www.cenetec.salud.gob.mx/descargas/gpc/CatalogoMaestro/322 IMSS 10 Hemorragia uterina disfuncional/EyR IMSS 322 10.pdf


21/05/13 18:31 Simulador Proedumed
Pgina 1 de 1 http://www.proedumed.com.mx/simulador/pages/examen/resultadoPregunta.faces
Anlisis del Caso Clnico
Identificacin del reactivo
Area: GINECOLOGA Y OBSTETRICIA
Especialidad: GINECOLOGA
Tema: PATOLOGA BENIGNA Y MALIGNA DE OVARIO
Subtema: PATOLOGA BENIGNA Y MALIGA DE OVARIO
CASO CLNICO CON UNA PREGUNTA

MUJER DE 32 AOS DE EDAD, EN QUIN SOSPECHA LA PRESENCIA DE ENFERMEDAD TROFOBLSTICA GESTACIONAL BENIGNA. DURANTE SU ESTUDIO SE
ENCUENTRAN QUISTES TECALUTENICOS.

ELEMENTOS CLAVE A CONSIDERAR EN EL CASO CLNICO:
Edad: Mujer en edad reproductiva
Antecedentes:
Sintomatologa:
Exploracin:
Laboratorio y/o gabinete: para detectarlos se debio realizar ultrasonido elemento fundamental
para realizar el diagnstico

67 - ESTE HALLAZGO PUEDE SER EXPLICADO POR AUMENTO EN LA PRODUCCIN DE:

17 ALFA
HIDROXIPROGESTERONA
El embarazo molar mola hidatiforme o mola vesicular es la
consecuencia de una alteracin gentica que acontece en el momento
de la fecundacin Se caracteriza segn la definicin clsica de Hertig
por la degeneracin hidrpica y el edema de estroma vellositario la
ausencia de vascularizacin en las vellosidades coriales y la
proliferacin del epitelio trofoblstico pero conservndose la estructura
diferenciada vellositaria Se incluye junto a los tumores trofoblsticos
gestacionales (Mola invasiva o corioadenoma destruens coriocarcinoma
y tumor trofoblstico del lecho placentario) dentro de la enfermedad
trofoblstica gestacional (ETG) siendo la forma ms benigna de este
conjunto de alteraciones proliferativas del trofoblasto Es una
enfermedad localizada que normalmente se resuelve con la evacuacin
uterina y en principio no debe ser considerada ni invasiva ni
neoplsica ni maligna Sin embargo en un porcentaje significativo de
casos (alrededor del 10%) da lugar a una enfermedad trofoblstica
persistente (ETP) por lo que es imprescindible un adecuado
seguimiento tras su evacuacin ncluso puede aparecer tras un parto a
trmino con feto vivo (1/50 000) Mola total o completa: es la forma ms
frecuente de presentacin Se origina como consecuencia de la
fecundacin de un vulo "vaco" con material gentico ausente o
inactivo Por tanto la carga cromosmica es slo de origen paterno
comportndose como un heterotrasplante En ms del 90% de los
casos la fecundacin se produce por un espermatozoide 23X que se
duplica resultando un cariotipo 46XX Ms raramente el vulo "vaco"
es fecundado por dos espermatozoides 23X y 23Y siendo el cariotipo
46XY El riesgo de ETP es del 15 20% Mola parcial: En contraste con la
mola completa tiene material gentico materno El cariotipo suele ser
triploide generalmente 69XXY como consecuencia de la fecundacin
de un vulo por dos espermatozoides Con mucha menor frecuencia el
cariotipo es diploide El riesgo de ETP es menor (4 8%)
ESTRIOL Diagnstico Gracias a la ecografa sistemtica del primer trimestre de la
gestacin su diagnstico es cada da ms precoz y ello ha modificado
su espectro de signos y sntomas clsicos Hay que pensar en la
presencia de una mola vesicular cuando en dicho periodo aparecen las
siguientes manifestaciones clnicas: 1 Metrorragia es el signo ms
habitual (97%) y el motivo principal de consulta Es variable en
frecuencia e intensidad provocando a veces una anemia grave Se
produce por la rotura de los vasos maternos al separarse las vesculas
de la decidua En ocasiones se acompaa de dolor en hipogastrio e
hidrorrea 2 Nusea vmitos estn presentes en el 30% de los casos
y se deben al igual que el aumento de los sntomas subjetivos de
embarazo al incremento de los niveles de HCG 3 Preeclampsia 4
Expulsin de vesculas 5 Hipertiroidismo 6 nsuficiencia respiratoria
aguda: es excepcional (2%) y guarda relacin con la embolizacin
pulmonar de clulas trofoblsticas as como con la asociacin de
preeclampsia e hipertiroidismo Exploracin ginecolgica 1 Cervix
cerrado Metrorragia de cuanta variable Rara vez se observa la
expulsin de vesculas 2 Desproporcin entre el tamao uterino y la
edad gestacional a favor del primero (60%) No obstante en el 20% de
las pacientes el tamao uterino se corresponde con la amenorrea y en el
20% restante es incluso menor que la misma El tero es regular
simtrico y de consistencia blanda 3 Tumoraciones ovricas (30%)
que son quistes tecalutenicos muchas veces bilaterales
GONADOTROPINA
CORIONICA
Se conoce con el nombre de enfermedad trofoblstica gestacional
(ETG) a un conjunto de procesos benignos y malignos poco habituales
derivados de una proliferacin anormal del trofoblasto de la placenta
humana (hiperplasia) y del genoma paterno con una contribucin
materna ocasional; incluye la mola hidatiforme completa invasiva o no
la mola parcial y los tumores trofoblsticos gestacionales
coriocarcinoma y tumor del lecho o sitio placentario (TSP) Es
extremadamente raro hallar trastornos similares en otras especies
diferentes a la humana Estas enfermedades se consideran peculiares
ya que el producto de la concepcin a partir del cual se originan es
genticamente extrao al anfitrin materno La ETG representa un
espectro nico de patologas interrelacionadas con el denominador
comn de una hipersecreccin de gonadotropina corinica (hCG) que
constituye un marcador tumoral sensible que se correlaciona bien con la
progresin y persistencia de la enfermedad excepto el TSP que
produce lactgeno placentario (hPL) con tendencias variables a la
invasin local y a las metstasis y se encuentra entre las raras
enfermedades que se pueden curar incluso en casos de extensa
diseminacin La mola hidatiforme es un producto de la concepcin que
se caracteriza por una hiperplasia trofoblstica y por la tumefaccin
edematosa de las vellosidades corinicas cuyo dimetro oscila entre
0 1 y 3 cm adquiriendo la morfologa de bandas y cmulos de vesculas
que confieren el tpico aspecto de racimos de uvas La variedad de
mola hidatiforme ms fcilmente identificable es la completa o clsica
en la que falta el feto y donde todas las vellosidades presentan
degeneracin hidrpica y son avasculares aunque puede observarse
algn vaso degenerado siendo notable la hiperplasia del citotroblasto y
del sincitiotrofoblasto Se suele identificar precozmente por un patrn
ecogrfico caracterstico descrito hace muchos aos Ecografa Es de
gran utilidad aportando informacin sobre el contenido uterino aspecto
de los ovarios as como sobre una posible invasin miometrial Son
datos ultrasnicos sugestivos de embarazo molar los siguientes: 1
tero mayor que amenorrea aunque puede ser igual o menor 2
Ausencia de estructuras embrionarias en la mola completa 3 Cavidad
ocupada por multitud de ecos de baja amplitud que corresponden al
tejido trofoblstico proliferado Es la imagen tpica de Copos de nieve o
Panal de abejas A veces se visualizan zonas anecoicas que traducen
la presencia de hemorragias intratumorales 4 Quistes tecalutenicos
que ofrecen una imagen ultrasnica redondeada econegativa y
multilocular la mayor de las veces bilateral Determinacin de B HCG
Por su alta sensibilidad y especificidad es de gran utilidad en el
diagnstico y seguimiento de la enfermedad trofoblstica gestacional La
determinacin de B HCG puede efectuarse en plasma y en orina En el
embarazo normal sus valores se incrementan progresivamente hasta
alcanzar las 100 000mU /ml en la semana 12 para despus ir
descendiendo En la mola vesicular sus niveles se encuentran muy
elevados aunque su curva siempre debe compararse con la obtenida en
el embarazo normal para la misma edad gestacional laboratorio y
mtodo Cifras > 200 000 mU /ml sin embargo son muy sugestivas de
enfermedad trofoblstica gestacional
GONADOTROPINAS
HIPOFISARIAS
Tratamiento especfico Evacuacin de la mola El modo de llevarla a
cabo va a depender del estado de la enferma intensidad del sangrado
tamao uterino edad y deseos reproductivos futuros En la mujer menor
de 40 aos y que quiere tener ms descendencia el mtodo de eleccin
es el legrado por aspiracin que se completa con el paso suave de una
legra cortante Es recomendable su realizacin bajo control ecogrfico
Los agentes oxitcicos se administraran tras la dilatacin cervical y
evacuacin parcial a fin de facilitar la hemostasia Las contracciones
uterinas previas a la evacuacin pueden facilitar embolizaciones de
material trofoblstico Por eso no se recomienda la induccin del aborto
con agentes oxitcicos o con prostaglandinas para la preparacin del
cuello antes del legrado Las principales complicaciones de la
evacuacin de la mola son la perforacin uterina hemorragia infeccin
y la embolizacin pulmonar trofoblstica El legrado de repeticin no
est indicado salvo que persistan restos molares En determinados
casos (edad > 40 aos edad comprendida entre 35 40 aos con la
descendencia deseada patologa uterina asociada perforacin uterina o
hemorragia incontrolable)se puede contemplar la histerectoma
abdominal puesto que el riesgo de enfermedad trofoblstica gestacional
es mucho mayor Aunque existan quistes tecalutenicos los anejos se
pueden conservar A las gestantes (Rh negativo) se les debe administrar
inmunoglobulina anti D en las primeras 48 72 horas tras la evacuacin
Seguimiento posterior a la evacuacin Una vez finalizado el embarazo
molar se plantean dos posibles opciones La primera es administrar
quimioprofilaxis (metotrexate o antinomicina D) a todas las pacientes o
al menos a aqullas con factores de riesgo o si el seguimiento de la
paciente es imposible Otra opcin ms conservadora consiste en
realizar un estricto seguimiento: 1 Determinaciones de B HCG
plasmtica: inicialmente de forma semanal hasta alcanzarse 3 ttulos
negativos (<5 mU /ml) consecutivos A continuacin las
determinaciones se harn de forma peridica durante 6 12 meses 2
Control ginecolgico y ecogrfico seriado: valorando fundamentalmente
el tamao y la consistencia del tero y la aparicin de metrorragia Se
practicarn a las dos semanas de la evacuacin y despus cada tres
meses 3 Estudio radiolgico torcico: la periodicidad del mismo
depender de la evolucin clnica y analtica de cada paciente Durante
el tiempo de seguimiento se ha de evitar una nueva gestacin En el
90% de los casos de proceso se resuelve satisfactoriamente y los ttulos
de B HCG van descendiendo negativizndose en 6 10 semanas Se
habla de remisin cuando se obtienen tres ttulos negativos
consecutivos La paciente debe evitar una nueva concepcin al menos
hasta que lleve seis meses con cifras de B HCG normales El riesgo de
una nueva gestacin molar es bajo (1/55)

Bibliografa: TRATADO DE GINECOLOGA DE NOVAK. JONES III, H.W. INTERAMERICANA MC GRAW HILL. EDICIN 11A. 1991.



21/05/13 18:32 Simulador Proedumed
Pgina 1 de 2 http://www.proedumed.com.mx/simulador/pages/examen/resultadoPregunta.faces
Anlisis del Caso Clnico
Identificacin del reactivo
Area: GINECOLOGA Y OBSTETRICIA
Especialidad: OBSTETRICIA
Tema: COMPLICACIONES DEL EMBARAZO Y DEL PARTO
Subtema: ATENCIN DEL PARTO Y DISTOSIAS
CASO CLNICO SERIADO

MUJER DE 39 AOS DE EDAD PRIMIGESTA, QUE ACUDE CON TRABAJO DE PARTO DE 36 HORAS QUE ESTABA SIENDO ATENDIDO EN SU DOMICILIO POR PARTERA.

ELEMENTOS CLAVE A CONSIDERAR EN EL CASO CLNICO:
Edad: 39 AOS, PRIMIGESTA
Antecedentes: TRABAJO DE PARTO 36HRS
Sintomatologa: -
Exploracin: -
Laboratorio y/o gabinete: -


68 - DURANTE SU VALORACIN USTED DEBER CONSIDERAR EL SIGUIENTE FACTOR COMO EL DE MAYOR RIESGO PARA GENERAR UNA DISTOCIA DEL
TRABAJO DE PARTO:

TALLA
MENOR A
1.50M
Algunos de los factores relacionados las distocias son: mal control prenatal, bajo peso
fetal al nacer, baja estatura materna, cesrea previa, gran multiparidad, nuliparidad,
embarazos postrmino, trabajo de parto prolongado y excesiva ganancia de peso
durante el embarazo. La distocia de hombros es la ms frecuente de todas las distocias.
Uno de los factores de riesgo materno ms importante para sta es la talla baja. EN
ORDEN DE FRECUENCIA DISTOCIA DE HOMBROS OCUPA EL PRIMER LUGAR DE
DISTOCIAS DEL TRABAJO DE PARTO Y, LA BAJA TALLA ES UNO DE SUS
PRINCIPALES FACTORES DE RIESGO POR LO QUE ELEGIMOS STA OPCIN
COMO CORRECTA.
EDAD
ENTRE LOS
35-40AOS
La edad materna mayor a 35 aos y menor a 18, es un factor de riesgo para las
distocias del trabajo de parto. AUNQUE SI ES UN FACTOR DE RIESGO ASOCIADO
PRESENTA EL MISMO RIESGO EN JVENES ADOLESCENTES POR LO QUE SE
DESCARTA.
INDICE DE
MASA
CORPORAL
MAYOR DE
27
La obesidad constituye un factor de riesgo importante para las distocias. Obesidad se
considera cuando el indice de masa corporal es mayor o igual a 30. En el embarazo el
aumento de peso estar determinado por el peso previo al embarazo y el aumento
ponderal de acuerdo a la semana de gestacin. La excesiva ganancia de peso durante
el embarazo es el principal factor relacionado con el peso. EL NDICE DE MASA
CORPORAL SIMPLE NO ES VALORABLE DURANTE EL EMBARAZO POR LO QUE
NO SE CONSIDERA UN FACTOR DE RIESGO EN STE CASO.
PARIDAD Las distocias en general se presentan en un 25% del total de los partos de pacientes en
nulparas, y en un 10% de los partos en multparas; stas prevalencias varan mucho de
acuerdo al tipo de distocia. NO LO CONSIDERAMOS EL FACTOR PRINCIPAL YA QUE
TANTO LA NULIPARIDAD COMO LA MULTIPARIDAD SON FACTORES DE RIESGO
DE DISTOCIA DEL TRABAJO DE PARTO.

Bibliografa:GINECOLOGIA Y OBSTETRICIA APLICADAS. AHUED AHUED J. ROBERTO. MANUAL MODERNO. EDICIN 2. 2003. PAG. 199.



69 - DURANTE LA EXPLORACIN DE LA PACIENTE USTED DEFINIR LA PRESENTACIN COMO:

POSICIN QUE
ADQUIERE EL
PRODUCTO DE
ACUERDO AL EJE
MATERNO
La SITUACIN FETAL es la relacin entre el eje longitudinal del feto respecto
del de la madre y puede ser longitudinal o transversal. STA
CARACTERSTICA CORRESPONDE A LA SITUACIN FETAL.
RELACION QUE
GUARDA EL
PUNTO
TOCONMICO
CON LA
HEMIPELVIS
MATERNA
La POSICIN FETAL es la relacin de una porcin arbitrariamente elegida de
la presentacin respecto del lado derecho o izquierdo del conducto del parto.
Con cada presentacin puede haber 2 variedades de posicin, derecha o
izquierda. El occipucio, el mentn y el sacro fetales son los puntos
determinantes de las presentaciones de vrtice, cara y plvica,
respectivamente. ESTA OPCIN CORRESPONDE A LA DEFINICIN DE
POSICIN FETAL.
PARTE DEL
PRODUCTO QUE
SE ABOCA AL
ESTRECHO
SUPERIOR Y
DESENCADENA EL
TRABAJO DE
PARTO
PRESENTACIN es la parte del producto que se aboca al estrecho superior de
la pelvis materna que es capaz de llenarlo en su totalidad y desencadena
trabajo de parto. Se puede percibir a travs del cuello uterino por tacto vaginal.
PUNTO DE LA
PRESENTACIN
QUE NOS ORIENTA
HACIA LA
SITUACIN DEL
PRODUCTO
El PUNTO TOCONMICO es el punto de referencia de la presentacin para
conocer la variedad de posicin de la presentacin.

Bibliografa:GINECOLOGIA Y OBSTETRICIA APLICADAS. ROBERTO AHUED / CARLOS FERNANDEZ DEL CASTILLO. MANUAL MODERNO. EDICIN 2. 2003. PAG.
261.


FIN DEL CASO CLNICO SERIADO

21/05/13 18:33 Simulador Proedumed
Pgina 1 de 1 http://www.proedumed.com.mx/simulador/pages/examen/resultadoPregunta.faces
Anlisis del Caso Clnico
Identificacin del reactivo
Area: GINECOLOGA Y OBSTETRICIA
Especialidad: GINECOLOGA
Tema: TRANSTORNOS MENSTRUALES
Subtema: AMENORREA
CASO CLNICO CON UNA PREGUNTA

MUJER DE 38 AOS DE EDAD, CASADA DESDE HACE 1 AO, PRESENTANDO AMENORREA SIN EMBARAZO DESDE HACE 2 MESES. USTED SOSPECHA LA POSIBILIDAD DE
FALLA OVRICA PREMATURA.

ELEMENTOS CLAVE A CONSIDERAR EN EL CASO CLNICO:
Edad: Mujer de 38 aos de edad
Antecedentes: CASADA
Sintomatologa: AMENORREA
Exploracin:
Laboratorio y/o gabinete:

70 - USTED SOLICITA NIVELES SRICOS DE FSH Y LH ESPERANDO ENCONTRARLAS:

LH
ALTA
Y FSH
ALTA.
Hasta el momento los investigadores no han podido identificar exactamente qu pasa en la
falla ovrica prematura que en la mayor a de los casos hace que se detenga el
funcionamiento normal de los ovarios Es importante recordar que los niveles de FSH son
altos cuando los ovarios no logran producir suficiente estrgeno Los niveles de LH tambin
se mantienen altos en muchos casos an en las raras ocasiones en que los fol culos logran
crecer exitosamente Los fol culos maduros en los ovarios producen estrgeno as como
otras sustancias incluyendo la prote na inhibina En mujeres con falla ovrica prematura tienen
niveles bajos de estrgeno
LH
BAJA
Y FSH
ALTA.
LH
ALTA
Y FSH
BAJA.
LH
BAJA
Y FSH
BAJA.

Bibliografa: GINECOLOGIA DE NOVAK. BEREK S. JONATHAN. MC GRAW-HILL. INTERAMERICANA. EDICIN 12. 1996. PG. 854.



21/05/13 18:33 Simulador Proedumed
Pgina 1 de 1 http://www.proedumed.com.mx/simulador/pages/examen/resultadoPregunta.faces
Anlisis del Caso Clnico
Identificacin del reactivo
Area: GINECOLOGA Y OBSTETRICIA
Especialidad: GINECOLOGA
Tema: PATOLOGA BENIGNA Y MALIGNA DE OVARIO
Subtema: PATOLOGA BENIGNA Y MALIGA DE OVARIO
CASO CLNICO CON UNA PREGUNTA

MUJER DE 23 AOS DE EDAD QUE ACUDE A LA CONSULTA PARA RECIBIR RESULTADOS DEL ESTUDIO PATOLGICO REALIZADO A UN TUMOR DE OVARIO EXTIRPADO
HACE UN MES. SE REPORTA TUMOR OVRICO QUE CONTIENE ELEMENTOS DERIVADOS DE LAS TRES CAPAS EMBRIONARIAS.

ELEMENTOS CLAVE A CONSIDERAR EN EL CASO CLNICO:
Edad: Mujer en edad fertil. en los tumores cuyo origen son las clulas
germinales, la edad es un elemento imporante que gua el diagnstico.
Antecedentes: -
Sintomatologa: -
Exploracin: -
Laboratorio y/o gabinete: El contenido macroscpico y microscpico del tumor, establece en todos
los casos el diagnstico diferencial de los tumores de ovario de origen
germinal.

71 - EN ESTE CASO SE TRATA DE UN:

CARCINOMA
EMBRIONARIO
la paciente con diagnstico de carcinoma embrionario siempre son ms
jvenes, con una edad promedio de 14 aos, que aquellas con otro tipo de
tumores de clulas germinales. Dichas neoplasias se forman de clulas
epiteliales parecidas a las del disco embrionario. Son distintivos las hojas
slidas desorganizadas de grandes clulas anaplsicas, espacios semejantes a
los glandulares y estructuras capilares y permiten la fcil identificacin de estos
tumores . Los carcinomas embrionarios casi siempre producen hCG y 75%
tambin secretan fetoprotena alfa.
DISGERMINOMA La mitad de los tumores ovricos malignos de clulas germinales son
disgerminomas y son las neoplasias malignas ovricas ms frecuentes
detectadas durante el embarazo. OJO. Tumor de ovario y embarazo. El 5% de
los disgerminomas se descubre en mujeres fenotpicas con anomalas
cariotpicas en las gnadas, en particular, con la presencia de un cromosoma Y
normal o anormal. Los digerminomas son los nicos tumores malignos de
clulas germinales con un ndice significativo de compromiso ovrico bilateral,
15 a 20%. El tratamiento incluye ciruga conservadora de fertilidad con
salpingooforectoma unilateral y estadificacin quirrgica cuidadosa de la
enfermedad. Los disgerminomas tienen el mejor pronstico de todas las
variantes tumorales ovricas de clulas malignas. La supervivencia a 5 aos es
mayor del 95%. Se consider muy sensible a la quimioterapia.
TERATOMA
INMADURO
Los teratomas inmaduros representan el 20% de todos los tumores ovaricos
malignos de clulas germinales y se aproximan a la frecuencia de los tumores
del saco vitelino. Estn formados por tejidos derivados de las tres capas
germinales: ectodermo, mesodermo y endodermo. Sin embargo, la presencia
de estructuras inmaduras o embrionarias distingue a estas lesiones del
teratoma qustico maduro benigno (quiste dermoide) que es mucho ms
comn). Los marcadores tumorales a menudo son negativos. La mayor parte de
los teratomas, son asintomticos y la forma ms frecuente de presentacin son
el dolor abdominal (48%) y hemorragia uterina anormal o concomitante (15%) o
aumento del volumen abdominal (15%). La rotura es rara, entre el 1-1,2% y
constituye una urgencia quirrgica. El tratamiento es quirrgico, siendo posible
la mayor parte de las veces una reseccin del mismo, respetando el resto del
ovario. La ecografa constituye el modelo diagnstico de eleccin y la
combinacin de ecografa con radiografa simple de abdomen proporciona un
diagnstico ms exacto en la mayora de los casos, siendo la resonancia
magntica o la TAC el que aporte el diagnstico diferencial.
CORIOCARCINOMA El coriocarcinoma ovrico primario originado de una clula germinal es similar a
un coriocarcinoma gestacional con metastasis ovricas. Es importante hacer la
diferenciacin porque los tumores no gestacionales tienen peor pronstico. La
deteccin de otros componentes de clulas germinales indica coriocarcinoma
no gestacional, mientras que un embarazo concomitante o prximo sugiere una
forma gestacional. Las manifestaciones clnicas son frecuentes y se deben a
concentraciones altas de hCG que producen dichas neoplasias. Tales
concentraciones pueden inducir precocidad sexual en nias prepuberales o
menometrorragia en mujeres en edad reproductiva.

Bibliografa: GINECOLOGA DE NOVAK. JONATHAN BEREK. MC. GRAW HILL. EDICIN 12A. 2004. PAG. 1035.



21/05/13 18:34 Simulador Proedumed
Pgina 1 de 1 http://www.proedumed.com.mx/simulador/pages/examen/resultadoPregunta.faces
Anlisis del Caso Clnico
Identificacin del reactivo
Area: GINECOLOGA Y OBSTETRICIA
Especialidad: OBSTETRICIA
Tema: COMPLICACIONES DEL EMBARAZO Y DEL PARTO
Subtema: ISOINMUNIZACIN RH
CASO CLNICO CON UNA PREGUNTA

MUJER DE 25 AOS GESTA 2, ABORTO 1, DESCONOCE CAUSA, ACUDE A CONTROL PRENATAL POR AMENORREA DE 7 SEMANAS Y PRUEBA DE EMBARAZO POSITIVA.
REFIERE SER TIPO SANGUNEO " O ", RH NEGATIVO.

ELEMENTOS CLAVE A CONSIDERAR EN EL CASO CLNICO:
Edad:
Antecedentes: ABORTO PREV O EMBARAZO 7SDG
Sintomatologa:
Exploracin:
Laboratorio y/o gabinete: T PO SANGU NEO "o" rh NEGAT VO

72 - EN ESTE CASO DEBER RECOMENDARSE:

ABORTO
TERAPUTICO
El aborto teraputico se reserva a aquellos casos en los que la patolog a fetal no
es compatible con la vida o las posibilidades de sta son muy bajas o en los
casos en los que continuar con la gestacin pone en riesgo la vida de la madre
EN EL CASO DE LA SO NMUN ZAC N POR RH SE PUEDE PREVEN R LA
ACT VAC N DE LA RESPUESTA NMUN TAR A ANTES DE QUE SE
PROVOQUEN LES ONES QUE PONGAN EN R ESGO AL FETO POR LO QUE
NO ES UNA OPC N RECOMENDABLE
AMNIOCENTESIS La amniocentesis es el procedimiento diagnstico que consiste en la obtencin de
l quido amnitico por puncin transabdominal preferentemente entre las semanas
16 18 (posible tambin >18 semanas) Las indicaciones de la amniocentesis son
estudio citogentico (cariotipo) riesgo de enfermedad monognica con
diagnstico bioqu mico en l quido amnitico riesgo de defecto del tubo neural
PCR para infeccin fetal estudio de madurez pulmonar y cuantificacin de
bilirrubinas fetales en isoinmunizacin materno fetal (permite valorar la
destruccin de los glbulos rojos existente) ES DE UT L DAD UNA VEZ QUE SE
SOSPECHA ER TROBLASTOS S FETAL SECUNDAR A A SO NMUN ZAC N
MATERNO FETAL CONF RMADA COOMBS ND RECTO POS T VO Toma en
cuenta que el manejo en las primeras semanas de embarazo est dirigido antes
que todo a la prevencin de respuesta materna a la sangre fetal
CESREA
ELECTIVA
Se aconseja el parto vaginal en los casos de isoinmuniacin materno fetal
siguiendo los controles de bienestar fetal intraparto habituales En caso de riesgo
de perdida fetal o en presencia de otra circunstancia obsttrica que as lo
requiera se recurrir a la practica de una cesrea LA CESREA SLO EST
ND CADA EN LOS CASOS DE COMPROM SO FETAL EN LA
SO NMUN AC N MATERNO FETAL
PROFILAXIS RH
PRENATAL
La gammaglbulina anti Rh esta indicada en todas las gestantes Rh negativas en la
semana 28 del embarazo a dosis de 1500U (300ng) intramuscular Se aconseja
la profilaxis con inmunoglobulina despus del primer contacto con el ant geno
(parto previo aborto embarazo ectpico) LA APL CAC N DE
GAMAGLOBUL NA ANT RH ES EL TRATAM ENTO DEAL EN LA
PREVENC N DE LA SO NMUN ZAC N MATERNO FETAL

Bibliografa: MEDICINA CLNICA EN OBSTETRICIA. NORBERT GLEICHER. PANAMERICANA. EDICIN 1RA. PAG. 1357.



21/05/13 18:34 Simulador Proedumed
Pgina 1 de 1 http://www.proedumed.com.mx/simulador/pages/examen/resultadoPregunta.faces
Anlisis del Caso Clnico
Identificacin del reactivo
Area: GINECOLOGA Y OBSTETRICIA
Especialidad: GINECOLOGA
Tema: TRANSTORNOS MENSTRUALES
Subtema: HEMORRAGIA UTERINA ANORMAL
CASO CLNICO SERIADO

MUJER DE 36 AOS DE EDAD DEPORTISTA CON ANTECEDENTE DE EVENTO TROMBOTICO EN PIERNA IZQUIERDA. ACUDE A CONSULTA REFIRIENDO CAMBIOS EN SU
CICLO MENSTRUAL A PARTIR DE SU LTIMO EMBARAZO HACE 2 AOS. CUENTA CON LOS SIGUIENTES ANTECEDENTES: MENARCA 13, INICIO DE VIDA SEXUAL 18 AOS,
RITMO 33X9 ABUNDANTE, GESTA 3, PARA 3. EN EL LTIMO PARTO SE REALIZA OCLUSIN TUBREA BILATERAL.

ELEMENTOS CLAVE A CONSIDERAR EN EL CASO CLNICO:
Edad: Femenina de 36 aos
Antecedentes: Trombosis en miembros plvicos
Sintomatologa: Mentruacin ritmo 33x9 abundante
Exploracin:
Laboratorio y/o gabinete:

73 - LA PACIENTE PRESENTA LA SIGUIENTE ALTERACIN MENSTRUAL:

MENORRAGIA LA DURAC N DE LA MENTRUAC N NORMAL ES DE 2 A 7 DAS CON UN
PROMED O DE 4 EL NTERVALO MESNTRUAL NORMAL T ENE UN RANGO
DE 25 A 35 DAS CON UN PRMED O DE 31 La MENORRAG A se define
como la menstruacin cclica prolongada o abundante La MENORRAG A
OVULATOR A es consecuencia bsicamente de la dilatacin vascular Las
mujeres con hemorragia ovulatoria pierden sangre a una velocidad tres veces
mayor que aquellas con una menstruacin normal pero no aumento el nmero
de arteriolas espirales Por lo tanto en stas mujeres se cree que los vasos
sanguneos que irrigan al endometrio tienen un tono vascular reducido y por lo
tanto la dilatacin genera una hemorragia mayor Se han sugerido varias causas
que provocan este cambio en el tono vascular y en ellas A MENUDO
PART C PAN LAS PROSTAGLAND NAS
METRORRAGIA La METRORRAG A describe la hemorragia intermenstrual Sangrado o
hemorragia intermenstrual es el sinnimo informal de la metrorragia que
acompaa a la administracin de hormonas El sangrado intermenstrual que se
acompaa de menstruaciones prolongadas y abundantes se conoce como
MENOMETRORRAG A
HIPOMENORREA La menstruacin escasa o de corta duracin se denomina H POMENORREA
OLIGOMENORREA La menstruacin normal aparece tpicamente cada 28 7 das Los ciclos con
intervalos mayores a 35 das se conocen como OL GOMENORREA

Bibliografa:- WILLIAMS. GINECOLOGA. MCGRAW-HILL INTERAMERICANA EDITORES. TRADUCCIN DE LA PRIMERA EDICIN EN INGLS. 2009. - GUA DE PRCTICA
CLNICA. DIAGNOSTICO Y TRATAMIENTO DE LA HEMORRAGIA UTERINA DISFUNCIONAL. EVIDENCIAS Y RECOMENDACIONES. CATLOGO MAESTRO DE GUAS
DE PRCTICA CLNICA: IMSS-322-10.
HTTP://WWW.CENETEC.SALUD.GOB.MX/DESCARGAS/GPC/CATALOGOMAESTRO/322_IMSS_10_HEMORRAGIA_UTERINA_DISFUNCIONAL/EYR_IMSS_322_10.PDF
- GUA DE REFERENCIA RPIDA. DIAGNSTICO Y TRATAMIENTO DE LA HEMORRAGIA UTERINA DISFUNCIONAL. GPC. CATALOGO MAESTRODEG UIS DE
PRACTICA CLNICA: IMSS-322-10.

http://www.isssteags.gob.mx/guias praticas medicas/gpc/docs/IMSS-322-10-RR.pdf


74 - EL TRATAMIENTO DE ELECCIN EN STE CASO ES:

AINES stos frmacos se administran por va oral son eficaces y bien tolerados en el
tratamiento de la Hemorragia Uterina Anormal (HUA) Se administran por la
supuesta participacin de las prostaglandinas en la patogenia de la HUA Las
mujeres pierden 90% del volumen sanguneo menstrual durante los primeros 3
das de la menstruacin Por lo tanto los A NES son efectivos al principio de la
menstruacin o poco antes y se prolongan durante toda la hemorragia La
ventaja es que slo se ocupan durante la menstruacin Algunos autores
proponen que los inhibidores COX 2 son ms eficaces para el tratamiento de la
MENORRAG A por su accin en las prostaglandinas EN LA MENORRAG A
HAY UNA PART C PAC N MPORTANTE DE LAS PROSTAGLAND NAS POR
LO TANTO STE SE CONS DERA EL TRATAM ENTO DE ELECC N EN
STE CASO
PROGESTGENOS
ORALES
El estmulo estrognico sin oposicin resultado de los ciclos anovulatorios
provoca proliferacin del endometrio y una hemorragia errtica Los
progestgenos detienen el crecimiento endometrial y permiten una descamacin
organizada cuando se suprimen Por lo tanto el tratamiento con progestgenos
en las mujeres con HUD anovulatoria casi siempre es satisfactorio De los
progestgenos orales se pueden utilizar la noretindrona o el acetato e
medroxiprogesterona Por el contrario LA MENORRAG A OVULATOR A no es
consecuencia de una deficiencia de progesterona sino de la sntesis alterada de
prostaglandinas o a la alteracin de la hemostasia Por lo tanto NO RESPONDE
A LA ADM N STRAC N DE PROGESTGENOS
CIDO
TRANEXMICO
Es un antifibrtico que ejerce sus efectos al bloquear en forma reversible los
sitios de unin de lisina en el plasmingeno La concentracin reducida de
plasmina resultante amortigua la actividad fibrionoltica dentro de los vasos
endometriales evitando la hemorragi Este medicamento carece de efectos
sobre otros parpmetros de la coagulacin como cuenta plaquetaria tiempo
parcial de tromboplastina activada y tiempo de protrombina En las mujeres con
hemorragia uterina disfuncional la actividad fibrinoltica es mayor dentro del
endometrio que en las mujeres con menstruacin normal SU USO SE HA
L M TADO POR LA NQU ETUD EN RELAC N CON LAS COMPL CAC ONES
QUE PUD ERAN SURG R POR UNA MAYOR ACT V DAD TROMBT CA
GENERAL ZADA
ESTRGENOS
CONJUGADOS
Las dosis elevadas de estrgenos son tiles para regular los EP SOD OS DE
HEMORRAG A AGUDA ya que provocan un crecimiento rpido del endometrio
para cubrir las reas desnudas Los estrgenos conjugados de origen equino se
adminsitran por va oral a dosis hasta de 10mg diarios divididos cada 6hrs Una
vez que la hemorragia disminuye se puede cambiar a anticonceptivos orales
que se reducen gradualmente SE UT L ZAN SLO PARA EL MANEJO DE LA
HEMORRAG A AGUDA

Bibliografa:- WILLIAMS. GINECOLOGA. MCGRAW-HILL INTERAMERICANA EDITORES. TRADUCCIN DE LA PRIMERA EDICIN EN INGLS. 2009. - GUA DE PRCTICA
CLNICA. DIAGNOSTICO Y TRATAMIENTO DE LA HEMORRAGIA UTERINA DISFUNCIONAL. EVIDENCIAS Y RECOMENDACIONES. CATLOGO MAESTRO DE GUAS
DE PRCTICA CLNICA: IMSS-322-10. - GUA DE REFERENCIA RPIDA. DIAGNSTICO Y TRATAMIENTO DE LA HEMORRAGIA UTERINA DISFUNCIONAL. GPC.
CATALOGO MAESTRODEG UIS DE PRACTICA CLNICA: IMSS-322-10.

http://www.cenetec.salud.gob mx/descargas/gpc/CatalogoMaestro/322 IMSS 10 Hemorragia uterina disfuncional/EyR MSS 322 10.pdf

FIN DEL CASO CLNICO SERIADO

21/05/13 18:35 Simulador Proedumed
Pgina 1 de 1 http://www.proedumed.com.mx/simulador/pages/examen/resultadoPregunta.faces
Anlisis del Caso Clnico
Identificacin del reactivo
Area: GINECOLOGA Y OBSTETRICIA
Especialidad: GINECOLOGA
Tema: TRANSTORNOS MENSTRUALES
Subtema: AMENORREA
CASO CLNICO SERIADO

ADOLESCENTE DE 11 AOS DE EDAD, QUE ACUDE A LA CONSULTA DE GINECOLOGA POR PRESENTAR SU PRIMERA MENSTRUACIN. SU MADRE DESEA QUE SU HIJA
RECIBA INFORMACIN ADECUADA SOBRE ESTA NUEVA ETAPA DE SU VIDA.

ELEMENTOS CLAVE A CONSIDERAR EN EL CASO CLNICO:
Edad: adolescente
Antecedentes: menarca
Sintomatologa:
Exploracin:
Laboratorio y/o gabinete:

75 - EL TIEMPO PROMEDIO DE CADA CICLO MENSTRUAL QUE DEBER COMENTRSELE A LA PACIENTE ES DE:

5
DAS.
En la mujer el perodo frtil empieza con la Menarca y termina con la Menopausia Este
perodo es dividido en ciclos de 28 a 35 das separados por la menstruacin
14
DAS.
El ciclo se divide en 2 perodos de variable duracin: La fase folicular que precede a la
ovulacin y la fase ltea que sigue a la ovulacin La duracin de la fase folicular depende de
la velocidad de crecimiento de los folculos ovricos y vara de mujer en mujer En un estudio
de Bishop p De 20 mujeres dur 15 4 2 5 das La duracin de la fase ltea de la duracin
de la vida til del cuerpo lteo sera menos variable 13 6 1 2 das
28
DIAS.
La menstruacin; perodo regla es la prdida de sangre por la vagina desde el tero y
representa el inicio del ciclo sexual femenino o ciclo menstrual que es un ritmo de
aproximadamente 28 das de duracin y suele aparecer a partir de los 10 14 aos de edad y
dura hasta la menopausia Promedio de la duracin en la edad reproductiva 28 das siendo
ms prolongados en adolescencia (Ciclos anovulatorios) y peri menopausia (Disminucin de
E2 y alteraciones de las gonadotrofinas)
30
DAS.
La relacin FSH/LH es mayor en estas dos etapas a expensas de la FSH y la relacin se
estabiliza en la edad reproductiva En fase prepuberal es por estimulacin insuficiente del de
GnRH la aparicin de aumentos de LH durante el sueo refleja la maduracin del eje y estos
aumentos desaparecen en la pubertad El aumento de gonadotrofinas en peri post
menopausia es por disminucin del efecto de retroalimentacin ( ) de los esteroides ovricos y
la inhibina

Bibliografa: DIAGNOSTICO Y TRATAMIENTO GINECO-OBSTETRICO. H. DE CHERNEY. MANUAL MODERNO. EDICIN 7. 1999. PG. 148-151.



76 - SE LE DEBER INFORMAR QUE EL TIEMPO PROMEDIO DE DURACIN DE LA MENSTRUACIN ES DE:

1 A 2
DAS.
El ciclo ovrico es parte integral de un sistema integrado por el hipotlamo hipfisis ovario y
tero El reloj biolgico responsable de la ritmicidad de los ciclos es la secrecin pulstil de un
decapptido hipotalmico: GnRH Gonadotropin Re leasing Hormone La secrecin pulstil de
GnRH depende de eventos externos (Factores psicolgicos el ritmo nictaemeral) que llegan al
hipotlamo del cortex por el sistema lmbico y de eventos ovricos a travs del efecto de feed
back que los esteroides sexuales producen sobre hipotlamo e hipfisis Esta secrecin
modulada controla la produccin y sntesis de las gonadotrofinas polipptidas pituitarias la
FSH y LH
2 A 4
DAS.
La FSH realiza el reclutamiento y crecimiento de folculos ovricos al igual que la seleccin del
folculo dominante La LH induce la ruptura folicular y sostiene el cuerpo lteo El estradiol y
la Progesterona son producidos por los folculos y el cuerpo lteo cuya secrecin es
gonadotrofino dependiente Unidos a protenas transportadoras transitan por la sangre y
regulan la secrecin de GnRH FSH y LH y producen proliferacin y diferenciacin del
endometrio para facilitar la implantacin del embrin si la fertilizacin tuvo lugar
3 A 5
DAS.
La duracin media de la menstruacin es de 5( / 1) das
4 A 6
DAS.
Los folculos ovricos estn compuesto por una capa externa de clulas tecales y una capa
interna de clulas granulosas que envuelven al ovocito y contienen el antrum Las clulas
tecales tiene receptores para LH y produce andrgenos (Testosterona y androstenediona) Los
andrgenos atraviesan la membrana basal llegando a las clulas granulosas donde las
aromatasas transforman a estos en estrgenos (Estradiol y Estrona) La aromataza es una
enzima FSH dependiente y los receptores de FSH estn en las clulas granulosas Al final del
ciclo precedente la baja de E2 y P (Por la desaparicin del cuerpo lteo) disminuye el
feedback negativo en la FSH y esta empieza a aumentar en sangre antes de la aparicin de la
menstruacin reclutando una cohorte de folculos sensibles a la FSH

Bibliografa: DIAGNSTICO Y TRATAMIENTO GINECO-OBSTETRICO. H. DE CHERNEY. MANUAL MODERNO. EDICIN 7. 1999. PG. 148-151.



77 - LE COMENTAR QUE LA CANTIDAD PROMEDIO DE SANGRE QUE PERDER EN CADA MENSTRUACIN ES DE:

10 A
20ML.
Durante la primer semana despus de la menstruacin (Da 28 del ciclo) la FSH contina
aumentado los folculos crecen intensamente y la FSH aumenta la expresin de sus propios
receptores de LH en las clulas granulosas Durante este periodo los folculos producen
Estradiol en pequeas cantidades y sus niveles son constantes en sangre Durante la
segunda semana siguen creciendo los folculos incrementan los receptores de FSH en la
granulosa donde las aromatasas transforman los andrgenos tecales en E2 que aumenta
significativamente Este incremento induce un feed back ( ) en la FSH que disminuye
significativamente en sangre
30 A
40ML.
Volumen promedio: 25 a 40 ml
50 A
60ML.
El folculo con ms nmero de receptores de FSH la mxima actividad aromatasa y que
produce la mayor concentracin de E2 es el folculo dominante seleccionado para ovular
Los otros folculos degeneran gradualmente en un proceso denominado ATRES A El
estradiol contina incrementndose fundamentalmente a expensas del folculo dominante y
llega al pico 72 Hs antes de la ovulacin Este alto nivel de E2 induce por feed back positivo
una pequea produccin de FSH y LH que determinan su pico
70 A
80ML.
La ruptura folicular (Ovulacin) ocurre 36hs despus del pico de LH Esto es debido a que
las clulas granulosas adquieren receptores de LH (por efecto de la FSH) y ahora responde a
la LH Este pptido induce la secrecin de enzimas que digieren la pared folicular l
incremento inicial de LH al inicio de su pico es suficiente para que las granulosas secreten
pequeas cantidades de Progesterona que participa en el mecanismo de induccin del pico de
LH Una vez que el ovocito sale del folculo roto la LH induce la secrecin de Prog por las
clulas granulosas remanentes que se organizan en una nueva glndula llamada el cuerpo
lteo (luteinizacin) La prog y el E2 aumentan y llegan a su plateau al da 22 Esto induce un
feed back negativo en la LH y FSH que disminuyen sensiblemente en la circulacin Si la
implantacin no ocurre no hay hGC el cuerpo lteo no es sustentado por mucho tiempo y
disminuye el E2 y la Progesterona Esta disminucin induce un aumento de FSH que
empieza a reclutar folculos para el prximo ciclo La menstruacin aparece por que los
niveles de progesterona bajan a los requeridos para mantener el endometrio secretor

Bibliografa: DIAGNSTICO Y TRATAMIENTO GINECO-OBSTETRICO. H. DE CHERNEY. MANUAL MODERNO. EDICIN 7. 1999. PG. 148-151.




78 - PARA EXPLICAR EL CICLO MENSTRUAL, DEBER SABER QUE LA FASE DEL CICLO EN QUE EL ENDOMETRIO SE ENCUENTRA BAJO EL EFECTO
PRINCIPAL DE LOS ESTRGENOS ES EN LA FASE:

OVULATORIA. El ciclo menstrual humano se puede dividir en 4 fases: W 1 FASE FOL CULAR
(temprana media tarda) W 2 FASE OVULATOR A (transicin folicular lteo)
W 3 FASE LUTEA (temprana media y tarda) W 4 FASE MENSTRUAL
(transicin lteo folicular)
SECRETORA. FASE FOL CULAR 1 Aumento progresivo de E2 e inhibina por el Folculo de
Gaaf en desarrollo 2 El folculo gnesis empieza durante la fase lteo tarda y
sigue durante la transicin luteofolicular Por muerte del cuerpo lteo y
disminucin de nhibina y E2 y P aumenta la FSH 2 das antes del comienzo de la
menstruacin 3 El aumento de FSH junto al retorno de pulsos rpidos de LH
inicia el reclutamiento durante los 4 5 primeros das seguido de la seleccin del
folculo dominante (8 12 das) y luego la ovulacin (13 15 das) 4 La seleccin
folicular depende de andrgenos estrgenos progestgenos e inhibina
modulada por la P 450 y por otros factores de crecimiento que actan en forma
paracrina y autocrina 5 Dado que el E2 y la inhibina son potentes supresores de
la secrecin de FSH su disminucin en fase folicular media y tarda estara
relacionado con la suspensin secuencial por retroalimentacin de E2 e nhiba 6
La LH por su parte tiene tendencia al aumento
PROLIFERATIVA. FASEPROL FERAT VA (Estrognica): Bajo la influencia de los estrgenos que se
secretan en cantidades crecientes durante la fase proliferativa las clulas del
estroma y las clulas epiteliales proliferan rpidamente La superficie endometrial
se reepiteliza completamente en 4 a 7 das tras la menstruacin Durante el
periodo previo a la ovulacin el endometrio se espesa en parte debido al
creciente nmero de clulas del estroma pero primordialmente al crecimiento
progresivo de las glndulas endometriales y de nuevos vasos sanguneos en el
interior del endometrio Al momento de la ovulacin el endometrio tiene unos 3 a 5
milmetros de espesor Las glndulas endometriales especialmente las de la
regin cervical secretan un moco poco denso filante Esta fase se describe en
funcin de la dinmica histolgica y estructural del endometrio que se divide
como sigue: FASE PROL FERAT VA TEMPRANA Menos de 2 mm de espesor;
la superficie epitelial endometrial se restablece al 5 da del ciclo por proliferacin
de clulas de las capas basales Glndulas rectas estrechas y tubulares
cubiertas de clulas epiteliales columnares bajas con ncleos basales redondos
La actividad mittica del epitelio y el estroma es evidente al 5 da y sigue hasta 3
das luego de la ovulacin El citoplasma epitelial tiene muchos poliribosomas
pero el retculo endoplasmtico y el aparato de Golgi no est bien desarrollados
FASE PROL FERAT VA TARD A Se engruesa por hiperplasia glandular y
aumento de la sustancia basal estromtica Glndulas con amplia separacin
entre ellas en la zona superficial con ms tortuosidad en las zonas ms
profundas El epitelio glandular se vuelve ms alto y seudoestratifica cerca del
momento de la ovulacin FASE SECRETORA TEMPRANA Luego de la
ovulacin hay tres zonas endometriales separadas: ZONA BASAL adyacente al
endometrio ZONA ESPONJOSA NTERMED A por arriba de la basal y ZONA
COMPACTA inmediatamente por debajo de la superficie endometrial Las dos
ltimas forman el endometrio Funcional El edema del estroma es un factor
importante en el engrosamiento de esta fase y la actividad mittica se limita a 3
das luego de la ovulacin Las clulas epiteliales glandulares comienzan a
acumular vacuolas ricas en glucgeno en su base y hay moderada actividad
secretora como acumulaciones eosinfilas en la luz glandular FASE
SECRETORA MED A y TARD A 5 6mm De espesor muy vascularizado y rico
en glucgeno Glndulas tortuosas y su actividad secretora llega al mximo a los
6 das luego de la ovulacin Las clulas estromticas alrededor de los vasos
sanguneos aumentan de tamao (Predesidualizacin) y son los cambios previos
a la mayor transformacin endometrial del embarazo Es caracterstico el
desarrollo de arterias espiraladas que se enrollan y se alargan ms rpido que el
engrosamiento endometrial FASE PREMENSTRUAL nfiltracin estromtica de
leucocitos polimorfonucleares y mononucleares Glndulas con Agotamiento
secretor ncleos basales y desaparecen el sistema de canales nucleares y las
mitocondrias gigantes El esqueleto reticular del estroma empieza a
desintegrarse El espesor endometrial disminuye en los das previos a la
menstruacin como resultado de prdida de lquido tisular y secreciones
MENSTRUAC ON Empieza con la supresin de la P pero no se sabe si es por
esto en si o si participan reguladores intermediarios Estudios de Markee; hablan
de una fase isqumica con vasoconstriccin de las arterias espiraladas y
arteriolas 4 a 24 hs antes de la menstruacin La isquemia da una mala perfusin
a los tercios superiores del endometrio La hemorragia ocurre luego de que las
arterias espiraladas; luego de un perodo de constriccin se relajan el endometrio
superficial se distiende por la formacin de hematomas y se desarrollan fisuras
que llevan al desprendimiento en fragmentos del tejido Tambin hay autofagia y
heterofagia por presencia de macrfagos Hay aumento de la actividad de
fosfatasa acida endometrial durante la segunda mitad del ciclo y una aceleracin
posovulatoria del desarrollo lisosmico del estroma La alteracin de la
membrana lisosmica por disminucin de P libera hidrolasas que participan en
la degradacin del endometrio La produccin de PGF2a endometriales es
incrementada por fosfolipasas liposmicas que liberan cido araquidnico que se
metaboliza a prostanoides Durante la menstruacin se elimina una parte
significativa del endometrio funcional constituyendo fragmentos de tejido
mezclados con sangre licuados por la actividad fibrinoltica del endometrio
Duracin promedio: 4 6 das Volumen promedio: 25 a 40 ml
LTEA. FASE OVULATOR A El pico de la oleada de LH no puede definirse con
exactitud El comienzo de su oleada es el punto de referencia de la dinmica
hormonal e intrafolicular durante la parte media del ciclo 2 3 das antes del
comienzo de la oleada; aumenta E2 nh Prog y 17 alfaHP El aumento de P
refleja el proceso de luteinizacin de las granulosas que adquirieron receptores
para LH y capacidad de de biosntesis de 17 OHP y P LH y FSH aumentan en
oleadas (LH duplica en 2hs) y se asocian al pico de E2 y aumento rpido de P
La oleada de LH dura 48hs y se asocia con disminucin de E2 y 17 OHP pero
aumento sostenido de nhibina Sigue una meseta en pico de gonadotrofinas que
dura 14hs y nivelacin de P La disminucin de LH se acompaa de un nuevo
aumento de P y disminucin de 17 OHP E2 e nh 36hs Luego del inicio de la
oleada de LH Los niveles de nh Tambin dependen de la produccin del
folculo preovulatorio y el cuerpo lteo inicial El intervalo preciso entre el
comienzo de la oleada de LH y la ovulacin es un tanto incierto; sugirindose que
la misma ocurre 1 2hs antes de la fase final del aumento de Progesterona o 35 a
44 hs Luego del comienzo de la oleada de LH FASE LUTEA Cambia la
dominancia de E2 a los de P La luteinizacin de las tecales y granulosa luego
de la ovulacin implica gran actividad de enzimas esteroideognicas P 450 en las
clulas lteas y aumento P y menos E2 Los niveles pico de E2 y P constituyen
la ventana de 3 das durante la cual el endometrio secretor conduce a la
implantacin Si no ocurre implantacin se produce la letolisis con disminucin
rpida de P E2 nh durante los ltimos 4 5 das finales de la vida del cuerpo
lteo La actividad secretora y vida funcional del cuerpo lteo dependen del
respaldo de LH La FSH llega a sus niveles ms bajos por accin de la nh E2
y Progesterona FASE MENSTRUAL El crecimiento folicular del ciclo siguiente
depende de la regresin del cuerpo lteo La baja de nh y aumento de FSH
ocurren 2 das antes del comienzo de la menstruacin resultado de la reactivacin
del sistema GnRH y gonadotrofinas

Bibliografa: DIAGNSTICO Y TRATAMIENTO GINECO-OBSTETRICO. H. DE CHERNEY. MANUAL MODERNO. EDICIN 7. 1999. PG. 148-151.


FIN DEL CASO CLNICO SERIADO

21/05/13 18:36 Simulador Proedumed
Pgina 1 de 1 http://www.proedumed.com.mx/simulador/pages/examen/resultadoPregunta.faces
Anlisis del Caso Clnico
Identificacin del reactivo
Area: GINECOLOGA Y OBSTETRICIA
Especialidad: GINECOLOGA
Tema: AFECCIONES BENIGNAS Y MALIGNAS DEL UTERO
Subtema: HIPERPLASIA ENDOMETRIAL Y CNCER DE ENDOMETRIO
CASO CLNICO CON UNA PREGUNTA

MUJER DE 48 AOS DE EDAD, CON DIAGNSTICO ACTUAL DE CNCER ENDOMETRIAL EN ETAPA I.

ELEMENTOS CLAVE A CONSIDERAR EN EL CASO CLNICO:
Edad: mUJER DE 48 AOS. LO HABITUAL ES QUE SEA MAYOR DE 55
AOS. ES RARO A ESTA EDAD.
Antecedentes: -
Sintomatologa: -
Exploracin: rECUERDA QUE LA ETAPA i SE LIMITA AL ENDOMETRIO. T1a: el
cncer se encuentra en el endometrio (recubrimiento interior del tero) y
puede que haya crecido a travs del endometrio a menos de la mitad de
la capa muscular subyacente del tero (el miometrio). T1b: el cncer ha
crecido del endometrio al miometrio, creciendo a travs de ms de la
mitad de la capa miometrial. El cncer no se ha propagado ms all del
cuerpo del tero.
Laboratorio y/o gabinete: -

79 - EL TRATAMIENTO INDICADO PARA ESTA PACIENTE SER:

HISTERECTOMA
MS
RADIOTERAPIA
OPCIONES DE TRATAMIENTO PARA EL CANCER DEL ENDOMETRIO EN
LA ETAPA II El tratamiento del cncer del endometrio en la etapa IIA es
generalmente una combinacin de terapias, incluida radioterapia interna y
externa y ciruga. Etapa IIA El tratamiento del cncer del endometrio en la
etapa IIA puede incluir lo siguiente: Histerectoma (ciruga para extirpar el
tero)y salpingooforectoma bilateral (ciruga para extirpar ambos ovarios y las
trompas de Falopio). Los ganglios linfticos en la pelvis y el abdomen pueden
extirparse tambin para su examen bajo un microscopio para verificar la
presencia de clulas cancerosas. Histerectoma y salpingooforectoma
bilateral, con o sin remocin de ganglios linfticos en la pelvis y el abdomen,
ms radioterapia interna o radioterapia de haz externa en la pelvis. Despus de
la ciruga, puede colocarse un cilindro plstico que contiene una fuente de
radiacin en la vagina a fin de eliminar todas las clulas cancerosas restantes.
Etapa IIB El tratamiento del cncer del endometrio en la etapa IIB puede incluir
lo siguiente: Histerectoma (ciruga para extirpar el tero)y salpingooforectoma
bilateral (ciruga para extirpar ambos ovarios y las trompas de Falopio),
remocin de los ganglios linfticos en la pelvis y el abdomen para su examen
bajo un microscopio con el propsito de verificar la presencia de clulas
cancerosas, seguido por radioterapia. Radioterapia interna y radioterapia de
haz externa, seguida por histerectoma y salpingooforectoma bilateral y
extirpacin de ganglios linfticos en la pelvis y el abdomen para su examen
bajo un microscopio con el propsito de verificar la presencia de clulas
cancerosas. Histerectoma radical (ciruga para extirpar el cuello uterino, el
tero, las trompas de Falopio, los ovarios y parte de la vagina) con o sin
remocin de los ganglios linfticos en la pelvis para su examen bajo un
microscopio con el propsito de verificar la presencia de clulas cancerosas.
HISTERECTOMA
RADICAL MAS
RADIOTERAPIA Y
HORMONOTERAPIA
OPCIONES DE TRATAMIENTO PARA EL CANCER DEL ENDOMETRIO EN
LA ETAPA III El tratamiento del cncer del endometrio en la etapa III puede
incluir lo siguiente: Histerectoma radical (ciruga para extirpar el cuello uterino,
el tero, las trompas de Falopio, los ovarios y parte de la vagina), remocin de
los ganglios linfticos en la pelvis para su examen bajo un microscopio con el
propsito de verificar la presencia de clulas cancerosas, seguida por
radioterapia interna y radioterapia de haz externa. Radioterapia solamente para
pacientes que no pueden someterse a ciruga. Terapia hormonal para
pacientes que no pueden someterse a ciruga o radioterapia.
HISTERECTOMA
ABDOMINAL CON
SALPINGO-
OOFORECTOMA
BILATERAL
OPCIONES DE TRATAMIENTO PARA EL CANCER DEL ENDOMETRIO EN
LA ETAPA I El tratamiento del cncer del endometrio en la etapa I puede incluir
lo siguiente: Histerectoma (ciruga para extirpar el tero)y salpingooforectoma
bilateral (ciruga para extirpar ambos ovarios y las trompas de Falopio). Los
ganglios linfticos en la pelvis y el abdomen pueden extirparse tambin para su
examen bajo un microscopio para verificar la presencia de clulas cancerosas.
Histerectoma y salpingooforectoma bilateral, con o sin remocin de ganglios
linfticos en la pelvis y el abdomen, seguidas por radioterapia interna o
radioterapia de haz externa en la pelvis. Despus de la ciruga, puede
colocarse un cilindro plstico que contiene una fuente de radiacin en la vagina
a fin de eliminar todas las clulas cancerosas restantes.
QUIMIOTERAPIA,
RADIOTERAPIA Y
HORMONOTERAPIA
ADYUVANTE.
OPCIONES DE TRATAMIENTO PARA EL CANCER DEL ENDOMETRIO EN
LA ETAPA IV El tratamiento para el cncer del endometrio en la etapa IV
puede incluir lo siguiente: Radioterapia interna y radioterapia de haz externa.
Terapia hormonal. Estudios clnicos de quimioterapia.

Bibliografa:GINECOLOGIA DE NOVAK. BEREK S. JONATHAN. MC GRAW-HILL. INTERMERICANA. EDICIN 12. 1996. PAG.
1123.



21/05/13 18:36 Simulador Proedumed
Pgina 1 de 1 http://www.proedumed.com.mx/simulador/pages/examen/resultadoPregunta.faces
Anlisis del Caso Clnico
Identificacin del reactivo
Area: GINECOLOGA Y OBSTETRICIA
Especialidad: OBSTETRICIA
Tema: ENFERMEDADES QUE COMPLICAN EL EMBARAZO
Subtema: INFECCIN DE VAS URINARIAS Y CERVICOVAGINITIS DURANTE
EL EMBARAZO
CASO CLNICO SERIADO

MUJER DE 29 AOS, GESTA 1 CON EMBARAZO DE 25 SEG QUE ACUDE A CONTROL PRENATAL. REFIERE SECRECIN VAGINAL ABUNDANTE Y PRURITO VULVAR. A LA
EXPLORACIN SE ENCUENTRA SECRECIN VAGINAL AMARILLENTA, ESPUMOSA, FTIDA, ERITEMA VULVAR, VAGINAL Y CERVICAL CON APARIENCIA DE "FRESA".

ELEMENTOS CLAVE A CONSIDERAR EN EL CASO CLNICO:
Edad:
Antecedentes: EMBARAZO 25 SDG
Sintomatologa: SECREC N VAG NAL ABUNDANTE Y PRUR TO VULVAR
Exploracin: SECREC N VAG NAL AMAR LLENTA ESPUMOSA FT DA
ER TEMA VULVAR VAG NAL Y CERV CAL CON APAR ENC A DE
"FRESA"
Laboratorio y/o gabinete:

80 - LO MS PROBABLE ES QUE LA INFECCIN CRVICO VAGINAL DE ESTA PACIENTE SEA CAUSADA POR:

CNDIDA
ALBICANS
En la candidiasis genital se presenta prurito y ardor vulvar vulvodinea sensacin de
quemadura dispareunia y sntomas urinarios (disuria polaquiuria y tenesmo) Hay
presencia de flujo variable de aspecto de leche cortada (en requesn) o blanco
grisceo eritema vulvar eritema vaginal test de aminas negativo y pH vaginal 4 4
0 7 Tambin se acompaan lesiones descamativas exulceraciones y lceras
(secundarias a rascado) y rara vez lesiones costrosas despulimiento de la mucosa
edema o congestin intensa RECUERDA QUE EL ASPECTO DE LA LEUCORREA
POR CND DA ES BLANQUES NO EN GRUMOS ESPESO NO CORRESPONDE A
LA CLN CA DE LA PAC ENTE
CLAMIDIA
TRACOMATIS
En la mujer embarazada la infeccin por C trachomatis puede ser responsable de
cervicitis ruptura temprana de membrana bajo peso del recin nacido aborto y
muerte del neonato que puede infectarse al pasar por el canal del parto Se reporta
que el 50% de los nios nacidos de madres infectadas por C trachomatis presenta
conjuntivitis y el 20 % neumona LA MAYOR PARTE DE LAS NFECC ONES POR
CLAM D A DURANTE EL EMBARAZO SON AS NTOMT CAS EN LA MAYORA DE
LOS CASOS LA SOSPECHA V ENE TRAS UNA RUPTURA DE MEMBRANAS
PREMATURA
TRICOMONA
VAGINALIS
En la tricomoniasis la leucorrea puede ser variable en cantidad de color amarillento
verdoso o gris inodora o de olor fuerte Otros sntomas son el prurito vulvar ardor y la
irritacin genital dolorosa que puede llegar a provocar intensa dispareunia A la
exploracin vaginal se observa crvix con aspecto de fresa esto por dilatacin capilar
y las hemorragias puntiformes generalmente el endocervix no muestra afeccin y si
es as se encuentra una cervicitis mucopurulenta LEUCORREA AMAR LLENTA
ESPUMOSA Y FT DA CON CERV X EN FRESA ES EL CUADRO CLN CO
CLS CO DE LA NFECC N POR TR COMONA VAG NAL S
NEISSERIA
GONORREAE
La infeccin por Neisseria Gonorreae se caracteriza por disuria polaquiuria leucorrea
purulenta amarillenta o amarillo verdosa: la localizacin endocervical es la ms
frecuente e importante de las gonococias bajas Puede presentarse bartholinitis
unilateral generalmente con dolor en uno de los labios mayores y tumefaccin La
vulvovaginitis es muy frecuente en las prepberes embarazadas y
postmenopusicas LO HAB TUAL ES QUE LOS SNTOMAS CUANDO EX STEN
SEAN MN MOS Y MUCHAS VECES SE PRESENTE COMO GONORREAS
AS NTOMT CAS

Bibliografa: WILLIAMS OBSTETRICS. CUNNINGHAM/LEVENO/BLOOM/HAUTH/GILSTRP/WENSTROM. MC GRAW-HILL. EDICIN 22. 2005. PAG. 1319.



81 - EL TRATAMIENTO DE ELECCIN PARA ESTE TIPO DE PACIENTE ES:

FLUCONAZOL El tratamiento para Candida albicans incluye uno de los siguientes esquemas:
traconazol 200 mg VO cada 212 horas por un da o Fluconazol 150 mg VO
dosis nica o Clotrimazol 100 mg va vaginal dos tabletas diarias por tres das
EL FLUCONAZOL EST ND CADO COMO MANEJO DE SEGUNDA LNEA
EN NFECC ONES POR CAND DA ALB CANS
BENCILPENICILINA La bencilpenicilina parenteral sigue siendo el medicamento de eleccin en el
tratamiento de la siflis en todas las fases La sfilis en mujeres embarazadas
debe tratarse con penicilina a dosis adecuadas a la fase de la enfermedad El
retratamiento en los embarazos satisfactorios no es necesario en ausencia de
indicios clnicos o serolgicos de nueva infeccin Cuando las embarazadas con
sfilis son alrgicas a las penicilinas las autoridades de los Estados Unidos
recomiendan la hospitalizacin y la desensibilizacin EL USO DE
BENC LPEN C L NAS EST ND CADO PARA EL TRATAM ENTO DE LA
SF L S DURANTE Y FUERA DEL EMBARAZO
ERITROMICINA La eritromicina es de acuerdo a la OMS un antibitico recomendado para el
tratamiento de la Chlamydia trachomatis en el embarazo Tiene efectos
secundarios significativos principalmente nuseas y vmitos La amoxicilina
demostr ser igual sino ms efectiva que la eritromicina LA ER TROM C NA
CONST TUYE EL PR NC PAL TRATAM ENTO RECOMENDADO EN LA
ACTUAL DAD PARA LA NFECC N POR CHLAMYD A EN EL EMBARAZO
METRONIDAZOL De acuerdo a la NOM Para la prevencin y control de las infecciones de
transmisin sexual Para el tratamiento de la Trichomona vaginalis debe
utilizarse uno de los siguientes esquemas: Metronidazol 2 g VO en una dosis o
Metronidazol 500 mg VO cada 12 horas por siete das nvestigaciones
publicadas por la OMS revelan que el metronidazol es un frmaco efectivo para
el tratamiento de la tricomoniasis durante el embarazo y que registra tasas de
cura parasitolgica de alrededor del 90%; sin embargo puede aumentar el
riesgo de parto prematuro EL METRON DAZOL ES EL TRATAM ENTO DE
ELECC N PARA LA TR COMON AS S VAG NAL

Bibliografa: WILLIAMS OBSTETRICS. CUNNINGHAM/LEVENO/BLOOM/HAUTH/GILSTRP/WENSTROM. MC GRAW-HILL. EDICIN 22. 2005. PAG. 1319.


FIN DEL CASO CLNICO SERIADO

21/05/13 18:37 Simulador Proedumed
Pgina 1 de 1 http://www.proedumed.com.mx/simulador/pages/examen/resultadoPregunta.faces
Anlisis del Caso Clnico
Identificacin del reactivo
Area: GINECOLOGA Y OBSTETRICIA
Especialidad: GINECOLOGA
Tema: LESIONES BENIGNAS Y MALIGNAS DE LA MAMA
Subtema: CNCER DE MAMA
CASO CLNICO SERIADO

MUJER DE 32 AOS DE EDAD, PRIMIGESTA, QUE CURSA ACTUALMENTE CON SU PRIMER EMBARAZO DE 33 SEMANAS DE GESTACIN. ACUDE A CONSULTA DE
URGENCIA AL PERCATARSE DE TUMORACIN EN MAMA DERECHA Y CAMBIO EN LA COLORACIN DE LA PIEL. A LA EXPLORACIN SE ENCUENTRA MAMA DERECHA
NDULO DE 2.5 CENTMETROS DE DIMETRO EN EL CUADRANTE SUPERIOR EXTERNO, CON BORDES MAL DEFINIDOS, FIJO E INDOLORO.

ELEMENTOS CLAVE A CONSIDERAR EN EL CASO CLNICO:
Edad: Mujer de 32 aos de edad con mayor riesgo de presentar cncer de
mama
Antecedentes:
Sintomatologa: En ocasiones se retrasa el diagnstico atribuyndole a los cambios
propios del embarazo la sintomatologa mamaria eso explica que
muchas veces el diagnstico de cncer se retrase durante este periodo
Exploracin: gualmente el comienzo del estudio de una masa en mama o ndulo
mamario se retrasa al atribuirle al embarazo la aparicin de estos
cambios En este caso la masa tiene todas las caractersticas de
malignidad
Laboratorio y/o gabinete: El abordaje de la masa o ndulo mamario no debe retrasarse en la
paciente embarazada

82 - EL DIAGNSTICO CLNICO MS PROBABLE ES:

MASTALGIA
CCLICA.
La frecuencia de dolor mamario es de 66 % y es mayor en las mujeres con
menopausia que en las mujeres jvenes La causa precisa de la mastalgia se
desconoce pero probablemente tiene relacin con los cambios gobernados por
los estrgenos y la progesterona en el contenido de agua intersticial y por lo
tanto de la presin intersticial La mastalgia por lo general se clasifica como
cclica y no cclica La mastalgia no cclica a menudo es circunscrita y no tiene
relacin con el ciclo menstrual La causa ms frecuente de la mastalgia
circunscrita es un quiste simple pero algunos cnceres de mama se acompaan
de dolor mamario focal Por consiguiente esta molestia requiere de la valoracin
clnica detallada con estudios de imagen y una biopsia con aguja de cualquier
anomala palpable La paciente no presenta dolor por lo cual se descarta esta
opcin de respuesta
CNCER DE
MAMA.
La mama es un rgano nico en el sentido de que existe como primordial durante
10 aos o ms antes de entrar en un estadio altamente proliferativo durante la
menarquia y no madura por completo sino hasta despus de nacer el primer hijo
El epitelio mamario inmaduro es ms sensible a los carcingenos que el epitelio
que se forma despus de la lactancia Por lo tanto entre ms se retrasa el primer
hijo nacido vivo mayor es el riesgo de padecer cncer de mama En relacin con
la nuliparidad si el primer hijo nace antes de los 28 aos de edad el riesgo de
cncer mamario es menor pero si nace despus el riesgo es mayor El riesgo de
padecer cncer mamario disminuye en las mujeres que tienen a su primer hijo
nacido vivo jvenes y en las que tienen un mayor nmero de hijos nacidos vivos
El encontrarse embarazada la paciente no descarta la posibilidad de cncer
mucho menos cuando las caractersticas de la masa son notoriamente de
malignidad La asociacin de cncer de mama y embarazo se define como la
aparicin de un tumor maligno mamario en la gestacin o durante el primer ao
posparto La frecuencia global oscila entre el 0 2 al 3 8 % del total de los tumores
malignos de la mama El cncer de mama se diagnostica por trmino medio en 1
de cada 3000 gestaciones Los procesos oncolgicos que con ms frecuencia se
asocian a la gestacin son el cncer de mama y el cncer invasor de cuello
uterino representando cada uno el 25 % del total de cnceres y embarazo A
distancia y en orden decreciente de frecuencia aparecen otras neoplasias como;
los melanomas cncer de ovario cncer tiroideo leucemias y linfomas y los
tumores malignos de origen colorectal y seos Los medios convencionales para
el diagnstico de los tumores mamarios presentan las mismas indicaciones para
la mujer gestante que para la no gestante con algunas modificaciones en cuanto
a la sensibilidad de las mismas que dependen de las modificaciones anatmicas
de la glndula mamaria durante el embarazo Es fundamental en la embarazada
la exhaustiva exploracin clnica durante las primeras semanas de su gestacin y
se considera mala praxis no emplear todas las tcnicas de diagnstico incluidas
las punciones para la obtencin de material histolgico; la frecuencia de aparicin
de fstula lctea es despreciable
MASTOPATA
FIBROQUSTICA.
Mastopata Fibroqustica Concepto: Conjunto de alteraciones histolgicas tanto
proliferativas como involutivas tanto del tejido granular de la mama como del
tejido de sostn del mismo (tej conectivo) como resultado de una alteracin "No"
tumoral Desde el punto de vista clnico tampoco es exacto llamar Enfermedad
a una entidad que slo produce sntomas incapacitantes entre 5 % y 10 % de las
pacientes y que se observa clnicamente en 50 % de las mujeres e
histolgicamente en 90 % Se presenta entre la adolescencia y la menopausia
Generalmente es de carcter bilateral y difuso Mayor incidencia en mujeres
nulparas Durante embarazo y lactancia se produce un estado de reposo
Alteraciones y manifestaciones durante el ciclo menstrual (indicador) Etiologa:
Como en muchos otros trastornos en los que se desconoce su causa la MFQ
tiene una etiologa multifactorial y rara es la vez que se deba a una sola causa de
las que se exponen Entre las causas tenemos: nestabilidad hormonal: En
general se ha encontrado un disbalance entre estrgenos y prolactina (que
aumentan) y progesterona (que disminuye) Alteracin del metabolismo de los
andrgenos Trastorno tiroideos Metilxantinas: Sustancia del t caf
chocolate o bebidas de cola (todos alimentos estimulantes del SNC) Teora
alrgica: Se cree que a travs de sustancias vasoactivas de los mastocitos se
produce alteraciones fibroqusticas Las caractersticas de la masa descartan esta
posibilidad diagnstica
FIBROADENOMA. Tumor benigno que predomina en mujeres jvenes entre los 15 y 45 aos de
edad Clnicamente se trata de una masa de consistencia elstica muy mvil y de
bordes bien delimitados la mayora de las veces nica pero puede ser mltiple
hasta en 15 % de los casos; alcanza tamaos promedios de 3 cm con tendencia
a permanecer de igual tamao Hasta un 30 % pueden desaparecer con el tiempo
y otro porcentaje disminuir de tamao Los fibroadenomas no complejos tiene un
riesgo similar al de la poblacin general de padecer cncer de mama No se
conoce la etiologa pero debido a que la proliferacin lobular es una respuesta a
estimulacin estrognica se supone que es hormonodependiente Otras
caractersticas hacen pensar que el fibroadenoma tiene un origen hormonal son
el hecho de que desaparezca en la menopausia y la circunstancia de que puedan
aumentar de tamao en mujeres que inician reemplazo hormonal Las
caractersticas de la masa descartan esta posibilidad diagnstica
independientemente de si la paciente se encuentra embarazada o no

Bibliografa:OBSTETRICIA Y MEDICINA PERINATAL. SAMUEL KARCHMER K. COLEGIO MEXICANO DE ESPECIALISTAS EN GINECOLOGA Y OBSTETRICIA.
EDICIN 1RA. 2006. PG. 401-405.



83 - EL TRATAMIENTO CONSISTE EN:

INDICAR AINES. La paciente no presenta dolor se descarta un proceso inflamatorio per se por lo
que no estara indicado en ningn caso el uso del A NES en la paciente El
objetivo debe ser realizar el diagnstico de la masa y con base en ello realizar el
tratamiento especfico
SUPRESIN DE
METILXANTINAS.
Los tratamientos ms tiles contra la MPQ son de tipo emprico y mejoran pero no
curan la enfermedad: Supresin de metilxantinas (muy efectivo): Dietas pobres
en metilxantinas (caf te chocolate y bebidas carbnicas) producen mejoras en
un 60 % de los casos Tratamiento hormonal: Progesterona (el ms efectivo): Se
da en forma de gel y se debe aplicar durante 2 a 3 meses para que se alcancen
niveles teraputicos de progesterona No corresponde a una medida til ante la
alta sospecha de cncer
REALIZAR
TUMORECTOMA

MASTECTOMA.
Es fundamental considerar el momento de la gestacin en el que se realiza el
diagnstico ya que las controversias ms importantes se establecen en los casos
en los que el cncer de mama aparece durante las primeras semanas de
gestacin Globalmente considerado el primer trimestre del embarazo supone
una gran dificultad a la hora del diseo teraputico; la viabilidad del embrin est
muy alejada y la interaccin de los tratamientos oncolgicos complementarios
quimioterapia y/o radioterapia sobre el desarrollo normal del embrin provoca no
pocos interrogantes entre los autores 23 La conductas se simplifican cuando la
edad gestacional permite conseguir en un plazo de tiempo breve 7 10 semanas
una viabilidad fetal suficiente como para interrumpir la gestacin Los avances de
la Neonatologa permiten un pronstico favorable de recin nacidos por encima de
las 30 semanas de gestacin Teniendo en cuenta la edad gestacional podemos
decir que los protocolos teraputicos del cncer de mama durante el embarazo
son los mismos que para el de la mujer no gestante y se basan en el estadio de la
enfermedad situacin local y estudio de extensin modificando la secuencia en
relacin a su posible interaccin con el embrin feto Debers recordar que el
tratamiento principal e incluso procedimiento de estadificacin ser el tratamiento
quirrgico pudiendo ir desde la tumorectoma hasta la mastectoma En el caso
de la paciente al contar ya con 33 semanas de gestacin la viabilidad del
producto es alta independientemente de cuando se decida la interrupcin
TAMOXIFENO. Para los casos ms graves de mastalgia cclica; se recomiendan diversos
medicamentos que son efectivos si se administran durante las ltimas dos
semanas de ciclo menstrual Estos comprenden: Danazol 200mg diarios el
modificador selectivo de los receptores estrognicos toremifeno 20mg diarios o
tamoxifeno 20mg diarios Debes considerar que la paciente esta embarazada y
que el tamoxifeno es un producto potencialmente teratognico

Bibliografa:OBSTETRICIA Y MEDICINA PERINATAL. SAMUEL KARCHMER K. COLEGIO MEXICANO DE ESPECIALISTAS EN GINECOLOGA Y OBSTETRICIA.
EDICIN 1RA. 2006. PG. 401-405.


FIN DEL CASO CLNICO SERIADO

21/05/13 18:37 Simulador Proedumed
Pgina 1 de 1 http://www.proedumed.com.mx/simulador/pages/examen/resultadoPregunta.faces
Anlisis del Caso Clnico
Identificacin del reactivo
Area: GINECOLOGA Y OBSTETRICIA
Especialidad: OBSTETRICIA
Tema: COMPLICACIONES DEL EMBARAZO Y DEL PARTO
Subtema: PARTO PREMATURO
CASO CLNICO CON UNA PREGUNTA

MUJER DE 29 AOS DE EDAD, GESTA 1, QUE INGRESA A LA UNIDAD TOCOQUIRRGICA CON BITO DE 24 SEG.

ELEMENTOS CLAVE A CONSIDERAR EN EL CASO CLNICO:
Edad: -
Antecedentes: BITO DE 24 SDG.
Sintomatologa: -
Exploracin: -
Laboratorio y/o gabinete: -


84 - LA CAUSA MS FRECUENTE DE MORTALIDAD INTRAUTERINA DE ORIGEN FETAL Y QUE DEBER CONSIDERARSE EN ESTE CASO
ES:

OLIGOHIDRAMNIOS Las causas de muerte fetal pueden ser: origen fetal (33% y para algunos
autores hasta el 45%), placentarias (30%), maternas (7%), desconocidas
(30%). Se define al oligohidramnios como la presencia de un ndice de lquido
amnitico inferior a 5 o de una mxima columna vertical inferior a 2 cm.
Podemos distinguir tres grandes grupos de etiologas de oligohidramnios. 1.
Causas fetales: Crecimiento intrauterino restringido, gestacin
cronolgicamente prolongada, infeccin fetal por citomegalovirus, obstruccin
tracto urinario (obstruccin ureteral bilateral, valvas uretrales posteriores),
patologa renal (agenesia renal bilateral, displasia renal multiqustica bilateral,
riones poliqusticos) y defectos del tubo neural. 2.Causas placentarias-
membranas: rotura prematura de membranas, 3. Causas maternas: medicacin
materna (inhibidores de la sntesis de prostaglandinas, inhibidores del enzima
convertidor de la angiotensina). LA PRINCIPAL CAUSA DE
OLIGHIDRAMNIOS ES LA FETAL Y DE STA LAS MALFORMACIONES, LO
QUE CONVIERTE AL OLIGOHIDRAMNIOS PRINCIPALMENTE EN UN
SIGNO ASOCIADO Y NO UNA CAUSA.
POLIHIDRAMNIOS El polihidramnios se define como la presencia de un ndice de lquido amnitico
mayor a 24 a 25cm el cual corresponde a ms de los percentiles 90 y 97.5. Se
asocia con malformaciones fetales, en especial del sistema nervioso central o
del tubo digestivo. En los casos de anencefalia se encuentra polihidramnios en
al menos un 50% de stos. EL POLIHIDRAMNIOS NO SE CONSIDERA UN
FACTOR PRIMARIO SINO UNA PATOLOGA ASOCIADA PRINCIPALMENTE
CON MALFORMACIONES CONGNITAS Y DE STAS, LAS DEL TUBO
NEURAL OCUPAN EL PRIMER LUGAR.
MALFORMACIONES
CEREBRALES
Las malformaciones fetales constituyen la primera causa de muerte fetal; las
ms frecuentes son las del tubo neural seguida por hidropesa, hidrocefalia
aislada y cardiopata congnita compleja. LAS MALFORMACIONES
CEREBRALES CORRESPONDE A UNA MALFORMACIN CONGNITA DEL
TUBO NEURAL QUE CORRESPONDE AL ORIGEN MS FRECUENTE DE
CAUSA FETAL DE MUERTE INTRAUTERINA.
INSUFICIENCIA
PLACENTARIA
Se entiende como insuficiencia placentaria la incapacidad de este rgano para
ejercer correctamente sus funciones de nutricin y proteccin del feto,
producindose, como consecuencia, una alteracin en la homeostasis fetal.
Los productos con hipoxia crnica pueden presentar retraso en el crecimiento
uterino y signos clsicos de sufrimiento fetal. La insuficiencia placentaria es la
principal causa de hipoxia fetal intra o anteparto, y puede provocar la muerte
intratero, o al menos un nio que ha sufrido un grave compromiso de su
oxigenacin. LA INSUFICIENCIA PLACENTARIA ES LA PRINCIPAL CAUSA
DE HIPOXIA FETAL Y NEONATAL; EL FACTOR PLACENTARIO
CONSTITUYE EL SEGUNDO LUGAR QUE DA ORIGEN A LA MUERTE
FETAL.

Bibliografa: NELSON TRATADO DE PEDIATRA. RICHARD E. BEHRMAN. MCGRAW - HILL. EDICIN 15. PAG. 542.



21/05/13 18:38 Simulador Proedumed
Pgina 1 de 1 http://www.proedumed.com.mx/simulador/pages/examen/resultadoPregunta.faces
Anlisis del Caso Clnico
Identificacin del reactivo
Area: GINECOLOGA Y OBSTETRICIA
Especialidad: GINECOLOGA
Tema: TRANSTORNOS MENSTRUALES
Subtema: AMENORREA
CASO CLNICO CON UNA PREGUNTA

ADOLESCENTE DE 15 AOS DE EDAD, SIN VIDA SEXUAL ACTIVA Y QUE ACUDE CON DIAGNSTICO DE AMENORREA SECUNDARIA.

ELEMENTOS CLAVE A CONSIDERAR EN EL CASO CLNICO:
Edad: Adolescente de 15 aos de edad
Antecedentes: Sin vida sexual activa
Sintomatologa: Amenorrea secundaria
Exploracin:
Laboratorio y/o gabinete:

85 - EL TRATAMIENTO DE ELECCIN EN STE TIPO DE PACIENTES PARA INDUCIR LA MENSTRUACIN, DEBE SER A BASE DE:

ESTROGENOS. Ocasiona proliferacin del endometrio pero no produce menstruacin
PROGESTERONA. Si la paciente no presenta signos de alteracin sistmica y hay integridad de la
v a genital se debe realizar induccin del sangrado con medroxiprogesterona (5
mg VO diarios por 5 d as) Las situaciones que se pueden presentar son las
siguientes 1 SANGRADO PRESENTE Se presentar entre 7 y 10 d as
posteriores a la administracin de progestgeno en algunas oportunidades se
puede retrasar a En caso de prolactina elevada (mayor de 20 30 ng/ml) se
debe realizar RM cerebral para descartar prolactinoma En caso de confirmarse
el diagnstico debe iniciarse el tratamiento respectivo Si la prolactina es normal
se descarta b Alteraciones tiroideas (hipotiroidismo) o c S ndrome de ovario
poliquistico el cual se corrobora por la relacin FSH LH > 3 y un eco plvico 2
SANGRADO AUSENTE Si la induccin con progestgenos no produce
hemorragia por supresin debe pensarse que no ha tenido lugar la preparacin
estrognica preliminar del endometrio Ante esta situacin se administran
estrgenos conjugados por v a oral para estimular la proliferacin del endometrio
a Si hay sangrado y los niveles de FSH estn elevados el diagnstico es falla
gonadal que puede ser debida a una insuficiencia ovrica por antecedente de
radiacin quimioterapia enfermedad autoinmune viral Si la FSH se encuentra
dentro de los valores normales hay que precisar datos positivos para anorexia
nerviosa desnutricin aguda prctica de deportes intensos y estrs en tales
casos la amenorrea es secundaria a una disfuncin hipotlamo hipfisis de
carcter transitoria b Si no hay sangrado el diagnstico es falla del rgano
terminal deben descartarse malformaciones congnitas de tero anexos y v a
genital no detectada al examen f sico
ANDRGENOS. No produce menstruacin en ste tipo de paciente
ANLOGOS DE
GN RH.
No estn indicacdos en este tipo de paciente

Bibliografa:MEDICINA DE REPRODUCCIN HUMANA. AQUILES RUIZ AYALA. EDITORES DE TEXTOS MEXICANOS. 2A.ED. 2006. PGS. 209-
210.

bvs.sld cu/revistas/end/vol9 1 98/end07198.htm


21/05/13 18:38 Simulador Proedumed
Pgina 1 de 1 http://www.proedumed.com.mx/simulador/pages/examen/resultadoPregunta.faces
Anlisis del Caso Clnico
Identificacin del reactivo
Area: GINECOLOGA Y OBSTETRICIA
Especialidad: GINECOLOGA
Tema: TRANSTORNOS MENSTRUALES
Subtema: AMENORREA
CASO CLNICO CON UNA PREGUNTA

MUJER DE 27 AOS DE EDAD, SIN VIDA SEXUAL ACTIVA, ACUDE POR PRESENTAR AMENORREA DESDE HACE 3 MESES.

ELEMENTOS CLAVE A CONSIDERAR EN EL CASO CLNICO:
Edad: Mujer de 27 aos de edad
Antecedentes: Sin vida sexual activa
Sintomatologa: Amenorrea desde hace 3 meses
Exploracin:
Laboratorio y/o gabinete:

86 - COMO PARTE DEL ABORDAJE DE LA PACIENTE, USTED DEBER SOLICITAR NIVELES SRICOS DE:

ESTRIOL, 21
HIDROXILASA Y
ANDROSTENEDIONA.
La deficiencia de la enzima 21 hidroxilasa es la causa ms comn de
hiperplasia suprarrenal congnita ocurre entre un 90 y 95 % de los casos su
diagnstico inicial se realiza midiendo los niveles de 17 hidroxiprogesterona
se puede realizar el diagnstico prenatal de esta patolog a La
androstenediona es liberada al torrente sangu neo por las clulas tecas La
funcin de esto es proveer sustratos de androstenediona para la produccin
de estrgeno en las clulas granulosas ya que estas clulas carecen de
17 20 liasa requerida para la s ntesis de androstenediona Del mismo modo
las clulas tecas carecen de la enzima aromatasa requerida para la
produccin de estrgeno Por lo tanto las clulas tecas y granulosas trabajan
juntas para formar el estrgeno No esta indicada su solicitud en este caso
5 ALFAREDUCTASA,
HORMONA
LUTEINIZANTE Y
PREGNENOLONA.
La 5 alfa reductasa esta involucrada en el metabolismo de esteroides
Participan en 3 v as metablicas La bios ntesis del cido biliar el
metabolismo de andrgenos y estrgenos y el cncer prosttico
Pregnenolona es una hormona esteroidea involucrada en la esteroidognesis
de la progesterona mineralocorticoides glucocorticoides andrgenos y
estrgenos Como tal es una pro hormona
HORMONA
FOLCULO
ESTIMULANTE,
HORMONA
LUTEINIZANTE Y
ESTRADIOL.
Al iniciar el protocolo de estudio se solicitan estas hormonas Hormona
luteinizante ste examen se solicita si la paciente est teniendo dificultad
para quedar embarazada no tiene per odos regulares o tiene signos de un
trastorno asociado con niveles anormales de hormona luteinizante (HL) En
las mujeres un incremento en los niveles de esta hormona en la mitad del
ciclo provoca la ovulacin FSH por lo regular se hace para ayudar a
diagnosticar problemas con el desarrollo sexual la menstruacin y la
fecundidad Estradiol este examen se puede hacer para ver qu tan bien
funcionan los ovarios la placenta o las glndulas suprarrenales si el ciclo
menstrual (Per odo) se ha detenido (Los niveles de estradiol var an
dependiendo del momento del mes)
HORMONA
GONADOTROPINA
CORIONICA.
Los niveles de hCG pueden medirse en la sangre o la orina Con mayor
frecuencia se hace como una prueba de embarazo destinada a indicar la
presencia o ausencia de un embrin implantado Tambin pueden hacerse
pruebas de hCG para el diagnstico o seguimiento de clulas germinales y
tumores trofoblsticos La mayor parte de las pruebas emplean un anticuerpo
monoclonal (MAb) espec fico de la subunidad de la hCG (hCG) Este
procedimiento se emplea para asegurar que las pruebas no dan falsos
positivos por confusin de la hCG con la LH y la FSH Estas dos ltimas
estn siempre presentes en diferentes niveles en el cuerpo mientras que la
presencia de hCG casi siempre indica el embarazo

Bibliografa:GINECOLOGIA DE NOVAK. BEREK S. JONATHAN. MC GRAW-HILL. INTERAMERICANA. EDICIN 12. 1996. PG.
854.



21/05/13 18:39 Simulador Proedumed
Pgina 1 de 1 http://www.proedumed.com.mx/simulador/pages/examen/resultadoPregunta.faces
Anlisis del Caso Clnico
Identificacin del reactivo
Area: GINECOLOGA Y OBSTETRICIA
Especialidad: OBSTETRICIA
Tema: HEMORRAGIAS DE LA PRIMERA MITAD DEL EMBARAZO
Subtema: EMBARAZO ECTPICO
CASO CLNICO CON UNA PREGUNTA

MUJER DE 23 AOS, TALLA 1.53 M, OBESA CON ANTECEDENTE DE CICLOS MENSTRUALES IRREGULARES Y DISMENORREA. LTIMO CICLO MENSTRUAL HACE 60 DIAS.
IVSA DESDE HACE 1 AO. ACUDE A SU CONSULTORIO POR PRESENTAR DESDE HACE 12 HORAS DOLOR EN FOSA ILIACA DERECHA QUE SE IRRADIA A LA REGION
LUMBAR Y MUSLO DERECHO Y SE ACOMPAA DE NUSEAS, VMITOS, LIPOTIMIA Y DOLOR EN HOMBRO DERECHO.

ELEMENTOS CLAVE A CONSIDERAR EN EL CASO CLNICO:
Edad: edad frtil
Antecedentes: amenorrea 60 d as vsa
Sintomatologa: dolor en fosa iliaca derecha
Exploracin:
Laboratorio y/o gabinete:

87 - EL TRATAMIENTO MS ADECUADO EN ESTE MOMENTO CONSISTE EN PRACTICAR:

OOFERECTOMA
POR
LAPAROTOMA
De acuerdo al caso cl nico el diagnstico ms probable es embarazo ectpico
En condiciones normales el blastocisto se implanta en el revestimiento
endometrial de la cavidad uterina Su implantacin en cualquier otro sitio se
considera un embarazo ectpico El sitio ms frecuente de implantacin del
embarazo ectpico es la tuba y de ella la mpula seguida de la fimbria Debido a
sto el tratamiento quirrgico est dirigido a una salpingectom a y no a la
ooferectom a
SALPINGECTOMIA
POR
LAPAROSCOPIA
El mtodo quirrgico preferido para el embarazo ectpico es la laparoscopia a
menos que la mujer se encuentre inestable en terminos hemodinmicos LA
PAC ENTE NO MUESTRA DATOS DE NESTAB L DAD HEMOD NAM CA N
ABDOMEN AGUDO POR LO CUAL NO SE CONTRA ND CA STE MANEJO
OOFERECTOMA
POR
LAPAROSCOPA
El sitio ms frecuente de implantacin del embarazo ectpico es la tuba y de ella
la mpula seguida de la fimbria Debido a sto el tratamiento quirrgico est
dirigido a una salpingectom a y no a la ooferectom a
SALPINGECTOMIA
POR
LAPAROTOMIA
Dado que la paciente no muestra datos de inestabilidad hemodinamica ni
abdomen agudo la laparotom a no es el tratamiento de eleccin en ste
momento

Bibliografa: OBSTETRICIA. CUNNIGHAM F GARY. PANAMERICANA. EDICIN 21. 2004. PAG. 765.



21/05/13 18:39 Simulador Proedumed
Pgina 1 de 1 http://www.proedumed.com.mx/simulador/pages/examen/resultadoPregunta.faces
Anlisis del Caso Clnico
Identificacin del reactivo
Area: GINECOLOGA Y OBSTETRICIA
Especialidad: OBSTETRICIA
Tema: COMPLICACIONES DEL EMBARAZO Y DEL PARTO
Subtema: ATENCIN DEL PARTO Y DISTOSIAS
CASO CLNICO SERIADO

MUJER DE 37 AOS CON EMBARAZO DE 40 SEG QUE PASA A SALA DE EXPULSIN CON DILATACIN COMPLETA Y CON PRODUCTO EN 3ER PLANO. UNA VEZ PREPARADA
LA PACIENTE SE OBSERVA SALIDA DE POLO CEFALICO PERO ALGO IMPIDE LA SALIDA DEL RESTO DEL PRODUCTO.

ELEMENTOS CLAVE A CONSIDERAR EN EL CASO CLNICO:
Edad:
Antecedentes: EMBARAZO 40SDG
Sintomatologa: D LATAC N COMPLETA Y CON PRODUCTO EN 3ER PLANO
Exploracin: AL DA DE POLO CEFAL CO PERO ALGO MP DE LA SAL DA DEL
RESTO DEL PRODUCTO
Laboratorio y/o gabinete:

88 - EL DIAGNSTICO CLNICO MS PROBABLE ES:

DISTOCIA DEL
TRABAJO DE
PARTO
La distocia del trabajo de parto se dividen en el avance menor comparado con el
normal o interrupcin del avance del trabajo de parto en la fase activa LAS
D STOC AS DEL TRABAJO DE PARTO SE DAN DURANTE LA FASE ACT VA
DEL TRABAJO DE PARTO LA PAC ENTE HA TERM NADO LA FASE ACT VA
DADO QUE T ENE D LATAC N COMPLETA No corresponde al tipo de distocia
DISTOCIA DE
PRESENTACIN
La presentacin ceflica se considera normal y ocurre en cerca del 97% de los
partos La distocia de presentacin se presenta cuando sta difiere de la ceflica
por ejemplo presentacin de nalgas plvica podlica LA PRESENTAC N
CEFL CA EN EL PRODUCTO DE NUESTRA PAC ENTE SE CONS DERA
NORMAL POR LO QUE NO CONST TUYE UNA D STOC A DE PRESENTAC N
DISTOCIA DE
PARTES
BLANDAS
La distocias de partes blandas es la dificultad del trabajo de parto originado por
alguna de las estructuras q componen alcanal de parto o las causadas por la
presencia de rganos o tejidos que al encontrarse en la pelvis materna obstruyen
y desplazan la presentacin RECUERDA QUE ANTE LA PRESENC A DE UNA
D STOC A DE PARTES BLANDAS SE HACE D FC L LA SAL DA DE LA
PRESENTAC N POR LO QUE NO CORRESPONDE STA AL CASO C TADO
DISTOCIA DE
HOMBROS
La distocia de hombros es la alteracin mecnica que se produce cuando con la
expulsin de la cabeza los hombros no se introducen en el canal del parto en una
situacin de desequilibrio entre los dimetros plvicos y el dimetro bisacromial El
problema puede afectar a cualquiera de los hombros o a ambos LA D STOC A DE
HOMBROS SE PRODUCE CUANDO TRAS LA SAL DA DE LA CABEZA FETAL
SE DET ENE LA PROGRES N DEL PARTO

Bibliografa:GINECOLOGIA Y OBSTETRICIA APLICADAS. ROBERTO AHUED / CARLOS FERNANDEZ DEL CASTILLO. MANUAL MODERNO. EDICIN 2. 2003. PAG.
599.



89 - EN ESTE MOMENTO SE DEBER INDICAR:

SINFISIOTOMA La sinfisiotoma es una operacin en la cual las fibras de la snfisis pubiana se
dividen parcialmente para permitir la separacin de la articulacin y por lo tanto el
aumento de las dimensiones pelvianas durante el parto Las indicaciones incluyen:
el fracaso del progreso del trabajo de parto cuando la cesrea no est disponible es
insegura o la madre se niega a que se le realice; y en el parto de la cabeza ltima
obstruido en un feto con presentacin podlica LA S NF S OTOMA SE
CONS DERA ACTUALMENTE UNA OPERAC N NACEPTABLE DEB DO A LAS
PERCEPC ONES DE QUE ES UN PROCED M ENTO EXTREMO QUE PUEDE
DAR LUGAR A NESTAB L DAD DE LA C NTURA PELV ANA E NCONT NENC A
UR NAR A SOLAMENTE SE ND CA EN CASOS DONDE NO HAYA OTRA
OPC N TERAPUT CA
CESREA La Maniobra de Zavanelli es la ms espectacular de las manibras ante una distocia
de hombros por que supone la reintroduccin de la cabeza en el canal del parto y la
extraccin fetal mediante cesrea Existen publicados con xito en numerosos
casos probablemente otros muchos intentos fracasados no se han publicado y
tambin hay casos en los que se describe la enorme dificultad y la morbilidad
subsiguiente Seria una maniobra extrema quizs como primera consideracin en
aquellos casos en que el desprendimiento de la cabeza fetal es parcial y apenas
alcanza a que se vea la boca cuando la relajacin uterina la reposicin y la
extraccin mediante cesrea podra ser los menos traumtico para el feto y para la
madre ES UNA MAN OBRA EXTREMA Y DEBE EFECTUARSE SLO EN CASO
DE QUE TODAS LAS MAN OBRAS N C ALES HAYAN S DO AGOTADAS
FORCEPS El forceps es un instrumento con forma de pinza destinado a tomar la cabeza fetal y
extraerla por las vas naturales El parto debe ser mecnicamente posible no debe
existir una barrera que dificulte el parto DADO QUE LA D STOC A DE HOMBROS
ES UNA ALTERAC N MECN CA DEL TRABAJO DE PARTO NO ESTN
ND CADOS LOS FORCEPS
HIPERFLEXION
LAS CADERAS
La Maniobra de McRoberts es la de primera eleccin ante la distocia de hombros
Consiste en provocar una hiperflexin de las caderas de las gestantes bien con su
propia colaboracin o con la de dos ayudantes La modificacin plvica que produce
es la apertura del ngulo lumbosacro con aplanamiento de la pelvis de unos 6 lo
que facilita el descenso del hombro posterior No produce modificaciones
apreciables en los dimetros de la pelvis en ningn nivel LA H PERFLEX N DE
LAS CADERAS ES LA MAN OBRA DE PR MERA ELECC N EN D STOC A DE
HOMBROS ASOC ADO A LA SUPRAPB CA RESUELVEN EL 60% DE LAS
D STOC AS DE STE T PO

Bibliografa:GINECOLOGIA Y OBSTETRICIA APLICADAS. ROBERTO AHUED / CARLOS FERNANDEZ DEL CASTILLO. MANUAL MODERNO. EDICIN 2. 2003. PAG.
600.



90 - LA COMPLICACIN FETAL MAS FRECUENTE DE ESTA DISTOCIA ES:

FRACTURA DE
CLAVICULA
Las fracturas de clavcula se producen por la manipulacin en especial en
aquellas maniobras en las que se ejerce una accin directa sobre las mismas
en los intentos de rotacin de los hombros Algunas se provocan como
instrumento para resolver la distocia Suelen cicatrizar sin secuelas LAS
LES ONES MECN CAS FETALES SON LA COMPL CAC N MS
FRECUENTE EN LA D STOC A DE HOMBRO DE STAS LA PR NC PAL ES
LA FRACTURA DE CLAVCULA
SUFRIMIENTO
FETAL
La hipoxia fetal con sus secuelas neurolgicas y de otros rganos est
relacionada principalmente con la condicin previa del feto y el tiempo en que
trata de resolverse la distocia NO ES LA COND C N MS FRECUENTE
PUES S LA S TUAC N PREV A FETAL ES FAVORABLE Y SE REAL ZAN
ADECUADAMENTE LAS MAN OBRAS PARA EL NAC M ENTO ES BAJA SU
NC DENC A
HEMORRAGIA
INTRAVENTICULAR
La hemorragia intracraneana es el accidente hemorrgico enceflico del recin
nacido de pretrmino o trmino que ocurre durante el parto o en los primeros
das de vida De mayor frecuencia en prematuros puede producir secuelas
neurolgicas y muerte neonatal La hemorragia periventricular e
intraventricular es la hemorragia de la matriz germinal o del tejido
subependimario a nivel del ncleo caudado y que puede extenderse a la
cavidad ventricular; ms frecuente en RN de menos de 32 semanas ES UNA
COMPL CAC N DE PRODUCTOS PREMATUROS LO QUE NO
CORRESPONDE A LA EDAD GESTAC ONAL DEL PRODUCTO DE LA
PAC ENTE NO SE RELAC ONA CON LA D STOC A DE HOMBROS
CEFALOHEMATOMA El cefalohematoma es una coleccin de sangre sub peristica causada por la
ruptura de vasos sanguneos que atraviesan desde la calota craneana al
peiostio En su patogenia esta involucrada la compresin repetida de la
cabeza fetal contra la pelvis durante un trabajo de parto prolongado o
dificultoso NO APL CA STA COMPL CAC N PUESTO QUE YA HA
SAL DO EL POLO CEFL CO

Bibliografa:GINECOLOGIA Y OBSTETRICIA APLICADAS. ROBERTO AHUED / CARLOS FERNANDEZ DEL CASTILLO. MANUAL MODERNO. EDICIN 2. 2003. PAG.
601.


FIN DEL CASO CLNICO SERIADO

Das könnte Ihnen auch gefallen